You are on page 1of 90

ELECTROCHEMISTRY Rg.

2018 - 2020

IIT–JEE Syllabus
Electrochemical cells and cell reactions, Standard electrode potentials, Nernst equation and its
relation to DG, Electrochemical Series, emf of galvanic cells, Faraday’s laws of electrolysis,
Electrolytic conductance, Specific, equivalent and molar conductivity, Kohlrausch’s law,
Concentration cells.

CONTENTS

1. THEORY 2 – 32

2. SOLVED SUBJECTIVE PROBLEMS 33 – 44

3. FOUNDATION BUILDER (OBJECTIVE) 45 – 57

4. FOUNDATION BUILDER (SUBJECTIVE) 58 – 63

5. GET EQUIPPED FOR IIT-JEE 64 – 75

6. WINDOWS TO IIT-JEE 75 – 84

7. ANSWER KEY 85 – 90

CENTERS : MUMBAI /DELHI /AKOLA /LUCKNOW /NASHIK /PUNE /NAGPUR /BOKARO /DUBAI # 1
ELECTROCHEMISTRY Rg. 2018 - 2020

1. INTRODUCTION
The connection between chemistry and electricity is a very old one, going back to Allesandro volta’s
discovery in 1793, that electricity could be produced by placing two dissimilar metals on opposite
sides of a moistened paper. Oxidation - Reduction reactions involve a transfer of electrons. Because
a flow of electrons constitutes a electric current, we can at least in principle use any redox reaction to
produce a electric current. Conversely we can use an electric current to carry out redox reactions
that do not proceed spontaneously. Electrochemical cells are the devices in which interconversion of
electrical energy and chemical energy takes place. Electro-chemical cells are of two basic types:-
(1) Galvanic cells (2) Electrolytic cells

2. GALVANIC CELLS
Galvanic cell is a device which uses a spontaneous chemical reaction to produce electricity. A typical
galvanic cell is made up to two half cell, in one of which oxidation takes place and in the other
reduction takes place.When we place a piece of zinc in a solution of copper (II) sulphate, the zinc
soon become coated with a red - brown deposit of copper, and the blue color of the solution, due to
the hydrated copper ion, fades. If enough zinc is present, the solution eventually becomes colorless.
These color changes result from the reaction in which zinc is oxidized to Zn2+(aq) while Cu2+(aq) is
reduced to copper. The sulphate ions take no part in this reaction, in which electrons are transferred
from zinc to copper. When the reaction is carried out in this way, the transfer of electrons occurs on
the surface of the zinc and cannot be directly observed.
But if the oxidation and reduction reactions are separated from each other, the electron transfer can
occur through an external circuit. We then have an electrochemical cell that is producing an electric
current.

2.1 HALF CELLS AND ELECTRODES


A piece of zinc, called the zinc electrode (a solid metal rod connected to the external circuit), is
immersed in a solution of zinc (II) sulphate, and a piece of copper, the copper electrode, is immersed
in a solution of copper (II) sulphate. The two solutions are kept apart by a porous barrier that permits
ions to move from one solution to the other but prevents the two solutions from mixing. A
conducting metal wire connects the two electrodes. At the surface of the zinc electrode, Zinc atoms
are oxidized to Zn2+ ions, which pass into solution. The electrons lost by the zinc atoms leave the
electrode and flow through the wire to the copper electrode. Here they reduce Cu2+ ions, so a
deposit of copper forms on the surface of the electrode. The electrode at which reduction occurs is
called cathode and the electrode at which oxidation occurs is called anode.The two compartments of
the cell are called half cells. The zinc electrode dipping into a zinc sulphate solution is one half-cell,
and the copper electrode dipping into a copper sulphate solution the other half - cell.
Any electrochemical cell consists of two half-cells, oxidation takes place in one, and reduction takes
place in the other. Neither half cell reaction can take place by itself, because each must be
accompanied by another half cell reaction that uses up or supplies the
necessary electrons. These electrons constitute the electric current that
flows through the external circuit. Not only must the circuit be
complete outside the cell, but it must be complete inside as well; that
is, ions must be able to move from one electrode to the other. This
movement is made possible by separating the half cells with a porous
partition (of clay pottery, for example)or a salt bridge.

CENTERS : MUMBAI /DELHI /AKOLA /LUCKNOW /NASHIK /PUNE /NAGPUR /BOKARO /DUBAI # 2
ELECTROCHEMISTRY Rg. 2018 - 2020

Salt bridge:- It is U - shaped glass tube filled with a jelly like substance, agar - agar, mixed with an
electrolytes like KCl , KNO3 , NH4 NO3 , etc. It performs following functions.
(i) It prevents voltage drops, i.e. prevents junction potential.
(ii) It allows flow of current by completing the circuit, i.e. migration of anion from anode to cathode
half cell
(iii) It prevents accumulation of charges and maintains the electrical neutrality of solutions by
intermigrations of ions into two half cell reactions.

Characteristics of electrolyte used as salt bridge:

(i) The electrolyte should be inert and should not react chemically with the electrolyte of either of the
two half cells.
(ii) The cation as well as anion of the electrolyte should have same ionic mobility and almost same
transport number, viz, KCl , KNO3 , NH4NO3 etc.

In each half-cell, we can consider the metal electrode in equilibrium with the metal ions in the
electrolyte, with the electrons residing in the metal:
Zn(s) Zn 2 ( aq)  2e  (in Zn metal)
Cu(s) Cu 2 (aq)  2e  (in Cu metal)
From the direction of the overall spontaneous reaction, we know that Zn gives up its electrons more
easily than Cu, so it is stronger reducing agent. Therefore, the equilibrium position of the Zn half-
reaction lies further to the right: Zn produces more electrons than does Cu. You might think of the
electrons in the Zn electrode as having a greater electron “pressure” than those in the Cu electrode, a
greater potential energy (referred to as electrical potential) ready to “push” the electrons through the
circuit. Close the switch and electrons flow from the Zn to the Cu electrode to equalize this electron
pressure (electrical potential) difference. The flow disturbs the equilibrium at each electrode. The Zn
half-reaction shifts to the left to remove the electrons flowing in. Thus, the spontaneous reaction
occurs as a result of the different abilities of these metals to give up their electrons and the ability of
the electrons to flow through the circuit.
Zinc atoms become ions and attract the attention of water molecules. Now the solvated Zn2+(aq) ions
diffuse away into solution, thereby exposing a new ‘workface’ of zinc atoms on the electrode.
Water molecules would be attracted by Zn2+, a cation of quite considerable charge density, in an ion-
dipole interaction.Electrons leave the zinc electrode and make their way round to the Cu/Cu2+ side of
the cell.
The electrons arrive from the zinc side of the cell, and use the copper electrode as a solid conductor
to bring them in contact with the species that will act as receivers of electrons, namely the Cu2+(aq)
ions. The exchange is completed, and the newly formed copper atoms come out of solution and
crystallize on to the solid lattice of the electrode material.
Pair of counter-currents run through the salt bridge, anions into zinc side of the cell, and cations into
the copper side. In fact the entire current path, both inside and outside the cell, can only be kept open
and flowing by the avoidance of build-ups of one sort of charge or another.

2.2 ELECTRODE POTENTIAL


If you were to place a zinc rod into a solution containing zinc ions, equilibrium would be set up
between them. There is tendency for zinc atoms on the surface of the rod to be attracted into the
solution. However, they do not enter the solution as atoms, but as zinc ions( Zn2+). In this guise they
can be solvated by water molecules. The electrons left behind when a zinc atom is transformed into a
positive ion remain on the rod. As a result the region of solution very close to the rod suffers an

CENTERS : MUMBAI /DELHI /AKOLA /LUCKNOW /NASHIK /PUNE /NAGPUR /BOKARO /DUBAI # 3
ELECTROCHEMISTRY Rg. 2018 - 2020

increase in positive charge (owing to the extra Zn2+ ions), while the rod carries a layer of negative
charge (the electrons left behind).
Metal rod Metal rod

+ + + +
+ + + +
+ + +
+ + +
+ + + +
+ + + +
+ + ++ + +
+ + + +
Solution Solution

Other metals dipping into solutions of their ions may undergo a similar but opposite change. For
them, some of the ions in the solution cling on to the metal and attract electrons out of the rod this
leaves the rod with a positive charge. Because the solution near to the rod loses positive ions, it
becomes slightly negatively charged.
An electric double layer is set up in both cases. This layer is known as the Helmholtz double layer.
If we call the metal M and we assume that it makes a positively charged ion Mn+, the same
equilibrium is involved in both cases:Mn+(aq) + ne- M(s)
The difference is that for some metals the equilibrium lies to the left (these give extra positive ions
into the solution),while for others it lies to the right (these take positive ions out of the
solution).Whenever there is a separation of positive and negative charges, we should be able to
measure a voltage. In this case there should be a voltage between the electrode and the surrounding
solution. At first sight it might seem an easy thing to measure this voltage. All we need is a voltmeter
and two pieces of wire. One piece we connect to the electrode, the other we dip into the solution.
Unfortunately this idea has a flaw in it. As soon as we dip the metal wire into the solution, another
equilibrium is set up. This time it is between the metal from which the wire is made and the ions it
gives in solution. We have introduced another Helmholtz double layer. The best we can do now is to
measure the difference in voltage between the two double layers. This is an unavoidable state of
affairs; we cannot directly measure the voltage between the Helmholtz double layer.
This is unfortunate because if we could measure the voltage it would tell us something about how
good a metal is at releasing electrons. Metals that release electrons easily are good reducing agents,
so by comparing the voltage for each metal we would be able to put them in order of their reducing
power.Given that we are bound to measure a voltage between two double layers, the most convenient
thing to do is to agree to keep one of them constant and always measure the difference between this
one and the others. The system that has been chosen as a standard is the standard hydrogen electrode.
We shall refer to this as the S.H.E.

2.3 STANDARD HYDROGEN ELECTRODE

A Standard hydrogen electrode is prepared by dipping a Platinum strip in 1M HCl around which H2
gas at 1 atm pressure bubbled. At 25°C , its standard reduction potential is defined to be zero.

CENTERS : MUMBAI /DELHI /AKOLA /LUCKNOW /NASHIK /PUNE /NAGPUR /BOKARO /DUBAI # 4
ELECTROCHEMISTRY Rg. 2018 - 2020

2.4 STANDARD CALOMEL ELECTRODE


Another reference electrode frequently used is calomel electrode. Calomel electrode is usually used
as cathode since it is well below in the e.m.f. series. At 25°C, its electrode potential (standard
reduction potential) was measured to be 0.2422volt.

2.5STANDARD ELECTRODE POTENTIALS


As discussed earlier that the potential of any electrode is the potential difference between it and the
electrolyte surrounding the electrode. The electrode potential depends upon the nature of the metal,
concentration of the metallic ions in solution and the temperature of the solution. When the ions are
at unit activity and the temperature is 25OC (298K), the potential difference is called a standard
electrode potential (E0) . The potential of a single electrode can not be determined but the potential
difference between the two electrodes can be accurately measured. Standard hydrogen electrode
(SHE), is used as the standard reference electrode. By convention, the electrode potential of SHE has
been assigned a value of zero volt.
Thus the standard electrode potential of a metal may be defined as the potential in volts developed in
a cell consisting of two electrodes , the pure metal in contact with a molar solution of its ions
andSHE.

Positive and negative electrode potentials. If on combining an electrode A with the SHE, a
reduction process (gain of electrons) occurs at the electrode, the potential of the electrode is called as
reduction potential . It is given a positive sign as reduction electrode potential or a negative sign as
oxidation electrode potential. On the other hand, if on combining an electrode B with the SHE, an
oxidation process (loss of electrons) occurs at B, the electrode potential (half - cell potential) is
called as the oxidation potential of electrode B. It is given a positive signs as oxidation electrode
potential and a negative sign as reduction electrode potential. Thus in short,
Standard oxidation potential for any half cell = - (Standard reduction potential)
Standard reduction potential for any half cell = - (Standard oxidation potential)

USES OF ELECTRODE POTENTIAL


1. It predicts relative reducing strength of reducing agents. We know that a reducing agent is a
substance which has tendency to donate electrons in chemical reactions. Hence
1
Reducing strength Tendency to donate electron  Tendency to get oxidized  E o oxid  o
E Re d
Thus higher the standard reduction potential of a species, lesser will be its reducing strength. Thus Li
having least reduction potential is the strongest reducing agent in aqueous solution, and F2 having
maximum reduction potential is the weakest reducing agent.

2. It predicts the relative activity of metals. Those elements which have a tendency to form cations are
known as metals. Hence Reactivity of a metal Tendency to form cation  Tendency to lose outer
1
electrons  E o oxid  o
E Re d

CENTERS : MUMBAI /DELHI /AKOLA /LUCKNOW /NASHIK /PUNE /NAGPUR /BOKARO /DUBAI # 5
ELECTROCHEMISTRY Rg. 2018 - 2020

Thus lesser the standard reduction potential of an element (metal), greater will be its activity.
Thus elements of group 1 having very high values of standard oxidation potential (low value of
reduction potentials) are highly reactive and hence found only in combined state.
Remember that a more active metal (lesser E o Re d or E o oxid higher) displaces less active (higher E o Re d or
lower ) metal from its salt solution.

3. It predicts whether a metal can displace hydrogen gas from a hydracid or not. Hydracids posses
hydrogen as H+ ions which are in need of electrons to get converted into hydrogen atoms which then
combine to form H2 . Hence for hydracids to evolves H2 gas , these must be treated with electron
donor element (metal). Thus only those metals which have positive value of standard oxidation
potential ( or negative value of ) will be able to donate e- to H+ of hydracids forming H2 gas.
HA  H+ + A- , M + H+(aq)  M+(aq) + H , H + H  H2(g)
Thus metals having positive value of or negative value E o Red (above H2 in the activity
series) will displace hydrogen from hydra acids and they themselves will be oxidized.
4. An oxidizing agent witha higher reduction potential will oxidize any reducing agent with a lower
reduction potential.
5. A reducing agent with a lower reduction potential will reduce any oxidising agent with a higher
reduction potential.

2.6ELECTROCHEMICAL SERIES
When elements are arranged in increasing order of standard electrode reduction potential
(or decreasing order of standard electrode oxidation potential) as compared to that of standard
hydrogen electrode, it is called electrochemical series (or electromotive force series).

Electrochemical series exhibit following characteristics.

(i) Metals near the top of the series are strongly electropositive (or weakly electronegative). They lose
electrons readily to form cations.
(ii) Weakly electronegative metals in the upper part can displace more electronegative metals below
them from their salts. For example, iron displaces copper from CuSO4 solution
(iii) Hydroxides of metals in the upper part of the series are strongly basic and their salt do not undergo
hydrolysis. On the other hand, hydroxides of metals in the lower part of the series are weakly basic
and their salts undergo hydrolysis.
(iv) Metals lying above hydrogen are easily rusted. Those situated below are not rusted.
(v) Metals above hydrogen displace hydrogen from dilute acids.
(vi) More weakly electronegative metals like K , Na , Ca, etc. displace hydrogen from water.
(vii) Iron and other metals above it decompose steam and liberate hydrogen
(viii) Oxides of iron and other metals below it can be reduced easily.
(ix) Oxides of manganese and other metals above it are reduced when heated in a current of hydrogen.
(x) Oxides of mercury and other metals below it are decomposed on heating.

Ex : 1 Can F  be oxidised to F2 by any substance listed in the electrochemical series


Ans: No

Ex : 2 Will Mg reduce CuSO 4


Ans: Yes, Mg has higher oxidation potential

CENTERS : MUMBAI /DELHI /AKOLA /LUCKNOW /NASHIK /PUNE /NAGPUR /BOKARO /DUBAI # 6
ELECTROCHEMISTRY Rg. 2018 - 2020

2.7 NOTATION FOR REPRESENTING A GALVANIC CELL


Rather than describing a galvanic cell in words, it’s convenient to use a shorthand notation for
representing the cell. For example the Daniel cell which uses the reaction
Zn(s) + Cu2+(aq)   Zn2+(aq) + Cu(s)
(i) The interfaces across which a potential difference exists are shown by a semicolon (;) or a single
vertical line (|). For example , the two half - cells of the above galvanic cell are represented as below.
Zn; Zn2+ or Zn|Zn2+ and Cu2+; Cu or Cu2+|Cu
These indicate that potential difference exists at the Zn and Zn2+ ions interface, and similarly at the
Cu 2+ and Cu interface.
Sometimes, coma or plus sign is observed in the formulation of half - cells. For example,
Ag, AgCl | Cl - or Ag + AgCl | Cl -
Theses indicate that Ag and AgCl together constitute the electrode.
 Ag | Ag+& Ag, AgCl | Cl- are not the same electrode.

(ii) The contact between two solutions by means of a salt bridge is indicated by double vertical line (||)
between them e.g. , Cu 2+ || Zn 2+
(iii) The anode half - cell ( or the oxidation half - cell ) is always written on the left hand side and the
cathode half cell ( or the reduction half - cell ) on the right side , with the respective metal electrode
on the outside extremes , e.g., Zn ; Zn2+ || Cu 2+ ; Cu or Zn | Zn2+ || Cu2+ | Cu
(iv) An arrow when drawn below the cell formulation gives the direction of the positive current inside
the cellwhile an arrow drawn above the formulation gives the direction of the electrons flown in

the outer circuit , for example, Zn ; Zn2+ || Cu 2+; Cu

(v) The potential difference between the electrodes (i.e. , the cell potential or EMF ; emf ) is stated in
volts along with the temperature at which it is applicable . For example,
E25o C  1.130 volt or E 298K  1.130 volt
(vi) The concentration of solution , pressure of gases and physical state of solids and liquids involved
are indicated in the cell formation . For example,
Pt , H2 ( 0.9 atm ) | H+ ( a = 0.1) || Cu2+ ( a = 0. 1 ) | Cu

2.8 VARIOUS TYPES OF HALF CELLS


(1) Gas-Ion Half Cell:
In such a half cell, an inert collector of electrons, platinum or
graphite is in contact with gas and a solution containing a
specified ion. One of the most important gas-ion half cell is the
hydrogen-gas-hydrogen ion half cell. In this half cell, purified
H2 gas at a constant pressure is passed over a platinum electrode
which is in contact with an acid solution.
1
H+(aq) + e- H2 (g): This half cell is represented as
2
H  H 2 Pt .

(2) Metal-Metal Ion Half Cell:


This type of cell consist of a metal M in contact with a solution
containing Mn+ ions.
Mn+(aq) + ne- M(s) ; This half cell is represented as M n+ | M

CENTERS : MUMBAI /DELHI /AKOLA /LUCKNOW /NASHIK /PUNE /NAGPUR /BOKARO /DUBAI # 7
ELECTROCHEMISTRY Rg. 2018 - 2020

(3) Metal-Insoluble Salt - Anion Half Cell:


In this half cell, a metal coated with its insoluble salt is in
contact with a solution containing the anion of the insoluble
salt.eg. Silver-Silver Chloride Half Cell:
This half cell is represented as Cl- | AgCl | Ag.
The reaction that occurs at the electrode is
AgCl(s) + e- Ag(s) + Cl-(aq)

(4) Oxidation-reduction Half Cell:


This type of half cell is made by using an inert metal collector,
usually platinum, immersed in a solution which contains two ions
of the same element in different oxidation states, eg. Fe2+ - Fe3+
half cell. The reaction that occurs at the electrode is Fe3+(aq)
- 2+
+ e  Fe (aq)
This half cell is represented as Pt | Fe3+ , Fe2+

2.9 CELL POTENTIAL


To understand the concept of standard emf of a cell, let us consider a Daniel cell in which all the ions
are present at unit concentrations as shown in the cell reaction for the cell diagram:
Zn(s) | Zn2+ (1.0 M) || Cu 2+ (1.0 M) | Cu (s), isZn(s) + Cu 2+ 
 Zn2++ Cu(s)
The above cell reaction is spontaneous, because, when a piece of zinc metal is placed in an aqueous
Cu(II) sulphate solution, metallic copper is deposited on the surface of zinc. If voltage of the above
cell is measured experimentally, at 25oC, it is found to be 1.10 V. This voltage must be related
directly to the redox reactions occurring in anode and cathode compartments. As the overall cell
reaction is the sum of two half reactions, the measured emf can also be treated as the sum of
Re duction Re d
electrical potentials at Zn and Cu electrodes. Therefore,Emf(cell) = ECathode  E Anode
 E Oxid Re d
  E Anode
Anode 
.In the above equation, both electrode potentials are now expressed as their
reduction potential.
Ex -3: Using the following standard reduction potentials determine standard emf E for each of the  
following reactions:
E Cu 2 Cu   0.15V; E Ca

2
Ca  2.76 V; E Hg 2 Hg  0.80 V
2
0 0 0
E Cl2 |Cl 
 1.36V, E I 2 |I 
 0.54V, E Al 3 |Al
 1.66V

(a) Cl2  g   2I   aq  
 I 2  s   2Cl   aq 
(b) 2Al3  aq   3Ca  s  
 2Al  s   3Ca 2   aq 
(c) Hg 22  ( aq )  2Cu  ( aq ) 
 2 Hg (l )  2Cu 2  ( aq )
Answer: (a) Cl2  g  is reduced at cathode and I   aq  is oxidized at anode
Cl2  g   2e  
 2Cl  aq  E   1.36 V
2I   aq  
 I2  s  E   0.54 V
Adding: Cl2  g   2I   aq  
 2Cl  aq   I 2  s  E   0.82 V
(b) Redox reactions involved are:
Al3  aq   3e 
 Al  s  E    1.66V ……(i)
 Ca 2  aq   2e 
Ca  s   E   2.76 V …..(ii)
CENTERS : MUMBAI /DELHI /AKOLA /LUCKNOW /NASHIK /PUNE /NAGPUR /BOKARO /DUBAI # 8
ELECTROCHEMISTRY Rg. 2018 - 2020

Multiplying Eq (i) by 2 and to (ii) 3 and finally adding gives


2Al3  aq   3Ca  s  
 2Al  s   3Ca 2   aq  E   1.10 V
Note: Multiplying a half- reaction by a constant factor does not change the E  value because it
is an intensive property.

(c) The redox reactions involved are


Hg 22  aq   2e  
 Hg  l  E   0.80 V ……(i)
 2  0
Cu ( aq )  Cu ( aq )  e E  0.15V ……(ii)
Multiplying Eq. (ii) by 2 and adding to Eq. (i) gives
Hg 22  aq   2Cu   aq   2Hg  l   2Cu 2   aq  , E 0  0.65 V

Ex- 4: Decide with a little calculation, whether each of the following reactions will proceed inforward
or backward direction at 25 C when all concentrations and pressures are unity
(a) 2Cr  3Cu 2   2Cr 3   3Cu (b) Zn  Mg 2  Zn 2  Mg
(c) Fe  Cl2  g   Fe 2  2Cl (d) 2 Fe 2   Cl2 ( g )  2 Fe3  2Cl 
(e) 2 Ag  Cu   2 Ag   Cu (f) 2Ag  Cu 2   2Cl   2AgCl  s   Cu
Solution: (a) Cr is oxidized to Cr 3  and Cu 2  is reduced to Cu , the cell giving this reaction would be
Cr Cr 3 Cu 2 Cu with Ecell  ECu 2 Cu  ECr3 Cr  0.337 V   0.774 V   1.111V

Since E  is positive, the given reaction will proceed in the forward direction cell. E cell and
conclusion regarding whether the reaction proceeds in the forward or backward direction are
tabulated below:
Ecell
Reaction Cell Direction of reaction
V
(b) Zn  Mg 2 Zn Zn 2 Mg 2 Mg 2.37   0.763  Backward

 Zn 2   Mg
(c) Fe  Cl2  g  Fe Fe2 Cl Cl2 Pt 1.3595   0.440  Forward

 Fe 2   2Cl 
(d) 2Fe 2   Cl 2  g  Pt Fe2 , Fe3 Cl  Cl2 Pt 1.3595  0.771 Forward
 2Fe 3  2Cl 
(e) 2Ag  Cu 2 Ag Ag  Cu 2 Cu 0.337  0.7991 Backward

 2Ag   Cu
(f) 2Ag  Cu 2   2Cl  Ag AgCl Cl Cu 2 Cu 0.337  0.222 Forward
 2AgCl  s   Cu

CENTERS : MUMBAI /DELHI /AKOLA /LUCKNOW /NASHIK /PUNE /NAGPUR /BOKARO /DUBAI # 9
ELECTROCHEMISTRY Rg. 2018 - 2020

2.10 THERMODYNAMICS OF CELL

Maximum work obtained


The maximum amount of non PV work Wmax, available from a process carried out at constant
temperature and pressure is equal to the free energy change ∆G for the process.
Wmax = -∆G
We can write an equation that relates the potential of a cell and the free energy of the cell reaction,
because the maximum amount of work available can be related both to the cell potential and to the
free energy change of the cell reaction
Wmax = nFEcell = -G
For our purposes, the important part of equation is the relationship between the free energy change
and the cell potential :
G = − nFECell or, for standard state conditions,
O O
G 298 = − nFE Cell
These equations relate the free energy change for a cell reaction, where the initial state is that of the
reactants in the cell and the final state is that of the products in the cell. The equations indicate that
we can find G (or Go) for a reaction if we know the potential of a galvanic cell with the same
reaction. The equation also shows that a galvanic cell spontaneously produces current when it has a
positive potential. A positive potential corresponds to a negative value of G, which is the criterion
for spontaneity of a reaction.

Adding two reactions

Remember that Eo is an intensive property. If two reactions are added, Eo cannot be just added.
However, Go for the reactions can be added.However the potentials are additive for the process in
which half- reactions are added to yield an overall reaction but they are not additive when added to
yield to third half-reaction
 G 3  G1   G 2
or  n 3FE 3   n1 FE1  n 2 FE 2   F  n1E1  n 2 E 2 
n1E1  n 2 E 2
or E 3 
n3
But, when the two half- reactions are added to yield an overall reaction, the number of moles of
electrons involved in each half-reaction and the overall reaction are necessary the same, i.e
n1  n 2  n 3  n
n E1  n E 2
 E3   E1  E 2
n
However, when two half-reactions are added to yield a third half- reaction, the numbers of moles of
electrons in all the three half- reactions cannot be the same

For Example
Fe3+ + 3e− = Fe E oFe3 | Fe (1)
Fe2+ + 2e− = Fe E oFe 2 | Fe (2)
Fe3+ + e− = Fe2+ E oFe 3 , Fe 2 | Pt (3)
(3) can be obtained by subtracting (2) from (1). Thus, we can write
G o(3) = G o(1) - G o( 2 )

CENTERS : MUMBAI /DELHI /AKOLA /LUCKNOW /NASHIK /PUNE /NAGPUR /BOKARO /DUBAI # 10
ELECTROCHEMISTRY Rg. 2018 - 2020

o o
o o o o
EFe 3
, Fe2 
 2EFe 2
/ Fe
Hence, −F E Fe 3 , Fe 2  | Pt
= − 3FE Fe 3 | Fe
+ 2FE Fe 2 | Fe
or E Fe3 | Fe

3
o o
n1 E  n2 E
In general E ooverall  1 2
; if n1& n2 are electrons participating in two reduction half
n1  n2
reactions having SRP E 1o and E o2
0
Relation between E cell and K
We saw that the standard free – energy change for a reaction is related to the standard cell
potential by the equationGo = − nFEo
In addition, we showed previously that the standard freeenergy change is also related to the
equilibrium constant for the reaction:
Go = −RT ln K
Combining these two equations, we obtain− nFEo = −RT ln K
RT 2 .303 RT
or E o  ln K  log K
nF nF
Since 2.303 RT / F has a value of 0.0592 V at 25oC, we can rewrite this equation in
the simplified form
0.0592 o
Eo  log K in volts, at 25 C
n

Ex- 5: From the following standard electrode potentials at 25 C


Cu 2  2e  Cu; E 0  0.34V
Cu 2   e  Cu  ; E 0  0.15V
Calculate equilibrium constant of the reaction Cu + 2Cu 2+ = 3Cu +
Solution: Cu  Cu 2  2e; E 0  0.34; G   2  0.34  F  0.68F
2Cu 2   2e  2Cu  ; E 0  0.15, G   2  0.15  F  0.30 F
On adding,
Cu  2Cu 2   3Cu  : G 0  0.38 F
G   2.303RT log K
0.38F  2.303RT log K
2.303RT
or 0.38  log K
F
0.38  0.0591log K
0.38
or log K    6.429  K  3.72  10 7
0.0591

Ex -6: What is K c for the following reaction at 25 C ?


Cu 2  aq   Sn 2  aq   Sn 4  aq   Cu  s 
E 0 Cu 2
| Cu
 0.34V , E 0 Sn 4
|Sn 2 
 0.15V
Solution: The cell may be represented as
Sn2+ | Sn4+ || Cu2+ | Cu E 0 cell  0.34V  0.15V  0.19V ;
2  0.19
K  10  2.69 10 6
0.0591

CENTERS : MUMBAI /DELHI /AKOLA /LUCKNOW /NASHIK /PUNE /NAGPUR /BOKARO /DUBAI # 11
ELECTROCHEMISTRY Rg. 2018 - 2020

2.11 NERNST EQUATION


Up to this point, we have focused on cell potentials determined for cells with reactants and products
in standard states (E oCell ); however, cells need not operate at standard state conditions.
We can use the equation for the free energy change of a reaction to derive a relationship that tells us
how the cell potential changes as the concentrations and/or temperature change. The free energy
change for a reaction is G = Go + RT ln Q
where Go is the free energy change for the reaction with reactants and products at standard state
activities (1 molar concentrations, 1 atmosphere pressures, pure solids or liquids), G is the free
energy change under some other set of conditions, and Q is the reaction quotient for the reaction at
the second set of conditions. Because G = − nFECell and Go = − nFEoCell, we can write
−nFECell = −nFEoCell + RT ln Q
RT
Dividing throughout by − nF gives ECell  E o Cell  ln Q
nF
For use with cells at 25 oC, the Nernst equation is often written in a special form utilizing common
(base 10) logarithms: The constant 0.05916 contains the value of RT/F and the conversion from
0.05916
natural to common logarithms. ECell  E o Cell  log Q
n
EX-7: Calculate the cell potential at 25oC of a Daniel cell in which the concentration of Zn2+ ions is
0.1 M and that of Cu2+ is 0.001 M. Eo = 1.1 V.
Solution The reaction in Daniel cell is
Zn(s) + Cu2+(aq)   Zn2+(aq) + Cu(s)
o
E = 1.1 V and the reaction quotient is
Qc 

Zn 2 

 0.1
100  E  1 .1 
0.0591
 [log 100] = 1.04 V
Cu 
2

0.001 2

Ex 8: Determine the cell reaction and E cell for each of the following cells as 25  C . Assume
activity and fugacity equal to molar concentration and pressure (atm), respectively.
(i) Mg |MgI2 (0.1M) | I2 (s) | Pt
(ii) Ag | AgBr(s)| HBr(0.02 M) | H2 (0.5 bar) | Pt
  
(iii) Fe Fe  NO3  2 0.05 mol dm 3 Cu  NO3  2 0.01mol dm 3 Cu 
 
Solution: (i) Mg MgI 2 0.1mol dm 3 I 2  s  Pt
We have

Electrode Reduction reaction Half- cell potential

RT 2
Right I 2  s   2e   2I   aq  E I  I  E I  I 
2 2
2F
 
ln  I  
 
RT 1
Left Mg 2   aq   2e   Mg  s  E Mg 2 Mg  E Mg 2 Mg  ln
2F

 Mg 2 
  
we have
Mg  s   I 2  s   Mg 2  aq   2I   aq 
RT   2     2 

E cell  EI I  EMg 2 Mg 
2

ln Mg
2F  
I
      
Substituting the data, we get
CENTERS : MUMBAI /DELHI /AKOLA /LUCKNOW /NASHIK /PUNE /NAGPUR /BOKARO /DUBAI # 12
ELECTROCHEMISTRY Rg. 2018 - 2020

 0.059 15 V 
E cell   0.535 V    2.3630 V   
 2
 log 0.1  0.2

2
 
 2.8985V  0.0708V  2.969V
(ii) 
Ag AgBr  s  HBr 0.02 mol dm 3 H 2  0.5 bar  Pt 
We have

Electrode Reduction reaction

1
Right H   aq   e   H2 g
2
Left AgBr  s   e   Ag  s   Br   aq 

The cell reaction (reduction reaction at right electrode – oxidation reaction at left electrode) is
1
Ag  s   Br   aq   H   aq   AgBr  s   H 2  g 
2
1/ 2
RT  PH 
Hence 
E cell  E 0 H 
|H 2 |Pt
 E 0 Br 
|AgBr|Ag  
F
ln  2

 H   Br  
Substituting the data, we get
 0.51 2 
E cell   0.0V  0.0713V    0.05915 V  log 
 0.02  0.02 
 
 0.0713V  0.1921V  0.2634V
(iii)  
Fe Fe  NO3 2 0.05mol dm 3 Cu  NO 3 2 0.01mol dm 3 Cu  
We have

Electrode Reduction reaction

Right Cu 2  aq   2e  Cu  s 
Left Fe 2   aq   2e   Fe  s 

The cell reaction is


Fe  s   Cu 2   aq   Fe2   aq   Cu  s 
  Fe2  
 0.05915V   
Hence E cell   ECu 2 Cu 

 EFe2 Fe
2
 log 

 2  
  Cu  
 0.05915V   0.05 
 0.337 V   0.441V     log  
 2   0.01 
 0.778V  0.0207V  0.7573V

CENTERS : MUMBAI /DELHI /AKOLA /LUCKNOW /NASHIK /PUNE /NAGPUR /BOKARO /DUBAI # 13
ELECTROCHEMISTRY Rg. 2018 - 2020

2.12 CONCENTRTION CELLS


Because cell potentials change with concentration, it is possible to set up a cell whose potential is
determined entirely by concentrations. Two connected halfcells that are identical except for their
concentrations form a concentration cell. e.g. Zn | Zn2+ (c1) || Zn2+ (c2) | Zn.
Difference between galvanic cell and concentration cell is given below.

GalvanicCells Concentrations Cells


(i) A net chemical change occures inside the cell, (i) No net chemical change occurs , e.g.
e.g. Daniel cell Zn | ZnSO4 || CuSO4 | Cu M | M+ (c1) || M+ (c2) | M
Anode: Zn   Zn 2   2 e  Anode: M   M  ( c1 )  e 
Cathode: Cu  2e  
2
 Cu Cathode: M ( c 2 )  e  

 M
Redox Change: Zn  Cu 2   Cu  Zn 2  Net : M  ( c 2 ) 
 M  ( c1 )

(ii) A decrease in free energy during the redox (ii) A decrease in free energy during the
change brings in an equivalent amount of transfer of conc. from c1 to c2 brings in an
electrical work done. equivalent amount of electrical work done

(iii) The electrode having high standard oxidation (iii) The electrode having more dilute solution
potential works as anode surroundings, works as anode.
 
(iv) Ecell 0 (iv) Ecell 0

Concentration gradient may arise either in electrode material or in electrolyte. Thus there are two types
of concentrationcell.
(i) Electrode concentration cell
(ii) Electrolyte concentration cell
Different types of concentration cells are discussed here.

Electrode Gas concentration cell:


Pt H2(P1) | H+(C) | H2(P2) Pt
Here, hydrogen gas is bubbled at two different partial pressures at electrode dipped in the solution of
same electrolyte.
Cell process :
1
H 2 ( p1 )  H  (c)  e  (anode process)
2
1
H  (c)  e   H 2 ( p2 ) (cathode process)
2
1 1
H 2 ( p1 )  H 2 ( p2 )
2 2
 2.303RT  p  0.059 p 
E   log  2 , At 25o C , E  log  1 
 2F   p1  2F  p2 

CENTERS : MUMBAI /DELHI /AKOLA /LUCKNOW /NASHIK /PUNE /NAGPUR /BOKARO /DUBAI # 14
ELECTROCHEMISTRY Rg. 2018 - 2020

 Can you derive the condition for spontaneity of such cell?


Another example of the electrode concentration cell is that an amalgam with two different
concentrations of the same metal dipped in same electrolyte solution
The cell is represented as
Hg  Pb  c1 M  PbSO4  c M  Hg  Pb  c2 M 
At cathode: Pb 2  c   2e  
 Pb  c2 
At anode:  Pb 2   c   2e
Pb  c1  
_____________________
Pb  c1  
 Pb  c2 
Since, the concentration of Pb 2  for the two half cells is same as the electrolyte solution for the two
compartments is same, so the net reaction is independent of the electrolyte concentration. Applying
equation to the net cell reaction gives
0.059 c2
 
Ecell 25 C  E Pb

2

 c  Pb c2   E Pb 2 Pb c1   2 log c
1
0.059 c
 
Ecell 25 C  Ecell


2
log 2
c1
0.059 c

Ecell 25 C   2
log 2
c1

(since , Ecell  0)

The net cell reaction would be spontaneous, when the EMF of the cell is positive, which is possible
only when c1  c2 .

Electrolyte concentration cells:


Zn(s) | ZnSO4 (c1) || ZnSO4 (c2) | Zn(s)

In such cells, concentration gradient arises in electrolyte solutions. Cell process may be given as,

Zn(s)  Zn2+(c1) + 2e- (Anodic process)

Zn 2 (c2 )  2e   Zn( s )
(Cathodic process)
Zn 2 (c2 )  Zn 2 (c1 )

From Nernst equation, we have


2.303 RT C  2.303 RT C 
E0 log  1  E log  2 
2F  C2  2F  C1 

 Can you derive the condition for spontaneity of such cell?

Ex - 9: Calculate the EMF of the electrode concentration cell


Hg  Zn  c1M  Zn 2  c M  Hg  Zn  c2 M  at 25 C , if the concentration of the zinc amalgam
are 2g per 100 g of mercury and 1g per 100 g of mercury in anode and cathode half-cell
respectively.
(A) 6.8  10  2 V (B) 8.8  10  3V (C) 5.7  10 2 V (d) 7.8  10  3 V

Solution: (B)
CENTERS : MUMBAI /DELHI /AKOLA /LUCKNOW /NASHIK /PUNE /NAGPUR /BOKARO /DUBAI # 15
ELECTROCHEMISTRY Rg. 2018 - 2020

The reactions at the two half-cells are

At cathode: Zn 2   c   2e 
 Zn  c2 
At anode: Zn  c1   Zn 2  c   2e
_______________________________________
Net cell reaction : Zn  c1    Zn  c2 
Applying Nernst equation to the net cell reaction gives
0.059 c 0.059 c
E cell  E 0 cell  log 2  log 1  since E 0 cell  0 
2 c1 2 c2
0.059  2 65.4  3
Ecell  log    8.8 10 v
2  1 65.4 

2.13 COMMERCIAL CELLS

In this section, we describe some of the electrochemical cells which are commonly employed as
sources of electrical energy. The electrochemical cell may be broadly classified into two categories
as described below.

THE PRIMARY CELLS :


Such types of cells can be used only so long the active materials are present. Once these are
consumed, the cell cannot be recharged by passage of current through it, and hence it has to be
discarded.
Leclanche or Dry Cell
One of the examples is the Leclanche or dry cell represented as
Zn| NH4Cl(20%), ZnCl2 | MnO2 | C
The reactions involved are:
Anode Zn(s) → Zn2+ + 2e−
Cathode 2MnO2(s) + H2O(l) + 2e− → Mn2O3(s) + 2OH−
Overall Zn(s) + 2MnO2(s) + H2O(l) →Zn2+ + 2OH− + Mn2O3(s)

The OH generated in the above electrochemical reaction causes the following secondary reactions:
NH4Cl + OH− → NH3 + Cl− + H2O
Zn2+ + 2NH3 + 2Cl− → Zn(NH3)2Cl2
Since these reactions are not involved directly in the electrode reactions, they do not contribute
anything towards the cell emf of 1.5 V. Since the cathode potential is a function of pH, the cell
potential falls rapidly on continuous discharge.

 Can you write down the cathode potential as a function of pH.

Ruben- Mallory cell

Another example of the primary cell which provides more constant voltage is the Ruben-Mallory
cell. Here the use of a large excess of hydroxyl ions makes the cell less sensitive to pH change. The
cell potential is approximately 1.35 V and remains constant during its life as the overall reaction does
not involve any ion in solution whose concentration can change
Hg | HgO | KOH | Zn(OH)2 | Zn

CENTERS : MUMBAI /DELHI /AKOLA /LUCKNOW /NASHIK /PUNE /NAGPUR /BOKARO /DUBAI # 16
ELECTROCHEMISTRY Rg. 2018 - 2020

The reaction involved are


Anode Zn(Hg) + 2OH−(aq) → ZnO(s) + H2O (l) + 2e−
Cathode HgO+H2O + 2e− →Hg(l) + 2OH−(aq)
Overall Zn(Hg) + HgO (s) → ZnO (s) +Hg(l)

SECONDARY CELLS

Such types of cells can be used again and again by recharging the cell.

Lead Storage cell :


One of the examples is the lead storage cell where lead acts as anode and lead impregnated with lead
dioxide acts as cathode. The electrolyte is a solution of approximately 20 per cent sulphuric acid with
a specific gravity of about 1.15 at room temperature.
The reactions involved are:
Anode Pb(s) + SO 24  (aq) = PbSO4(s) + 2e−
Cathode PbO2(s) + 4H+(aq) +SO 24  (aq) + 2e− = PbSO4(s) + 2H2O(l)
Overall Pb(s) + PbO2(s) + 2H2SO4(aq) = 2PbSO4(s) + 2H2O(l)

The emf of the cell depends on the activity of sulphuric acid in solution. At 25oC, some of the values
are 1.90 V at 7.4 per cent H2SO4, 2.0 V at 21.4 per cent H2SO4, and 2.14 V at 39.2 per cent H2SO4.
To recharge the cell, it is connected with a cell of potential higher than that of the cell in such a way
that the lead storage cell now acts as an electrolytic cell where lead is deposited on the cathode, lead
dioxide is formed at the anode and H2SO4 is regenerated. The specific gravity of the solution rises
due to the increase in H2SO4 concentration. The recharging is carried out only up to a certain specific
gravity value.

Nickel – cadmium battery


Another example of the secondary cell is the ‘nicad’ or nickel – cadmium battery. The anode and
cathode are Cd and NiO2, respectively, and the electrolytic solution is KOH. The reactions are:

Anode Cd(s) + 2OH−(aq) →CdO (s) + H2O + 2e−


Cathode 2Ni(OH)3(s) + 2e− →2Ni(OH)2(s) + 2OH−(aq)
Overall Cd(s) + 2Ni(OH)3(s) → CdO(s) + Ni(OH)2(s)+ H2O

The emf of the cell is 1.4 V. This cell has comparatively longer life than a lead storage cell and can
be packed in a sealed unit. This cell finds its much application in electronic calculators and electronic
flash.

A few other example of secondary cells are:


(1) Edison cell (anode —iron, cathode—nickel oxide, electrolyte—KOH solution).
(2) Silver-Zinc cell (anode—zinc, cathode—silver oxide, electrolyte—KOH solution)
(3) Silver-Cadmium cell (anode—cadmium, cathode—silver oxide, electrolyte—KOH solution)
The term battery is used wherever a number of cells are connected in series or series-parallel.

Fuel cell
Fuel cells are another means by which chemical energy may be converted into electrical energy.
Here the cathode and anode constituents are continually supplied and thus energy can be withdrawn
indefinitely from a fuel cell as long as the outside supply of fuel is maintained. One of the examples
is the hydrogen-oxygen fuel cell. The cell consists of three compartments separated from one another
by porous electrodes. The hydrogen gas is fed into one compartment and the oxygen gas is fed into

CENTERS : MUMBAI /DELHI /AKOLA /LUCKNOW /NASHIK /PUNE /NAGPUR /BOKARO /DUBAI # 17
ELECTROCHEMISTRY Rg. 2018 - 2020

another compartment. These gases then diffuse slowly through the electrodes and react with an
electrolyte that is in the central compartment. The electrodes are made of a conducting material, such
as graphite, with a sprinkling of platinum to act as a catalyst, and the electrolyte is an aqueous
solution of a base. The reactions are:
Anode H2(g) + 2OH−(aq) →2H2O(l) + 2e−
Cathode O2(g) + 2H2O(l) + 4e− → 4OH−(aq)
Overall 2H2(g) + O2(g) → 2H2O(l)
One of the advantages of the fuel cell is that the energy is extracted from the reactants under more
nearly reversible conditions. This type of cell finds great importance in space vehicles.

2.14 APPLICATION
(1) DETERMINATION OF THERMODYNAMIC DATA
The free energy change accompanying a cell reaction can be obtained from the emf of the cell
through the relation.∆G= −nF ECell
The changes in enthalpy and entropy of the cell reaction can also bedetermined if the
temperature dependence of the emf of the cell is available. For this, we make use of the Gibbs-
Helmholtz equation
 (G) 
G  H  T  Now Since ∆G= −nF E
 T  p
 (G)   E 
we, therefore, get  T    nF  
 T  p
Substituting we get
p

 E    E  
 nFE  H  nFT   Rearranging, we have H   nF  E  T   
 T  p   T  p 

Thus, ∆H of the cell reaction can be determined from the knowledge of E and (E/T)p.Since E and F
are expressed respectively in volt and coulomb, the unit of ∆H is in volt-coulomb and is equal to the
joule unit.
H   G
Now since ∆G = ∆H − T∆S, therefore S 
T
 E 
Substituting the expressions of ∆H and ∆G, we get S  nF  
 T  p
Thus knowing (E/T)p, ∆S of the cell reaction can be computed

 E 
  is known as temperature coefficient of cell.
 T 

(2) Determination of charge of ions :Charges of certain doubtful ions may be determined using e.m.f.
measurement. The method was proposed in 1889 by Ogg. He first of all employed the method on

Hg | Hg2(NO3)2 (C1) || Hg2 (NO3)2 (C2) | Hg

Where C1> C2; using Nernst equation, we get


o  0.059 1 V  C
ECell   log 1 wheren = valency of ions
 n  C2
when C1 / C2 = 10. then e.m.f. was experimentally found to be 0.0295 volt.

CENTERS : MUMBAI /DELHI /AKOLA /LUCKNOW /NASHIK /PUNE /NAGPUR /BOKARO /DUBAI # 18
ELECTROCHEMISTRY Rg. 2018 - 2020

0.0591
Thus 0.0295  log [10] n = 2
n
Hence, charge of mercurous ion is (+2). The ion may be represented as Hg22+

(3) Determination of degree of hydrolysis: E.m.f. measurement helps to determine the degree of hydrolysis
accurately. Example of cell used for this purpose is
Pt (H2 1 atm) | NH4C1 || KCl (sat), Hg2Cl2 | Hg
E.M.F. of the cell may be determined as,
E =ECalomel - 0.0591 log [H+] ... (i)

Hydrolysis of NH4CI salt may be represented as


NH4+ + H2O  NH4OH + H+ or
At t=0 C 0 0
At equilibrium C(l -  ) C C 

[H+] = C  ...(ii)
From Eqs. (i) and (ii), we get
E = ECalomel - 0.0591 log (C  )
 can be calculated using E value & ECalomel value
(4)Determination of ionic product of water :Water undergoes ionization as.
H2O → H+ + OH-
KW = [H+][OH-] (i)
The cell used for the purpose is
Pt (H2 1 atm) | KOH (C1) | | KC1 (C2) , AgCl | Ag
Cell reaction may be represented as
1
At anode H 2 (g)  H   e
2
At cathode AgCl(s) + e Ag(s) + Cl-

1
Over all reaction H 2 ( g )  AgCl  H   Cl   Ag ( s )
2

Thus, using Nernst equation we have ,


0.0591
E  E0  log  H    Cl   (ii)
1   


From Eq. (i) we have  H  
Kw 
E  E0 
0.0591
log
 
Cl  K w
(iii)
 
OH    1  
OH 

E0 of the cell under consideration is 0.2222 volt ; [C1-] = C2 and [OH-] = C1


Eq. (iii) gives us
C K 
E= 0.2222 - 0.0591 log  2 W 
 C1 
Substituting the e.m.f. Kw may be calculated.

CENTERS : MUMBAI /DELHI /AKOLA /LUCKNOW /NASHIK /PUNE /NAGPUR /BOKARO /DUBAI # 19
ELECTROCHEMISTRY Rg. 2018 - 2020

(5) Determination of solubility of sparingly soluble salts:


Solubility of sparingly soluble salts like BaSO4, PbSO4 and AgCl may be determined using e.m.f.
measurement. Let us consider the determination of solubility of AgCl; The cell for the purpose may be
represented as
Ag | AgNO3(0.01N) || KCl (0.01N) Saturated with AgCl|Ag
0.0591  0.01
n is 1 for this cell. Using Nerst equation. we get E log   (i)
1  C2 

Where C2 is concentration of [Ag+] furnished by AgCl in cathodic chamber. Let solubility of AgCl is
‘S’ mole per litre was in waterthen its solubility product will be ‘S2’
Ksp (solubility product) = [Ag+] [Cl-] ; [Ag+] = C2
[Cl- ] = 0.01 (furnished by ionization of KC1)
KSP = C2 × 0.01 and S2 = C2  0.01 (ii)
Thus, From eq. (i) and (ii), we get
0.01
or E = 2 x 0.0591 log
s
Substituting the value of e.m.f. we may calculate solubility.
EX- 10: Find the solubility product of saturated solution of Ag2CrO4 in water at 298 K if the emf of
the cell Ag| Ag+ (satd. Ag2CrO4 soln.) || Ag+ (0.1 M)| Ag is 0.164 V at 298 K.
0.059 [ Ag  ]RHS 0.059 0.1
Sol: ECell  log10 
0.164  log
1 [ Ag ]LHS 1 [ Ag  ]LHS
1.66  10 4
[ Ag  ]LHS  1.66  10 4 M So, [CrO 42  ] 
2
 1.66  104 
K sp ( Ag2CrO4 )  [ Ag  ]2 [CrO42 ] 
 1.66  10  4  2

2

 
= 2.287  10−12 mol3 L−3

3. ELECTROLYTIC CELLS
An electrolytic cell uses electrical energy to drive a non spontaneous reaction (  G >0). In the cell
reaction, electrical energy from an external power supply converts lower energy reactants into high
energy products. The electrode connected to the positive pole of the electricity supply is called the
anode and that connected to negative pole is called cathode. At anode oxidation takes place and at
cathode reduction takes place. Conventionally the current flows from cathode to anode, but the actual
flow of electrons is in opposite direction.

3.1ELECTROLYSIS
The process of decomposition of an electrolyte on passing an electric current through its aqueous
solution or in the fused state is called electrolysis. An electrolyte (AB) when dissolved in water or
when melted dissociates to produce corresponding ions (e.g. A+ and B-). When the circuit is
completed by closing key, following reactions occur at the two electrodes.
(a) The cations move towards cathode. On reaching cathode, they gain electrons (supplied by
battery) and thus become neutral atoms.(At cathode) A+ + e-→ A (Reduction)
(b) The anions move towards anode and on reaching the anode they lose electrons and become
neutral atom (Atanode)
B - → B + e- ( Oxidation)

CENTERS : MUMBAI /DELHI /AKOLA /LUCKNOW /NASHIK /PUNE /NAGPUR /BOKARO /DUBAI # 20
ELECTROCHEMISTRY Rg. 2018 - 2020

Thus electrons from the source (battery) enter the solution (where they are taken up by cations) at the
cathode and leave the solution at the anode. As a result, flow of electricity continues
Thus electrolysis involves oxidation (de - electronation of the anion) at the anode, and reduction
(electronation of cation) at the cathode.
Note that the above cases are very simple where only dissociation of the salt is considered which is
true when the electrolyte is taken in molten state. However, when the aqueous solution of the
electrolytes is taken, water also dissociates to a small extent as
H2O H+ + OH−
Thus in such cases two different cations (H+ and A+ ) and two different anions ( OH-and B- ) are
present in solution. Of the two cations, that cation will be liberated at the cathode which has higher
reduction potential. Similarly, of the two anions, that anion will be discharged at the anode which
has lower reduction potential. The process of electrolysis is further complicated when the
electrodes are attacked by the ions of the electrolytes.

(i) Electrolysis of aqueous sodium chloride solution using platinum electrodes:In the solution,
sodium chloride dissociates completely while water dissociates to a small extent.
NaCl Na+ + Cl - (almost completely dissociated)
H2O H+ + OH - (dissociated to a small extent)
When electricity is passed , H ions ( although present in much smaller amount than Na+ ions)
+

having lower discharge ( or higher reduction) potential than Na+ ions are liberated at the cathode.
The discharge potential of an ion is defined as the minimum potential that must be applied across the
electrodes to bring about the electrolysis and subsequent discharge of the ion on the electrode.
H+ + e- → H
H + H → H2 
Thus hydrogen gas is liberated at the cathode.
Cl- ions having lower discharge (or lower reduction) potential than OH-ions,due to overvoltage for O2
liberation, will be liberated at the anode.
Cl -  Cl + e- Cl + Cl  Cl2 
Thus chlorine gas is liberated at the anode.
(ii) Electrolysis of aqueous copper sulphate solution using platinum electrodes. The dissociation of
the two compounds occurs as below
CuSO4 Cu2+ + SO42 (almost completely dissociated)
H2O  H+ + OH (dissociated to a small extent)
On passing electricity, Cu ions having a lower discharge potential than H+ions, will be liberated at
2+

cathode,in preference to H+ ions.


Cu2+ + 2e - →Cu
OH ions, having a lower discharge potential than SO42 ions, will be liberated at anode in preference
to SO42 ions.
OH → OH + e
4OH  2H2O + O2
(iii) Electrolysis of aqueous copper sulphate solution using copper electrodes. In this case, reaction at
cathode is similar to that discussed above , i.e. Cu2+ + 2e - → Cu
However, reaction at anode is complicated. Following three reactions are possible at anode.

(a) 2 SO42  S 2O82   2e  E o   2.0 volt



(b) 4 OH  2 H 2O  O2  4 e  E o   1 .2 volt
(c) Cu(from anode) → Cu 2+ + 2e Eo = - 0.34 vol

CENTERS : MUMBAI /DELHI /AKOLA /LUCKNOW /NASHIK /PUNE /NAGPUR /BOKARO /DUBAI # 21
ELECTROCHEMISTRY Rg. 2018 - 2020

Of the above three reactions , the last one involving the dissolution of copper electrode to form Cu 2+
ions has the lowest discharge potential and hence will take place in preference to that other two. The
net result will thus be the destruction (dissolution) of copper anode.

 In general , the least active ion ( ions having lower discharge potential) is discharged first at the
respective electrode. The decreasing order of activity or the increasing order of deposition of few
ions is as below.
For cations: Li+ , K+ , Na+ , Ca2+ , Mg2+ , Al3+ , Zn2+ , Fe2+ , Ni2+ , H+ , Cu 2+ , Hg2+ , Ag+ ,Au3+
Thus in aqueous solution containing any of the cation Li+ , Na+ , K+ , Ba2+ , Ca2+ , Mg2+ , or Al3+ , it is
water which is reduced at the cathode and not the metal cations.
For anions : SO 42  , NO 3 , OH  , Cl  , Br  , I 

 Thus, in aqueous solution of the SO 42 and NO3 , it is the water which is oxidised at the anode and
not the anions.
(i) The phenomenon of electrolysis appears only at the electrodes
(ii) The electrodes as a whole remains neutral during the process of electrolysis as equal number
of charges are neutralized at the electrodes.
(iii) The deposition of different ions at the electrodes take place only for the time electricity is
passed and stops as soon as electricity is switched off.
(iv) In case two or more types of positive and negative ions are present in the solution , the least
active ion is discharged first at the respective electrode.
(v) If electrode is active, metal goes on depositing on cathode and at anode metal is dissolved.

3.2FARADAY’S LAW
Laws of Electrolysis :The laws which govern the deposition of substances ( in the form of ions) on
electrodes with passage of electric current are called Faraday’s law of electrolysis.

1.Faraday’s first law: The mass of an ion discharged during electrolysis is proportional to the quantity of
electricity passed. Mathematically, WQ
where , W = Mass of ions librated in gm , Q = Quantity of electricity passed in coulombs.
But we know that Q = current in amperes (I)  Time in sec (t)
 W  I  t or W = Z  I  t
where Z is a constant , known as electrochemical equivalent (ECE) of the ion deposited. When a
current of 1 ampere is passed for 1 second ( i.e. Q = 1 ) , then W= Z
In case current efficiency ( ) is given , then W = Z It  /100
Thus electrochemical equivalent may be defined as the mass of the ion deposited by passing a
current of one ampere for one second ( i.e. by passing coulomb of electricity). Its unit is gram
per coulomb.
1 F of charge = Charge on one mole of electron
= NA  e = 6.023  10 23  1.602 x 10 -19 = 96514.8 C  96500 C
= The charge which discharges E g of ion, where E is equivalent weight of ion
Since 96500 C discharges E g of ion
E
1 C discharges = g of ion = Z
96500
Now substituting the value of Z in the reaction
EIT W It Q Q
W  ZIT  or   
96500 E 96500 96500 F

CENTERS : MUMBAI /DELHI /AKOLA /LUCKNOW /NASHIK /PUNE /NAGPUR /BOKARO /DUBAI # 22
ELECTROCHEMISTRY Rg. 2018 - 2020

2.Faraday’s second law:When the same quantity of electricity is passed through different electrolytes, the
masses of different ions librated at the electrodes are directly proportional to their chemical
equivalents weights. Suppose W1 and W2 are the weights of the elements deposited by passing
a certain quantity of electricity through their salt solutions and E1 and E2 are their respective
equivalent weights, then
W1 E Z1 It E Z E
 1 or  1 (W=ZIT)  1  1
W2 E2 Z 2 It E 2 Z2 E2
Thus the electrochemical equivalent (Z) of an element is directly proportional to its equivalent
weight (E),
i,e. E  Z or E = FZ

Where F is again a proportionally constant and has been found to be 96540 coulombs. It is
called faraday. Thus E = 96540  Z

Therefore , when 96540 ( or roughly 96500) coulombs of electricity is passed through an


electrolyte, one gram equivalent of its ions is deposited at the respective electrode. This quantity
of electricity which liberates one gram equivalent of each element is called one faraday and is
denoted by F.
 1 Faraday = 96500 Coulombs

Mass deposited or liberated No. of coulombs passed


(a) No. of gm.eq. = 
Eq. wt . 96500
Current in amperes  Time in Sec

96500
(b) One gram equivalent of a substance is deposited or liberated on passing 1 mole of
electrons
( 1 faraday or 96500 coulombs)
(c) One faraday ( 96500 coulombs) is the quantity of charge present on one mole of
electrons, i.e.
= 6.023  10 23 electrons
No. of electrons passed through
No. of faraday =
6.023  10 23
(d) No. of faradays required for depositing 1 gm atomic weight of an element at cathode
is equal to the charge on its cation. For examples,Mn+ (aq) ion
requires n Faradays for depositing one gram atom at cathode.
M n+ (aq) + ne - → M(s)
1 mole n mole 1 mole atom
1 mole n Faradays 1 gm atom

From the above stoichiometry, it is evident that in the reduction of n valent cation, n moles of
electrons or n Faradays or n  96500 coulombs are involved . Therefore, 1 faraday or 96500 C
or 1 mole of electrons (e-) will reduce :
1
(A) 1 mole of monovalent cation (B) or mole of divalent cation
2
1 1
(C) or mole of trivalent cation (D) or mole of n valent cations
3 n

CENTERS : MUMBAI /DELHI /AKOLA /LUCKNOW /NASHIK /PUNE /NAGPUR /BOKARO /DUBAI # 23
ELECTROCHEMISTRY Rg. 2018 - 2020

EX -11: Find the strength of current that will liberate 5.60 L of O2 at NTP from acidulated water in 3h
Solution: 5.6 L of O2 at NTP = 5.6 / 5.6 g eq. = 1.0 g equivalent
 1 g equivalent is liberated by 96500 C of electricity,
Q 96500
 I   8.8 A.
t 3  60  60

EX -12: The electrolysis of 250 mL of a brine solution was carried out for a period of 20.0 min with a
current of 2.0 A. If the resulting solution was titrated with 0.62 M HCl,what volume of acid
would be required to achieve neutral point?
Solution: Quantity of electricity passed
2400
 2  20  60  2400 C  F
96500
24
 Gram equivalent of NaOH produced = = gram equivalent of HCl needed
965
24 V
  0.62   V  40 mL
965 1000

EX - 13: How many hours can a calculator drawing 5  10−4 watt be operated by a mercury battery
having a cell potential 1.34 V if a mass of 1.0 g HgO is available at cathode?
1 96500
Solution:1.0 g of HgO  g equivalent  C
108 .6 108 .6
96500 -4
Therefore, total energy requirement = C  1 .34 V = 5.34 x 10 x t
108 .6
 t  619.38 hr
EX- 14: How many amperes must be passed through a Down’s cell to produce Na-metal at a rate of 30
kg/hr?
30000
Solution: No. of geq of Na metals required to be produced per hour is =  1304.34
23
 1304.34 F of electricity is required per hour.
Q 1304 .34  96500
 I    34965 A
t 3600
EX- 15 : Under suitable conditions, nitrobenzene can be reduced electrolytically to N-phenyl
hydroxylamine asC6 H5NO2   C6H5NH(OH)
In an experiment, a 0.2 g sample containing some nitrobenzene and remaining inert impurity,
were electrolyzedusing a current of 100 mA. It took 6.0 min for reducing the entire nitrobenzene
present in the sample into N- phenylhydroxyl amine. Determine the mass percentage of
nitrobenzene in sample.
6  60  0.1 36
Solution :Current passed   F = g equivalent of nitrobenzene.
96500 96500
1 36 9
 Moles of nitrobenzene   
4 96500 96500
9
 Mass   123  0.01147 g  5.74 %.
96500
EX 16: In a zinc manganese dioxide dry cell, the anode is made up of zinc and cathode of a carbon rod
surrounded by amixture of MnO2, carbon, NH4Cl and ZnCl2 in aqueous base. The cathodic
reaction may be represented as
2MnO2(s) + Zn2+ + 2e−   ZnMn2O4(s)
Let there be 8 g MnO2 in the cathodic compartment. How many days will the dry cell
continue to give a current of 4.0 mA?
CENTERS : MUMBAI /DELHI /AKOLA /LUCKNOW /NASHIK /PUNE /NAGPUR /BOKARO /DUBAI # 24
ELECTROCHEMISTRY Rg. 2018 - 2020

Solution :When MnO2 will be used up in cathodic process, the dry cell will stop to produce current.
4 3
2 MnO 2 ( s )  Zn 2   2e    Zn Mn 2 O4
Molecular mass 87
Equivalent mass of MnO 2  =  87
Change in oxidation state 1
From first law of electrolysis, weight of electrolyte discharged
It
  Equivalent Weight
96500
4  10 3  t  87 2218390.8
8 t = 2218390.8 s =  25.68 day
96500 3600  24

4.CONDUCTANCE
Substances which allow electrical current to pass through them are known as conductors or
electrical conductors. On the other hand, substances which to not allow electricity to pass through
them are known as non - conductors or insulators. Common examples of non - conductors are rubber,
wood, paper, all non - metals except carbon (graphite), etc.
Conductors , can be divided into two types , viz (i) metallic conductors , and (ii) electrolytic
conductors.

Comparison of metallic and electrolytic Conductors.


Metallic ( Electronic) conductors , e.g. Electrolytic conductors(electrolytes)e.g.
metals, graphites , certain minerals. solutions of electrolytes

1. Flow of electricity takes place 1. Flow of electricity takes place with the
without the decomposition of the decomposition of the substance
substance. (electrolyte).

2. Flow of electricity is due to the flow 2. Flow of electricity is due to the


of electrons, i.e. there is no flow of movement of ions and hence there is
matter. flow of matter.

3. The electrical conductance decreases 3. The electrical conductance increases


with increase of temperature. with increase of temperature.

Thusanelectrolyte may be defined as that substance which in the form of its solution or in its fused
state conducts electricity and simultaneously undergoes chemical decomposition. On the contrary
substances which in the form of their solutions or in their molten (fused) state do not conduct
electricity are called non - electrolytes , e.g. all organic compounds.The movement of ions gives
rise to what is known as the electrolytic conduction. The latter, thus, depends on the mobility of
ions and anything that inhibits the motion of ions causes resistance to current flow. Factors that
influence the electrical conductivity of solutions of electrolytes include interionic attraction,
solvation of ions and viscosity of solvent; these factors depend on the attraction of solute-solute,
solute-solvent and solvent-solvent, respectively. The average kinetic energy of the solute ions
increases as the temperature is raised and, therefore, the resistance of electrolytic conductors
generally decreases (i.e., conduction increases) as the temperature is raised.

CENTERS : MUMBAI /DELHI /AKOLA /LUCKNOW /NASHIK /PUNE /NAGPUR /BOKARO /DUBAI # 25
ELECTROCHEMISTRY Rg. 2018 - 2020

4.1 SPECIFIC CONDUTANCE


The strength of current, through the electrolytic as well as metallic conductors is governed by
Ohm’s law which states that the current strength (I) flowing through a conductor at a given
temperatureis proportional to the potential difference (E) and inversely proportional to resistance
(R).
E E
Mathematically, I  or R  ........(i)
R I
The resistance (opposition to the flow of current) of a conductor of uniform cross section is
directly proportional to its length (L) and inversely proportional to cross section (A) . It is
L L
measured is ohm. Mathematically, R  or R   ....(ii)
A A
where  is a constant , called resistivity or specific resistance and may be defined as the
resistance in ohms of a conductor of one cm length and one sq cm cross section or the resistance
between two
opposite faces of one cm cube of metal.
R. A ohm  cm 2
 Units of     ohm cm .....(iii)
L cm
The reciprocal of the specific resistance is called specific conductance. It is represented by
1
 ( kappa) . Thus,   .....(iv)

L
from equations (iii) and (iv)   …..(v)
A R
Since resistance (R) is reciprocal of electrical conductance (C) which is expressed in ohm -1 or
mho. The unit of electrical conductance is called siemen (S) . Thus equation (v) may also be
LC
represented as   … (vi)
A
Thus when L = 1 cm , A = 1 cm2 , then   C
Thus specific conductance is the conductance of 1 cm cube of the solution of an electrolyte.
L
Since = cell constant, equation (vi) may also be written as below.
A
 = cell constant  observed conductance. ......(vii)
Cell constant is determined with the help of a conductivity cell filled with N/50 solution of
chemically pure KCl. The cell constant is expressed in cm –1

4.2 EQUIVALENT & MOLAR CONDUCTANCE

Equivalent conductivityof a solution at a dilution V may be defined as the conductance of all the ions
produced from one gram equivalent of the electrolytes dissolved in V c.c. of the solution when the
distance between the two electrodes is one cm and the area of the electrodes is so large that whole of
the solution is contained between them. Equivalent conductivity is represented by  or 
Mathematically,  =   V ....(1)
where , = Specific conductivity or electrolytic conductivity
V = volume of solutions in c.c. containing 1 gm equivalent of the electrolyte.
The value of V can be calculated if the normality of the solution is known. For examples, a 5N
solution means 5 gram equivalent of it is dissolved in 1 litre or 1000 c.c.. Therefore , volume
containing 1 gram equivalent will be 1/c litre 1000/5 = 200 c.c.
1000
 In general,x N solution means, V  c.c.
x

CENTERS : MUMBAI /DELHI /AKOLA /LUCKNOW /NASHIK /PUNE /NAGPUR /BOKARO /DUBAI # 26
ELECTROCHEMISTRY Rg. 2018 - 2020

Relationship between equivalent conductance and specificconductance


Suppose the solution has a concentration of ‘c’ gram equivalent per litre. Then the volume V
1 1000
containing 1 gram equivalent of the electrolyte will be 1/c litre , i.e. V  litre  c.c
c c
1000 1000
Therefore , the equation (1) becomes      
c N
where N is normality of the solution
Units. Since the unit of electrolytic conductance is ohm - 1 cm - 1 and the unit of volume is cm 3, the
unit of equivalent conductance will be ohm- 1 cm2(gm – equiv) - 1 , as derived below.
cm3
    V  ohm 1 cm1  = ohm-1 cm2 ( g - eq) -1
g eq
Molar conductance of a solution at a dilution V is defined as the conductance of all the ions
produced from one mole of the electrolyte dissolved in V c.c. of the solution when the electrodes are
1 cm apart and the area of the electrodes is so large that whole of the solution is contained between
them. It is denoted by  or  m . Like equivalent conductance, it is related to electrolytic conductivity(
specific conductance ) as below.
1000 1000
    V or      
c M
( M is molarity of the solution) where  is the specific conductance , V is the volume of the solution
containing 1 mole of the electrolyte and c is the molar concentration, i.e. the number of moles
dissolved per litre.

Relation between equivalent conductivity and molar conductivity.

Molar conductivity
Equivalent conductivity 
Total ch arg e on cation or anion
For example , in KCl the number of charge on each ion is 1 , therefore

Molar conductivity
Equivalent conductivity   Molar conductivity
1
Molar conductivity
Similarly, in MgSO4 Equivalent conductivity 
2
EX– 17: The equivalent conductance of a 0.10 N solution of an electrolyte is 129 S cm2 eq−1 at 25 oC.
Another electrolyte, whose normality is 0.05 N, when analyzed in same conductivity cell, at
same temperaturegave a conductancevalue of 8.24 mS. Determine the equivalent
conductivity of the later solution. Resistance of the original solutionwas 85 ohms.
Solution : For original solution:
  1000 129  0.1
 eq     = 1.29  10−2 S cm−1
C 1000
L 1 L 2 −1
Also,            R  1.29  10  85 cm
 A R  A
For the solution, cell constant would remain same. Therefore, specific conductance (   ) for
the later solution:
l  1
       1.0965  8.24  10 3  9.035  10 3
 A R
 Equivalent conductance of the later solution
   1000 9.035  10 3  1000
 eq 
2 −1
  180 S cm eq
c 0.05
CENTERS : MUMBAI /DELHI /AKOLA /LUCKNOW /NASHIK /PUNE /NAGPUR /BOKARO /DUBAI # 27
ELECTROCHEMISTRY Rg. 2018 - 2020

4.3 EFFECT OF DILUTION


Both the conductivity and molar conductivity of a solution vary with concentration. The conductivity
increases with increase in concentration whereas the molar conductivity increases on dilution (i.e.
decrease in concentration). For strong electrolytes, conductivity increases sharply with increase in
concentration while for weak electrolytes it starts at lower values in dilute solutions and increases
much more gradually. In both the cases, this increase is due to the increase in the number of ions per
unit volume of the solution. For strong electrolytes, the number of ions per unit volume increase in
proportion to the concentration and that is why the increase in conductivity is very rapid. In weak
electrolytes, however, the increase in the number of ions is basically due to the change in the partial
ionization of the solute, and consequently, the conductivity increases very gradually. As stated
above, molar conductivity  mof both strong and weak electrolytes increases on dilution. The basic
reason for this is that the decrease in conductivity is more than compensated by increase in the value
of 1/c on dilution.The variation of molar conductivity on dilution for strong and weak electrolytes
shows altogether different behavior as can be seen from graph.

Strong electrolyte
Weak electrolyte

where  mhas been plotted against c . For strong electrolytes, the variation is almost linear in dilute
solutions while that for weak electrolytes, the variation is very rapid. As the molar conductivity is a
measure of the conducting power of all the ions that are available in 1 mole of the substance, it is,
therefore, obvious that the number of ions that are available for conductance increases on dilution.
Arrhenius suggested that the degree of dissociation of an electrolyte can be calculated using the

expression   c

Where c is the molar conductivity of the solution at a given concentration and  is that at infinite
dilution.

At infinite dilution complete dissociation can be assumed and  is related to that, on the other hand
 c is related to percentage dissociation at concentration C

4.4 KOHLRAUSCH LAW


The maximum ( limiting) value of equivalent conductivity ( value obtained at infinite dilution or zero
concentration) for a strong electrolyte can be obtained by extrapolating the equivalent conductance
graph to zero concentration.
However, in weak electrolytes the value of equivalent conductance at infinite dilution () or at zero
concentration (0) cannot be obtained by extrapolating the concerned graphs. The value of for weak
electrolytes is actually obtained by an indirect method based on Kohlrausch’s law.
According to this law “the equivalent conductance of an electrolytes at inifinite dilution is the sum of
two values , one depending upon the cation and the other upon the anion”
Mathematically,  = c + awhere cand a are the equivalent conductivities ( ionic conductances)
of the cation and anion respectively.Based on Kohlrausch’s law, it is possible to calculate the molar
conductivity of a weak electrolyte at infinite dilution by simply adding and subtracting  values of
appropriate electrolytes. Taking, for example, acetic acid, we have
(HAc) = (NaAc) + (HCl) – (NaCl)
= (Na+) + (Ac−) + (H+) + (Cl−) − (Na+) − (Cl−) = (H+) + (Ac−) = (HAc)

CENTERS : MUMBAI /DELHI /AKOLA /LUCKNOW /NASHIK /PUNE /NAGPUR /BOKARO /DUBAI # 28
ELECTROCHEMISTRY Rg. 2018 - 2020

o
EX- 18:Calculate molar conductivities at infinite dilution and 25 C for potash alum
K2SO4Al2(SO4)3 . 24 H2O Given
  ( K  )  73.52Scm2 mol-1
  ( Al 3 )  189Scm 2 mol-1
  ( SO42 )  160 Scm2 mol -1

Solution : m  2 ( K  )  2 ( Al 3  )  4 ( SO42  )


= 2  73.52 + 2  189 + 4  160 = 1165.04 S cm2 mol1

EX -19 : Calculate molar and equivalent conductivities at infinite dilution of the salt KOOC-COONa.
Given the ionic molar conductivities at infinite dilution are:
 C 2 O 42   = 148.5 S cm mol  K   = 50.1 S cm mol
2 −1 2 −1

 Na   = 73.5 S cm mol


2 −1
and

Solution :    
m   C2 O42    K    Na   
= 148.5 + 50.1 + 73.5, = 272.10 S cm2 mol−1

eq  m  n-factor = 2 = 136.05 S cm2 eq −1.
2

4.5 IONIC MOBILITY AND IONIC VELOCITY

Ionic Conductance and Ionic Mobility. As explained earlier, ionic conductance of an ion
(cation or anion) is defined as the contribution made by it towards the equivalent conductance of
the electrolyte at infinite dilution. These are represented by c and a
Since = c + a c = − a and a = − c

On the other hand, ionic mobility of an ion is defined as its absolute velocity ( i.e. the distance
travelled in cm/sec) under a potential gradient of one volt per cm.
ionic velocity(cm/s)
ionic mobility =
Potential difference(V) /distance between electrodes(cm)
Thus it must be noted that ionic conductance and ionic mobility are not the same thing. However,
these are found to be directly proportional to each other , i.e. ,
c u c and a u a
c = kuc and a = kua

where uc and u a are the ionic mobilities of the cation and anion respectively and k is a proportionality
constant. The value of k is always found to be 96500 which is the same thing as one faraday of
electricity , expressed in coulombs.Thus the ionic conductance and ionic mobility of an ion are
related as below.
c a
uc  and ua 
96500 96500
Ionic conductance
Thus in general , Ionic mobility =
96500
Ionic conductance is expressed in ohm - 1 cm2 while ionic mobility is expressed in cm2 Volt-1 sec- 1 .

CENTERS : MUMBAI /DELHI /AKOLA /LUCKNOW /NASHIK /PUNE /NAGPUR /BOKARO /DUBAI # 29
ELECTROCHEMISTRY Rg. 2018 - 2020

Ex-20 : For H+ the value of  0 is 349.8 . Calculate the mobility of this ion and its velocity if it is in a
cell in which electrodes are 5 cm apart and to which a potential of 2 Volts is applied
0
H0  349.8
Solution : UH    3.62 103 cm 2 volt 1 s 1
96500 96500
ionic velocity(cm/s)
ionic mobility =
Potential difference(V) /distance between electrodes(cm)
2
velocity of H   3.62 10 3   1.45 10 3 cm s 1
5

4.6 APPLICATIONS

Degree of Dissociation of a Weak Electrolyte and its Dissociation Constant


Degree of dissociation of a weak electrolyte can be obtained using Arrhenius equation

  c

Where c is the molar conductivity at the given concentration and  is the corresponding value at
infinite dilution when the weak electrolyte is present in completely ionized form.
Knowing , numerical value of the dissociation constant can be evaluated using the appropriate
expression.

Ex – 21: Specific conductance of a 0.0312 M aqueous solution of a weak base BOH is


1.53  10−4 S cm−1. Also, the equivalent conductance at infinite dilution is
236 S cm2 mol−1. Determine Kb of weak base.
  1000
Solution : Equivalent conductance at given concentration  
C
1.53  10 4  1000 2 −1
  ( 0.0312 M )   4.9 S cm eq
0 .0312
Also, since degree of ionization of weak electrolyte
c 4 .9
( )       2.07  10  2
 236
For weak base BOH: BOH  B   OH 
C C
2 2 2
C 0.0312 (2.07  10 )
Kb   = 1.37  10−5
1 1  2.07  10 2

Determination of the ionic Product of Water

The ionic product of water isKW = [H+] [OH−]


Since in pure water [H+] = [OH−]= K W
The conductivity  of pure water is related to its molar concentration by the equation
  [ H  ]  H   [OH  ] OH     K W (  H    OH  )
The ions are present in such low concentrations that the molar ionic conductivities may be taken
as the ones at infinite dilution, so that H   OH   547 .6  1 cm2 mol−1 and we obtain for KW:
2
  
KW   1 2 1 

 547.6  cm mol 

CENTERS : MUMBAI /DELHI /AKOLA /LUCKNOW /NASHIK /PUNE /NAGPUR /BOKARO /DUBAI # 30
ELECTROCHEMISTRY Rg. 2018 - 2020

Determination of Solubility and Solubility Product


The solubility of a salt in the solution can be determined from the conductivity measurements.

Since the molar conductivity of the solution is given by  
c

therefore, it follows that the concentration of the salt of the solution is given by c  …. (i)

In using this expression,  used should be that of the salt alone, the value of which can be
calculated by subtracting  of pure water from  of the solution, i.e.
(salt) = (soln.) (water)
If the salt is sparingly soluble, its solubility product can be determined from its solubility
obtained by using (i). Here  can be replaced by  as the solution will be fairly diluted i.e., it
will have very low concentration.

EX - 22: The specific conductivity of AgCl(aq) at 25oC is 1.26  10 −6 and ionic conductivity of
Ag+(aq) and Cl−(aq) atinfinite dilution are 62 S cm2 mol−1 and 76 S cm2 mol−1 respectively.
Determine solubility product of AgCl at 25 oC.
2 −1
Solution :  ( AgCl )  62  76  138 S cm mol
Since AgCl is a sparingly soluble salt, its saturated solution contain only very small amount
of Ag+ and Cl− ions, c   . Also concentration of Ag+ and Cl− ion in saturated solution is
same as solubility (s).
1.26  106  1000   1000 
 138     
S  C 
 S = 9.13  106  Ksp = S2 = 8.3  1011

4.7 CONDUCTOMETRIC TITRATION


Conductance measurements are frequently employed to find the end points of
acid-alkali and other titrations. The principle involved is that electrical
conductance depends upon the number and mobility of ions.
Consider, in the first instance, the titration of a strong acid, like hydrochloric
acid, with a strong base, like sodium hydroxide. The acid is taken in the
conductivity vessel and the alkali in the burette. The conductance of
hydrochloric acid is due to the presence of hydrogen and chloride ions. As
alkali is added gradually, the hydrogen ions are replaced by slow moving
sodium ions, as represented below :
H+(aq) + Cl–(aq) + [Na+(aq) + OH–(aq)] → Na+(aq) + Cl–(aq) + H2O(l)
(unionised)
Hence, on continued addition of sodium hydroxide, the conductance will go
on decreasing until the acid has been completely neutralized. Any subsequent
addition of alkali will result in introducing fast moving hydroxyl ions. The
conductance, therefore, after reaching a certain minimum value, will begin to
increase. On plotting the conductance against the volume of alkali added, the
points will lie on two straight lines AB and CD in figure. The point of
intersection X of these two lines gives the volume of alkali required for the
neutralization.
Suppose, it is required to titrate a weak acid like acetic acid, against a strong alkali sodium
hydroxide. The conductance of the acid will be low on account of its poor dissociation. On adding
the alkali, highly ionized sodium acetate is formed and hence the conductance begins to increase.
CH3COOH(aq) + [Na+(aq) + OH–(aq)] →CH3COO–(aq) + Na+(aq) + H2O(l) (unionised)

CENTERS : MUMBAI /DELHI /AKOLA /LUCKNOW /NASHIK /PUNE /NAGPUR /BOKARO /DUBAI # 31
ELECTROCHEMISTRY Rg. 2018 - 2020

When the acid is completely neutralized, further addition of alkali introduces excess of fast moving
hydroxyl ions. The conductance of the solution, therefore, begins to increase even more sharply than
before. On plotting the conductance against the volume of the alkali added, the two lines obtained
will be as shown in fig. The point of intersection X gives the end point.

When a mixture of a strong and a weak acid is to be titrated against a


strong alkali a combination of curves shown in above figuresis
obtained. Suppose a mixture of HCl and CH3COOH is to be titrated
against NaOH. Hydrochloric acid, being a much stronger acid, will get
titrated first. The titration of acetic acid will commence only after
hydrochloric acid has been completely neutralized. Hence, a
combination of curves as mentioned above is obtained. While the point
B corresponds to the neutralization of HCl, the point C corresponds to
the neutralization of CH3 COOH.

Finally, suppose it is required to titrate a strong acid like hydrochloric


acid, against a weak base, like ammonium hydroxide. The
conductance will fall at first due to the replacement of fast moving H+
ions by slow moving NH4+ ions.
H+(aq) + Cl–(aq) + [NH4OH] → NH4 +(aq) + Cl–(aq) + H2O(l)
After neutralization of the acid, further addition of weakly ionized
ammonium hydroxide will not cause any appreciable change in the
conductance. The curves obtained will be shown in figure.

Precipitation Titrations :
The titration of silver nitrate against potassium chloride can also be
carried out by this method. The reaction involved may be represented
as
Ag+(aq) + NO3–(aq) + [K+ + Cl–] → K+(aq) + NO3–(aq) + AgCl(s)
Since the mobility of potassium ion is nearly the same as that of silver ion
which it replaces, the conductance will remain more or less constant and will
begin to increase only after the end point. The curves obtained will be as
shown in figure.
In order to get accurate results, the change of volume during the titration
should be as small as possible. For this purpose the titration solution in the
burette is usually five to ten times stronger than the solution taken in the
conductivity vessel.
Conductometric titrations have several advantages. Coloured solutions, which cannot be titrated by
ordinary volumetric methods with the help of indicators, can be successfully titrated
conductometrically. The method can also be employed in the case of very dilute solutions and also
for weak acids and bases. Further, no special care is necessary near the end point as it is determined
graphically.

CENTERS : MUMBAI /DELHI /AKOLA /LUCKNOW /NASHIK /PUNE /NAGPUR /BOKARO /DUBAI # 32
ELECTROCHEMISTRY Rg. 2018 - 2020

SOLVED PROBLEMS
1. Consider the following galvanic cell:Pt | H2(g) (1.0 atm) | HX || KCl (1M) | Hg2Cl2(s) | Hg |Pt
Emf of cell at 25 oC is 612 mV. When HX is replaced by a buffer solution of pH 6.8, Emf was found
to be 741 mV. Find pH of HX solution.
Sol. The cell reaction is:
H2(g) + Hg2Cl2(s)   2H+(aq) + 2Cl(aq) + 2Hg(l)
The cell potential can be determined using Nernst equation as:
ECell = E oCell  0.059 log [H+] =E oCell + 0.059 pH
Now, solving the above equation under two different conditions:
0.612 V = E oCell + 0.059 pH1 0.741 V = E oCell + 0.059  6.8
Subtracting the first equation from second equation: 0.129 V = 0.4012 – 0.059 pH1
 pH1 = 4.6

2. Determine whether the following reaction are spontaneous as written, if not, give the reaction that is
spontaneous
(a)Cu(s) + 2H+(aq)   Cu 2+(aq) + H2(g) Eo(Cu 2+/ Cu) = 0.34 V
(b)3Cu(s) + 2NO 3 (aq) + 8H+ 
 3Cu2+(aq) + 2NO(g) + 4H2O
Eo(NO 3 | NO) = 0.96 V at 25oC.
Sol. (a) The half reactions involved are, Cu(S)   Cu2+(aq) + 2e−Eo = − 0.34 V (This is an oxidation
reaction)
2H+(aq)+ 2e−   H2(g) E0 = 0.00 V (conventional value)
Net: Cu(s) + 2H+(aq)   Cu 2+(aq) + H2(g); Eo = −0.34 V < 0
The above redox reaction is non-spontaneous as written. Its reverse reaction would be spontaneous.
(b) Half reactions involved are:
(i) Cu(s)   Cu2+(aq) + 2e− Eo = − 0.34 V
(ii) NO 3 (aq) + 4H+ + 3e− 
 NO + 2H2O Eo = 0.96 V
Net: 3Cu(s) + 2NO 3 (aq) + 8H+   3Cu 2+(aq) + 2NO(g) + 4H2O Eo = + 0.62 V > 0.
The cell reaction is spontaneous.

3. The emf of a cell corresponding to the reaction.


Zn + 2H+(aq)   Zn2+ (0.1 M) + H2(g) (1 atm) is 0.28 volt at 25oC. Write the half-cell reactions
and calculate the pH of the solution at the hydrogen electrode.
o
EZn 2
/ Zn
  0.76 volt and EHo  / H  0
2

o
Sol. ECell  0.76 volt
Applying Nernst equation
o 0.0591 [ Zn 2  ][ H 2 ] 0.0591 (0.1)  1 0.1 2  0.48
ECell  ECell  log  2 0.28  0.76  log  2
 log  2

2 [H ] 2 [H ] [H ] 0.0591

17.2436
log 0.1 − log [H+]2 = 16.2436 pH   8.62
2
4. Calculate the pH of the following half-cells solutions:
(a) Pt/H2 (1 atm)/ H+(HCl); E = 0.25 volt
+
(b) Pt/H2 (1 atm)/H (H2SO4); E = 0.3 volt
Sol. (a) H2   2H+ + 2e−
CENTERS : MUMBAI /DELHI /AKOLA /LUCKNOW /NASHIK /PUNE /NAGPUR /BOKARO /DUBAI # 33
ELECTROCHEMISTRY Rg. 2018 - 2020

0.0591 [ H  ]2 [H  ]
EH /H   EHo /H   log  0 .0  0.0591 log
2 2
2 PH 2 1
0.25
0.25 = 0.0591 pH pH   4.23
0.0591
0.0591 [ H  ]2 [H  ]
(b) EH /H
 EHo /H
 log = 0.0 − 0.0591 log
2 2
2 pH 2 1
0. 3
0.3 = 0.0591 pH pH =  5.076
0.0591

5. The emf of cells obtained by combining zinc and copper electrodes of the Daniel cell with
calomel electrodes are 1.083 volt and −0.018 volt respectively at 25 oC. If the reduction
potential of normal calomel electrode is +0.28 volt, find the emf of the Daniel cell.
Sol. For the cell: Zn electrode || Calomel electrode
ECell = reduction potential of calomel electrode − reduction potential of Zn electrode
So, reduction potential of Zn electrode = 0.28 − 1.083 = −0.803volt
For the cell : Cu electrode || Calomel electrode
ECell = reduction potential of calomel electrode – reduction potential of Cu electrode
So reduction potential of Cu electrode = 0.28 − (−0.018) = 0.298 volt
For the Daniel cell, Zn electrode || Cu electrode
ECell = reduction potential of copper electrode − reduction potential of Zn electrode
= 0.298 −(−0.803) = 1.101 volt

6. For the galvanic cell.


Ag | AgCl(s)| KCl (0.2 M) || KBr (0.001 M)| AgBr(s) | Ag
Calculate the emf generated and assign correct polarity to each electrode for the spontaneous process
after taking into account the cell reaction at 25oC.
Given : K sp ( AgCl )  2.8  10 10 ; K sp ( AgBr )  3.3  10 13
Sol. ECell = Ered pot. RHS electrode − ERed pot. LHS electrode
 0.0591 log
Ag  RHS  
Ag  LHS  
o o
[sin ce, E Ag / Ag 
 E Ag 
/ Ag
 0]
K sp AgBr

 0.0591 log
Br 

K sp AgCl
Cl 

3.3  1013 0.2


 0.0591 log  = −0.0371 volt
0.001 2.8  1010
The cell potential is negative; therefore, the cell reaction is non-spontaneous. For spontaneous
reaction, emf should be positive. Therefore, the correct cell reaction is
Ag | AgBr; KBr  0.001M  || KCl (0.2 M) AgCl | Ag
Anode Cathode

7. Consider the following cell:


Pt | Cr3+ (0.3 M), Cr2+ (0.01 M) || Cl (0.025 M)| AgCl(s), | Ag
For each of the following experiments, predict whether the voltage of the cell increases, decreases or
be unchanged?

CENTERS : MUMBAI /DELHI /AKOLA /LUCKNOW /NASHIK /PUNE /NAGPUR /BOKARO /DUBAI # 34
ELECTROCHEMISTRY Rg. 2018 - 2020

(a) Addition of 10g of solid AgCl to right half-cell


(b) Dissolution of 10g NaCl in the right half-cell
(c) Addition of 10 mL water to the left half-cell
(d) Addition of some soluble chromium (III) to the left half-cell
(e) Dissolution of some AgNO3 in the right half-cell
Sol. (a)Unchanged (b) Decreases (c) Unchanged (d)Decreases (e) Increases

8. A disposable galvanic cell Zn | Zn2+ || Sn2+| Sn is produced using 1.0 mL of 0.5M Zn(NO3)2 and 1 ml
of 0.5M Sn(NO3)2 .It is needed to power a pace maker that draws a current of 10-6 Amp to run it and
requires atleast 0.5 V to function. Also E0(Zn2+ | Zn) = −0.76 V; Eo (Sn2+ | Sn) = 0.14 V.
(a) What is the initial cell voltage at 298 K?
(b) What is the [Zn2+] when cell reaches 0.5 V at 298 K?
(c) What are the minimum weights of Zn and Sn electrodes you need?
(d) How long will the cell last before it needs to be replaced?
0.0592 [ Zn 2  ]
Sol. (a) E  E o  log 2
 E  E o  0.62 V
2 [ Sn ]
2+ 2+
 Initially, [Zn ] = [Cu ]
0.0592 0.5  x
(b) E  0.5 V  0.62  log
2 0.5  x
0.5  x 0.12  2
 Zn + Sn 2   Sn + Zn 2   log  4
0.5  x 0.5  x 0.5  x 0.0592
0.5  x
  10 4 or x  0 .5 M
0.5  x
 [Zn2+] = 1.0 M
(c) Zn-electrode is being consumed, therefore the amount of Zn-electrode must be greater
than 0.5 mmol = 32.5 mg. Sn is not consumed therefore there is no lower limit for weight of Sn
electrode.
(d) Almost all Sn2+ ions are consumed till the emf reaches to 0.5 V. Therefore, Sn2+consumed = 0.5
mmol = 1 meq
 Current generated = 1 mF = 96500  10 3 C = I.t
96500  10 3 6 1 1
 t 6
sec  96.5  10 sec   = 37.23 month.
10 60  60  24 30

9. In which direction can the reaction,


2Hg(l) + 2Ag+(aq)  2Ag(s) + Hg 22  (aq) proceed spontaneously at the following concentrations of
the ions participating in the reactions ?
(i) [Ag+] = 104 mol L1 and [Hg 22  ] = 101 mol L1
(ii) [Ag+] = 101 mol L1 and [Hg 22  ] = 104 mol L1
o o
Given : EHg 2
/ Hg
 0.79 V ; E Ag 
/ Ag
 0.80 V
2

2
[ Hg ] 101
Sol. (i) Q 2
 2
 4 2
 107
[ Ag ] [10 ]
o o o
E  E Ag  / Ag  EHg 2 / Hg  0.80  0.79  0.01 V
2

0 .059 0.059
E  Eo  log Q  0.01  log 10 7   0.1965 V
n 2
Negative value shows that the reaction will proceed from right to left, i.e., in backward direction.

CENTERS : MUMBAI /DELHI /AKOLA /LUCKNOW /NASHIK /PUNE /NAGPUR /BOKARO /DUBAI # 35
ELECTROCHEMISTRY Rg. 2018 - 2020

[ Hg 22  ] 10 4
(ii) Q   2
 1 2
 10 2 n=2
[ Ag ] [10 ]
Eo = 0.01 volt
0.059 0.059
E  Eo  log 10 Q  0.01  log 10  2
n 2
= 0.01 + 0.059 V = 0.069 V
Since, the value of cell potential is positive, the reaction will proceed spontaneously in forward
direction.

10. The normal oxidation potential of zinc referred to the standard hydrogen electrode is 0.76 volt and
that copper is 0.34 volt at 25oC. When excess of zinc is added to a solution of copper sulphate, the
zinc displaces copper till equilibrium is reached. What is the ratio of concentration to Zn2+ to Cu2+
ions at equilibrium?
Sol. The reaction is
Zn + CuSO4   Cu + ZnSO4
2+
or Zn + Cu   Cu + Zn2+
o 0.0591 [ Zn 2  ]
ECell  ECell  log
2 [Cu 2  ]
o 0.0591 [ Zn 2  ]
At equilibrium ECell = 0 ECell  log
2 [Cu 2  ]
o
[ Zn 2  ] 2  ECell o 2  1 .10
or log 2
 [ ECell  0.76  0.34  1.10 volt]   37 .225
[Cu ] 0.0591 0.0591
[ Zn 2  ]
 2
 1.679  10 37 : 1
[Cu ]

11. The standard electrode potentials of the electrodes Cu2+|Cu and Ag+|Ag are 0.337 and
0.799 V respectively. What would be the concentration of Ag+ ion in solution containing 0.06 M
Cu2+ ion such that both metals can be deposited together?
Sol. For simultaneous deposition of two metals present in a solution, the emf for their half-cell
must be equal, ie.
E(Cu2+ | Cu) = E (Ag+ | Ag)
0.0591 1 1

 E o Cu 2 | Cu   2
log 2
Cu 
 
 E o Ag  | Ag  0.0591 log
 Ag  
Solving ; [Ag+] = 0.37  10−8 M.

12. Copper sulphate solution (250 mL) was electrolysed using a plantinum anode and a copper
cathode. A constant current of 2 mA was passed for 16 min. It was found that after
electrolysis the absorbance of thesolution was reduced to 50% of its original value. Calculate
the concentration of copper sulphate in the solutionto begin with.
Sol. No. of faradays passed through the aqueous solution of CuSO4

I t

2  10 3   16  60 =1.98  10−5
96500 96500
Thus, no. of equivalents of CuSO4 involved in electrolysis from 1 litre solution
= 4  1.98  10−5 = 7.92  10 −5
Since, after electrolysis, the absorbance reduces to 50% hence, initial number of equivalents
of CuSO4 per litre = 2  7.92  10−5 = 1.584  10−4 [Absorbance  Concentration]
Or Normality of CuSO4 solution = 1.584  10−4 N.
CENTERS : MUMBAI /DELHI /AKOLA /LUCKNOW /NASHIK /PUNE /NAGPUR /BOKARO /DUBAI # 36
ELECTROCHEMISTRY Rg. 2018 - 2020

13. For how many minutes could a Cu/Zn cell keep a 100 watt lamp lit, assuming that one mole
of reactantsare transformed to products, and that the cell voltage is 90%of Eo?
Eo for the cellZn(s) | Zn2+ || Cu 2+ | Cu(s)is 1.10 V
Sol. 90% of Eo is 1.0 V. n = 2 (two electrons are transferred per mole ofreaction),
−1
 G = −(2 mol)  (96500 amp-sec mol )  1.0 volt = −193000 watt-s
The cell can deliver 100 watts for (193000/100) sec, or for about 32 min.

14. A constant current of 30 A is passed through an aqueous solution of NaCl for a time of 1.0 hour.
How many grams of NaOH and how many litres of Cl2(g) at STP are produced?
Sol. Q = I  t = 30  60  60 = 108000 C
108000 108000
No. of faradays passed   Mass of NaOH produced   40  44 .76 g
96500 96500
108000
 Also, volume of Cl2(g) liberated at anode   11.2 L  12.53 L
96500
15. How many atoms of calcium will be deposited from a solution of CaCl2 by a current of 25
milliamperes flowing for 60 seconds ?
Sol. Here I = 25 milliamp. = 0.025 amperes, t = 60 sec.
Q = 0.025  60 = 1.50 coulombs
Now, since, 96500 coulombs = 6.022 10 23 electrons
1.50  6.022  10 23
1.50 coulombs = electrons = 9.36  10 18 electrons
96500
Since 2 electrons are required for depositing 1 calcium atom
1  9.36  1018
9.36  1018 electrons will deposit = 4.68  1018 atoms of Ca
2
16. In a fuel cell, hydrogen and oxygen react to produce electricity. In the process hydrogen gas is
oxidised at the anode and oxygen at the cathode. If 67.2 litre of H2 at STP react in 15 minutes, what is
the average current produced? If the entire current is used for electro-deposition of copper from
copper (II) solution, how many grams of copper will be deposited?
Anode reaction : H2 + 2OH–   2H2O + 2e–
Cathode reaction : O2 + 2H2O + 2e–   4OH–
Sol. From the given reactions, it is obvious that 22.4 litres of H2 gas require 2 Faraday electricity
 67.2 litres of H2 will require = 6 Faraday electricity
Q = I t
6  96500 = I  15  60
6  96500
 I = = 643.3 ampere
15  60
Calculation of amount of Cu deposited by 6 F
63.5
Since 1 F deposits = = 31.75 g of Cu
2
6 F will deposit = 31. 75  6 g = 190 . 50 g

17. Chromium metal can be plated out from an acidic solution containing CrO3 according to the following
equation.
CrO3(aq) + 6H+ (aq) + 6e–   Cr(s) + 3H2O
Calculate (i) how many grams of chromium will be plated out by 24,000 coulombs and (ii) how long
will it take to plate out 1.5 g of chromium by using 12.5 amp current?
CENTERS : MUMBAI /DELHI /AKOLA /LUCKNOW /NASHIK /PUNE /NAGPUR /BOKARO /DUBAI # 37
ELECTROCHEMISTRY Rg. 2018 - 2020

Sol. CrO3(aq) + 6H+ (aq) + 6e–   Cr(s) + 3H2O


(i) To plate 1 mole or 52 gm of Cr 6 Faradays of electricity is used or 6  96500 coulombs of
electricity deposits 52 gm of Cr
52
 24000 coulombs will deposit =  24000 = 2.1554 gm of Cr
6  96500
2.1554 gm of Cr are plated out by 24000 coulombs
(ii) 52 gm Cr are plated by 6  96500 coulombs
96500
 1.5 gm Cr will be plated by 6   1.5 = 16701.92 coulombs
52
Q 16701.92
Q=I t t=  = = 1336.15 sec.
I 12.5

18. How many grams of silver could be plated out on a serving tray by electrolysis of a solution
containing silver in +1 oxidation state for a period of 8.0 hours at a current of 8.46 amperes? What is
the area of the tray if the thickness of the plating is 0.00254 cm? Density of silver is 10.5 g/cm3
EI t
Sol. Mass of silver deposited (w) = Z  I t w =
F
107.8  8.46  8  60  60
w= = 272. 178 g
96500
Wt 272.178
Volume of silver = = = 25.92 cc
Density 10.5
Volume 25.92 cc
Surface area = = = 10205.43 = 1.02  104 cm2
Thickness 0.00254 cm

19. Electrolysis of a solution of MnSO4 in aqueous sulphuric acid is a method for the preparation of MnO2
as per reaction.
Mn2+ (aq) + 2H2O   MnO2(s) + 2H+(aq) + H2(g)
Passing a current of 27 A for 24 hours gives only 1 Kg of MnO2. What is the value of current
efficiency?
EIt 87 I  24  60  60
Sol. We know that w = 1000 =  ( E of Mn = 87/2)
96500 2 96500
25.6
I = 25.6 ampere  Current efficiency =  100 = 94.8%
27

20. A current of 1.70 ampere is passed through 300.0 ml of 0.160 M solution of ZnSO4 for 230 sec. with a
current efficiency of 90 percent. Find out the molarity of Zn2+ after the deposition of Zn. Assume the
volume of the solution to remain constant during the electrolysis.
It (1.70 A) (230 s )
Sol. Amount of electricity passed = = = 0 . 00405 mol
F (96500C mol 1 )
 90 
Amount of electricity actually used in the reduction of Zn2+ = (0.00405)   = 0.00365 mol
 100 
0.00365
Amount of Zn2+ ions reduced = mol = 0.001825 mol
2
 1000 
Molarity of Zn2+ after the deposition of Zn = 0.160 M –  0.001825   M
 300 
= 0.154 M

CENTERS : MUMBAI /DELHI /AKOLA /LUCKNOW /NASHIK /PUNE /NAGPUR /BOKARO /DUBAI # 38
ELECTROCHEMISTRY Rg. 2018 - 2020

21. Calculate the quantity of electricity that would be required to reduce 12.3 g of nitrobenzene to aniline,
if the current efficiency for the process is 50 percent. If the potential drop across the cell is 3.0 volts,
how much energy will be consumed?
Sol. C6H5NO2 + 6H++ 6e–   C6H5NH2+ 2H2O
72 + 5 + 14 + 32 = 123
Reduction of 123 g of nitrobenzene requires 6 mole e-
6
Reduction of 12.3 g of nitrobenzene requires =  12.3 = 0.6 mole e-
123
96500
Q=  0.6 = 57900 coulombs
1
Since current efficiency is only 50%, Quantity of current required for reduction
= 2  57900 = 115800 coulombs, Energy consumed = 3  115800 volt-coulombs
= 347400 volt-coulombs = 347400 Joules = 347.4 kJ

22. An acidic solution of Cu 2+ salt containing 0.4 g of Cu2+ is electrolysed untill all the copper is
deposited. The electrolysis is continued for seven more minutes with the volume of solution kept at
100 ml and the current at 1.2 amp. Calculate the volume of gases evolved at NTP during the entire
electrolysis.
Sol. The chemical reaction taking place at the two electrodes are
At cathode: Cu 2+ + 2e–   Cu

H2O  H+ + OH–
However, note that only Cu ions will be discharged so these are present in solution and H+ ionswill
2+

be discharge only when all the Cu2+ ions have been deposited.
At anode : 2OH–   H2O + O +2e–
O + O   O2
Thus in first case, Cu2+ ion will be discharged at the cathode and O2 gas at the anode. Let us calculate
the volume of gas (O2) discharged during electrolysis.
According to Faraday’s second law 31. 75 g Cu 8 g oxygen  5.6 litres of O2 at NTP
5.6
0.4 g Cu =  0.4 litres of O2 at NTP = 0.07055 litres = 70.55 ml
31.75
As mentioned earlier, when all the Cu 2+ ion will be deposited at cathde, H+ ions will start going to
cathode liberating hydrogen (H2) gas, i.e.
H+ + e–H  H + H   H2
However, the anode reaction remains same as previous. Thus in the second (latter) case, amount of H2
collected at cathode should be calculated.
8 g of O2 1g of H2
5.6 litres of O2 at NTP = 11.2 litres of hydrogen
Quantity of electricity passed after 1st electrolysis, i.e. Q = I  t
= 1.2  7  60 = 504 coulombs
Since 96500 coulombs liberate 5.6 litres of O2 at NTP
5.6  504
504 coulombs will liberate = = 29.24 ml
96500
504
Similarly, H2 liberated by 504 coulombs = 11.2   = 58.48 ml
96500
 Total volume of O2 liberated = 70 55 + 29.24 = 99.79 ml
Vol. of H2 liberated = 58.48 ml

CENTERS : MUMBAI /DELHI /AKOLA /LUCKNOW /NASHIK /PUNE /NAGPUR /BOKARO /DUBAI # 39
ELECTROCHEMISTRY Rg. 2018 - 2020

23. A current of 3.7 ampere is passed for 6 hours between platinum electrodes in 0.5 litre of a 2M solution
of Ni(NO3)2. What will be molarity of solution at the end of the electrolysis ?
Sol. Calculation of gm eq. of nickel deposited.
Here, I = 3.7 amp., t = 6  60  60 seconds
It
Equivalents of nickel deposited is = = 0.828
F
moles of nickel deposited = (Equivalents of nickel)/2 = 0.414
No. of moles of Ni in 0.5 litre of the original solution = 2  0.5 = 1.0 mole
Since nickel deposited = 0.414 mole
Nickel left in 0.5 litre of solution = 1.0 – 0.414 = 0.586 mole
0.586  1
 Molarity of nickel = M = 1.172 M
0.5

24. Construct the cell in which following reactions are taking place.
(i) Fe + CuSO4   FeSO4 + Cu
(ii) Zn + H2SO4 
 ZnSO4 + H2
(iii) Fe + SnCl2 
 FeCl2 + Sn
1 1
(iv) H2 + Cl2   HCl
2 2
Sol. Always remember that the metal which goes into solution in the form of its salt is undergoing
oxidation and thus will act as negative electrode (i.e. anode); while the other metal acts as positive
electrode (i.e. cathode)
(i) In this, Fe is oxidised and thus will act as anode. Hence the cell will be represented as below
Fe FeSO 4 CuSO 4 Cu
(ii) In this, Zn is oxidised to ZnSO4, hence the cell will be
Zn | ZnSO4 | | H2SO4 | H2 ,Pt
(iii) In this, Fe is oxidised to FeCl2, hence the cell will be
Fe | FeCl2 | | SnCl2 | Sn
(iv) In this, H2 is oxidised, hence the cell reaction will be
Pt H2 | HCl | Cl2 ,Pt

25. Write the cell reactions and emf equations for the following cells.
(i) Zn | Zn2+ | | Cu2+ | Cu (ii) Fe | Fe2+ | |H2SO4 | H2 ,Pt
(ii) Pt, H2 | HCl | Cl2 ,Pt

Sol. (i) The reaction of oxidation half-cell is


Zn   Zn2+ + 2e–
The reaction of reduction half-cell is
Cu 2+ + 2e–   Cu
Thus the cell reaction is obtained on adding the two half-cell reactions.
Zn + Cu2+   Zn2+ + Cu
Equations for calculating the cell potential will be
0.059  Products
 
E  E0Cu 2 Cu  E0Zn 2 Zn 
2
log
 Reactants
Since the activity of the solid is taken as unity the above equation may be written as

CENTERS : MUMBAI /DELHI /AKOLA /LUCKNOW /NASHIK /PUNE /NAGPUR /BOKARO /DUBAI # 40
ELECTROCHEMISTRY Rg. 2018 - 2020

 Zn 2 
0.059
E  E0Cu 2 Cu  E 0Zn 2 Zn   log  
2 Cu 2 
 
(ii) Fe | Fe2+ | |H2SO4 | H2 Pt
Oxidation half cell: Fe   Fe2+ + 2e–
2H   2e    H2
Reduction half cell: 
Fe  2 H   Fe2  H 2
Thus the emf equation will be
 Fe2   PH 
0.059  2
 0 0
E  E H H  E Fe2 Fe   log  2
2
2  H 
 
(iii) Pt , H2 | HCl | Cl2, Pt
1
Oxidation half: H2   H+ + e–
2
1
Reduction half: Cl2 + e-   Cl–
2
1 1
Cell reaction: H2 + Cl2   H+ + Cl–
2 2
 H    Cl  
0.059
 0 0
 E  E Cl Cl  E H  H   log  1 2  
12
2 2
2  PH   PCl 
 2  2
Note : that the number of electrons involved in the half cell reaction is 1.

26. The following electrochemical cell has been set up.


Pt(1) | Fe3+ , Fe2+ (a = 1) | Ce4+ ,Ce3+ (a = 1) | Pt (2)
E0 (Fe3+ / Fe2+) = 0.77 V; E0 (Ce4+ / Ce3+ ) = 1.61 V.
If an ammeter is connected between the two platinum electrodes, predict the direction of flow of
current. Will the current increase or decrease with time ?
0 0
Given ECe 4
/ Ce3
 1.61V ; E Fe 3
/ Fe2
 0.77 V
Sol. Thus for Ecell to be positive, i.e. following reaction should occur
Ce4+ + Fe2+   Ce3+ + Fe3+
4+ 3+
Hence Ce / Ce electrode will act as cathode
 Current will flow from Ce electrode to iron electrode
Current will decreases with time.

27. Calculate emf of the following cells.


(i) Fe | Fe2+[0.6 M] | | Sn2+[0.2M]| Sn E 0Fe 2 Fe
= – 0.44 V 0
E Sn 2
Sn
= + 0.14 V
(ii) Pt, H2[3 atm]| HCl | H2[15 atm]Pt
Sol. When concentration of either of the half-cell is different from 1 M (standard condition), emf of such
cell is calculated according to this reaction.

E = (E0Right – E0Left ) 
0.059
log
 Products 
n  Reactants 
Now let us calculate emf of the given cells according to the following relation.

CENTERS : MUMBAI /DELHI /AKOLA /LUCKNOW /NASHIK /PUNE /NAGPUR /BOKARO /DUBAI # 41
ELECTROCHEMISTRY Rg. 2018 - 2020

(i) Oxidation half : Fe   Fe2+ + 2e–


Reduction: Sn2+ +2e–  Sn
cell reaction : Fe + Sn2+  Fe2+ + Sn
0
Here, E0right , E0cathode or E Sn Sn 2
= + 0.14 Volt
E0left , E0anode or E 0Fe 2 Fe
= – 0.44 Volt
[Fe2+] = 0.6, [Sn2+] = 0.2, n = 2
Thus substituting the various values in the above relation.
0.059 0.6
E = (+ 0.14) – (–0.44) – log = 0.58 – 0.0295 log 3
2 0.2
= 0.58 – 0.0295  0.4771 = 0.566 Volt

(ii) Oxidation half:  2H+ +2e–


H2 
Reduction half: 2H+ + 2e– 
 H2
Cell reaction : H2 
 H2
* Cell in which two electrodes are identical and dipped in the same solution is known as concentration
cell. For such cell, E02 – E01 = 0.
Substituting the various values as in (i), in the relation.
0.059  p H ,right 
E  E 0Right  E Left
0
 log  2   0  0.059 log 15
n  p H ,Left  2 3
 2 
= 0 – 0.0295 log 5 = – 0.0206 volt

28. The standard reduction potential for the half cell


NO3 (aq) + 2H+ (aq) + e–  NO2(g) + H2O is 0.78 V.
(a) Calculate the reduction potential in 8 M H+
(b) What will be the reduction potential of the half cell in a neutral solution? Assume all other
species to be at unit concentration ?
Sol. (a) In 8 M H+ solution, conc. of all other species is unity
0.059
 ERedu.Pot. = E0R P + log [H+]2
n
0.059
= 0.78 + log (8)2 = 0.8866 V
1
(b) In case of neutral solution, [H+] = 10 –7 M and concentration of other species is unity.
0.059 0.059
E R P = E0R P + log [H+]2 ERP = 0.78 + log (10–7)2 = – 0.046 V
n 1

29. The standard reduction potential of the Ag+ / Ag electrode at 298 K is 0.799 V. Given that for Agl,
Ksp = 8.7  10–17 evaluate the potential of the Ag+/Ag electrode in a saturated solution of Agl. Also
calculate standard reduction potential of the I– | AgI | Ag electrode.
0
Sol. Ag+ + e–   Ag EAg 
/ Ag
 0.799 V
0.0590 1
According to Nernst equation E = E Aglog
 
n [ Ag  ]
/ Ag

In a saturated solution of AgI AgI(s) Ag+ (aq) + I– (aq)


Ksp(Agl) = 8.7  10–17 = [Ag+][I–]
Since [Ag+] = [I–]

CENTERS : MUMBAI /DELHI /AKOLA /LUCKNOW /NASHIK /PUNE /NAGPUR /BOKARO /DUBAI # 42
ELECTROCHEMISTRY Rg. 2018 - 2020

 [Ag+] = 8.7  1017 = 9.3  10–9 M


Substituting the values in the Nernst equation
0.059 1
E = 0.799 – log  0.325 V
1 9.3  10 9
Calculation of standard reduction potential of I–| AgI | Ag
Considering I– / AgI / Ag half-cell
Cell reaction : AgI(s) + e– = Ag(s) + I– (aq)
RT
Cell potential : E I  / AgI / Ag  E I0 / AgI / Ag  ln [ I  ]
F
We will have : E Ag  Ag  E I AgI Ag
0 RT 1 RT
Equating equations (i) and (ii), we get E Ag 
/ Ag
 ln 
 E I0 / AgI / Ag  ln [ I  ]
F [ Ag ] F
RT RT
or EI0 / AgI / Ag  E Ag
0

/ Ag
ln [ Ag  ] [ I  ]
  E 0Ag  Ag  ln K sp  AgI 
F F
= 0.799V + (0.0591 V) log 8.7  10–17
= 0.799V – (0.0591  16.0605)V = 0.799V – 0.950 V = – 0.151 V

30. Zinc granules are added in excess to a 500 ml of 1.0 M nickel nitrate solution at 25 0 C until the
equilibrium is reached. If the standard reduction potential of Zn2+/Zn and Ni2+/Ni are – 0.75 V and
– 0.24 V respectively, find out the concentration of Ni2+ in solution at equilibrium.
Sol. The following chemical cell sets up : Zn | Zn2+ | | Ni2+ | Ni
The net cell reaction is: Zn + Ni2+Zn2+ + Ni
The e.m.f. is given by
 Zn 2 
0.059
0 0
E Ni2 Ni  E Zn 2 Zn  log  
2 2
 Ni 
 
 Zn 2   Zn 2 
E cell  0.24   0.75  0.0295log    0.51  0.0295log  
2
 Ni  2
 Ni 
   
At equilibrium Ecell = 0
Let x mol l–1 be the concentration of Ni2+ at equilibrium.
Then [Zn2+] = 1 – x [ 1 mole of Ni2+ gives 1 mole of Zn2+]
1 x 1 x 0.51 1 x
 0.0295 log = 0.51 orlog = = 17.29 or = 1.95  1017
x x 0.0295 x
1
or x= = 5.128  10 –18 mol l –1
1.95  1017

31. A cell, Ag | Ag+ || Cu 2+ | Cu , Initially contains 1 Litre of 1 M Ag+ and 1 M Cu2+ ions.
Calculate the change in the cell potential after the passage of 9.65 A of current for 1 h.
Ag | Ag+ || Cu2+ | Cu
1.0 M 1.0 M
Sol. The cell reaction 2Ag + Cu2+  2Ag+ + Cu
0.059
E Initial = E0 Initial  log Q Since [Ag+ ] = [Cu2+ ] = 1.0 M
n

CENTERS : MUMBAI /DELHI /AKOLA /LUCKNOW /NASHIK /PUNE /NAGPUR /BOKARO /DUBAI # 43
ELECTROCHEMISTRY Rg. 2018 - 2020

[ Ag  ]2
 Q= 1
[Cu 2 ]
EInitial = E0 ; On passing a current of 9.65 A for 1 hour
No. of equivalents of Ag+ lost = no. of equivalent of Cu2+ formed
I t
No. of equivalent of Ag+ lost   0.36 ;No. of moles of Cu2+ formed = 0.36/2 = 0.18
96500
No. of moles of Ag+ left = 1- 0.36 = 0.64 ;No. of moles of Cu 2+ left = 1+ 0.18 = 1.18
Substituting the value in Nernst equation
0.0592 (0.64) 2 0.059
Efinal = E0  log = Eo  log 0.3471 = E0 + 0.01355
2 1.18 2
Change in the cell potential E change = E0 - (E0 + 0.01355) = -0. 01355 V

32. Calculate the equilibrium constant for the reaction , Fe2+ + Ce 4+  Fe3+ + Ce3+
Sol. The given electrochemical reaction is :
Fe2+ + Ce4+ Fe3+ + Ce3+ 0
ECe 4
/ Ce3
 1.44 V E 0 Fe3 / Fe2  0.68 V
Here cell potential can be calculated as below
= 1.44 - 0.68 = 0.76 V
applying the nernst equation
However , at equilibrium , E = 0 and
 Products   k
 Reactant  eq
0.059
 E 0 Cell  log K eq
n
0.059
Substituting the various values 0.76  log K eq
1
On usual calculationsKeq = 7.6  10 12

33. Find the solubility product of a saturated solution of Ag2 CrO4 in water at 298 K if the emf of
the cell Ag | Ag+ , Saturated Ag2 CrO4 soln. || Ag+ (0.1 M) | Ag is 0.164 V at 298 K.
The cell reaction can be written as Ag | Ag+ , Ag2CrO4 || Ag+ ( 0.1 M) | Ag : E = 0.164 V
At cathode Ag+Cathode +e Ag ; At anode Ag  Ag+Anode + e
Net Reaction Ag+cathode  Ag+Anode ; E = 0.164 V
+
Thus here , n = 1 , E = 0 .164 V , [Ag ] cathode = 0.1 M
Let the solubility of Ag2 CrO4 = S M ; Since Ag2CrO4 gives 2 Ag+
0.059  Ag   anode

 concentration of [Ag+ ]anode = 2 S M 0.164   log
1  Ag   cathode
0.059 2S 0.059 0.1
0.164 =  log or 0.164  log 2S = 1.66 10 - 4
1 0.1 1 2S
Hence S = 0. 83 10 - 4 M
For Ag2 CrO4 ; Ag2 CrO4  2Ag+ + CrO 24 
Ksp = (2S)2 (S) = 4S3  Ksp = 4  (0.83  104)3= 2.29  10 – 12

CENTERS : MUMBAI /DELHI /AKOLA /LUCKNOW /NASHIK /PUNE /NAGPUR /BOKARO /DUBAI # 44
ELECTROCHEMISTRY Rg. 2018 - 2020

FOUNDATION BUILDERS (OBJECTIVE)


Galvanic cells
1. Which of the following statements about a salt bridge in a voltaic cell is TRUE?
(A) Free electrons flow through the salt bridge to maintain electrical neutrality in the two half-cells.
(B)The salt bridge allows the ions present in the two half-cells to mix extensively.
(C) The wire must be connected directly to the salt bridge,in order to maintain electrical neutrality in
the two half-cells.
(D)Ions from the electrolyte in the salt bridge flow into each half-cell to maintain electrical
neutrality.

2. Which of the following statements about porous disks in voltaic cells is TRUE?
(A)Free electrons flow through the porous disk to maintain electrical neutrality in the two half-cells.
(B) Ions present in the two half-cells flow through the porous disk to maintain electrical neutrality in
both half- cells.
(C) A porous disk contains a strong electrolyte like potassium chloride (KCl).
(D) A wire must be connected directly to the porous disk in order for the porous disk to be able to
maintain electrical neutrality in the two half-cells.

3. All galvanic cells do not contain


(A) a cathode (B) an anode (C) ions (D) a porous plate

4. In galvanic cells
(A) electrical energy is converted into chemical energy
(B) chemical energy is converted into electrical energy
(C) electrical energy is converted into heat
(D) chemical energy is converted into heat

5. If a salt bridge is removed between the two half cells, the voltage
(A) Drops to zero (B) Does not change
(C) Increases gradually (D) Increases rapidly

6. In the galvanic cell Cu Cu 2 1M  Ag  1M  Ag , the electrons will travel in the external circuit
(A) from Ag to Cu (B) from Cu to Ag
(C) electrons do not travel in the external circuit (D) can not be predicted

Reference Electrodes and Standard Electrode Potential

7. Which of the following half-reaction is involved in the STANDARD HYDROGEN ELECTRODE?


(A)Pt2+(aq) + 2e− → Pt(s) (B) 2H3O+(aq) + 2e− → 2H2O(l) + H2(g)
(C) H2O2(aq) + 2H3O (aq) + 2e → 4H2O(l) (D) O2(g) + 2H2O(l) + 4e− → 4OH−(aq)
+ −

8. Normal hydrogen electrode has been assigned a potential of


(A) One volt (B) Zero volt (C) Hundred volts (D) None of the above

9. The reference electrode is made from which of the following ?


(A)ZnCl2 (B) CuSO4 (C) Hg2Cl2 (D) HgCl2

CENTERS : MUMBAI /DELHI /AKOLA /LUCKNOW /NASHIK /PUNE /NAGPUR /BOKARO /DUBAI # 45
ELECTROCHEMISTRY Rg. 2018 - 2020

10. The equation representing the process by which standard reduction potential of zinc can be
defined as:
(A) Zn 2  ( s )  2 e   Zn (B) Zn ( g )  Zn 2  ( g )  2 e 
(C) Zn 2  ( g )  2e   Zn (D) Zn 2  ( aq )  2 e   Zn ( s )

Electrochemical Series
11. Which is the best reducing agent ?
(A) F- (B) Cl- (C) Br - (D) I –

12. Four colorless salt solutions are placed in separate test tubes and a strip of copper is dipped in
each.Which solution finally turns blue?
(A) Pb (NO3)2 (B) AgNO3 (C) Zn(NO3)2 (D) Cd(NO3)2
13. From the electrochemical series, it can be concluded that :
(A) Zn2+ will liberate H2 from 1 M HCl (B) Ag metal reacts spontaneously with Zn2+
(C) Zn metal will liberate H2 from 1 M HCl (D) Ag metal will liberate H2 from 1 M HCl

14. Which one is the wrong statement about electrochemical series?


(A) Active metals have negative reduction potentials
(B) Active non - metals have positive reduction potentials
(C) Metals above hydrogen liberate hydrogen from acids
(D) Metals below hydrogen are strong reducing agents.

15. The standard electrode potentials of four elements A , B , C and D are -3.05 , 1.66, -0.40 & 0.80
volt. The highest chemical activity will be shown by :
(A) A (B) B (C) C (D) D

16. Consider the following standard reduction potentials:


Ca2+ + 2e− Ca; E o = −2.76 V
Pb2 + 2e− Pb E o = −0.13 V
2+ −
Cu + 2e  Cu; E o = 0.34 V
Hg 22  + 2e− Hg 2 ; E o = 0.80 V
Pt2+ + 2e− Pt E o = 1.20 V
Which of the following metals is the strongest REDUCING AGENT
(A)Ca (B)Pb (C) Cu (D) Hg

17 Stronger the oxidising agent, greater is the


(A) Reduction potential (B) Oxidation potential
(C) Ionic behavior (D) None of the above

18. If a strip of copper metal is placed in a solution of ferrous sulphate


(A) Copper will precipitate out (B) Iron will precipitate out
(C) Copper and iron both will be dissolved (D) No reaction will take place

19. Which of the following metals can deposit copper from copper sulphate solution?
(A) Mercury (B) Iron (C) Gold (D) Platinum

20. On the basis of position in the electrochemical series, the metal which does not displace hydrogen
from water and acids is:
(A) Hg (B) Al (C) Pb (D) Ba
CENTERS : MUMBAI /DELHI /AKOLA /LUCKNOW /NASHIK /PUNE /NAGPUR /BOKARO /DUBAI # 46
ELECTROCHEMISTRY Rg. 2018 - 2020

21. When a piece of copper wire is immersed in a solution of silver nitrate, the colour of the solution
becomes blue. Thisis due to the
(A) Oxidation of Cu (B) Reduction of Cu
(C) Formation of a soluble complex (D) Oxidation of Ag

22. The position of some metals in the electrochemical series in decreasing electropositive character is
given Mg > Al > Zn > Cu > Ag. What will happen if a copper spoon is used to stir a solution of
aluminium nitrate?
(A) The spoon will get coated with aluminium (B) An alloy of copper and aluminium is formed
(C) The solution becomes blue (D) There is no reaction

23. When a rod of metal A is dipped in an aqueous solution of metal B (concentration of B2+ ion being 1
M) at 25 0C, the standard electrode potentials are A2+ /A = – 0.76 Volts, B2+ / B = + 0.34 volts
(A) A will not dissolve (B) B will deposit on A
(C) No reaction will occur (D) Water will decompose into H2 and O2

24. The reaction Zn2+ + 2e– Zn has a standard potential of – 0.76 V. This means
(A) Zn can’t replace hydrogen from acids (B) Zn is a reducing agent
(C) Zn is an oxidizing agent (D) Zn2+ is a reducing agent

25. If the half cell reaction A + e– A– has a large negative reduction potential, it follows that
(A) A is readily reduced (B) A is readily oxidized
(C) A– is readily reduced (D) A– is readily oxidized

26. CuSO4 is not stored in aluminium bottles because


(A) Cu gets oxidized (B) Cu gets reduced
(C) Al gets oxidized (D) CuSO4 gets decomposed

Cell Potential
27. The standard electrode potentials for the reactions,
Ag  ( aq )  e   Ag ( s )
Sn 2  ( aq )  2 e   Sn ( s )
at 250 C are 0.80 volt and -0.14 volt respectively. The emf of cell
Sn | Sn2+ ( 1M) || Ag+ ( 1M ) | Ag is :
(A) 0.66 volt (B) 0.80 volt (C) 1.08 volt (D) 0.94 volt

28. Which one of the following is not the correct representation?


0
(A) Ecell = reduction potential of cathode + oxidation potential of anode.
0
(B) Ecell = reduction potential of cathode - oxidation potential of anode.
0
(C) Ecell = reduction potential of cathode - reduction potential of anode.
0
(D) Ecell = -oxidation potential of cathode + oxidation potential of anode.

29. Two half - cells have standard electrode potentials - 0.44 and 0.799 volt respectively. These two are
coupled to make a galvanic cell. Which of the following will be true?
(A) Electrode of half - cell potential - 0.44 V will act as anode
(B) Electrode of half - cell potential - 0.44 V will act as cathode
(C) Electrode of half - cell potential 0.799 V will act as anode
(D) Electrode of half - cell potential - 0.44 V will act as a positive terminal

CENTERS : MUMBAI /DELHI /AKOLA /LUCKNOW /NASHIK /PUNE /NAGPUR /BOKARO /DUBAI # 47
ELECTROCHEMISTRY Rg. 2018 - 2020

30. What is the cell potential (standard EMF ‘Eo) for the reaction below? [Eo(Fe2+(aq)/Fe) = −0.44V and
Eo(O2(g)/H2O/OH-) = +0.4V]2Fe(s) + O2(g) + 2H2O(l)  2Fe2+(aq) + 4OH− (aq)
(A) E 0Cell  -0.48 V (B) E 0Cell  −0.04 V (C) E 0Cell  +0.84 V (D) E 0Cell  +1.28 V

31. The standard reduction potentials, E  , for the half- reactions are as
Zn  Zn 2   2e; E 0  0.76 V
Fe  Fe 2   2e; E 0  0.41V
the EMF for the cell reaction,
Fe 2   Zn  Zn 2   Fe is
(A) 0.35 V (B) 0.35 V (C) 1.17 V (D) 1.17 V

32. From the following E 0 values of half cells,


(i) A  e  A  ; E 0  0.24 V (ii) B   e  B 2 ; E 0  1.25 V
(iii) C   2e  C 3 ; E 0  1.25 V (iv) D  2e  D 2 ; E 0  0.68 V
When combination of two half cells would result a cell with the largest potential?
(A) (ii) and (iii) (B) (ii) and (iv) (C) (i) and (iii) (D) (i) and (iv)

Thermodynamics of Cell

33. The equilibrium constant for the reaction,Sn(s) + 2H+ → Sn2+ + H2(g) at 25 oC is:
Eo (Sn2+ | Sn) = -0.14
(A)5.5  104 (B) 2.3  102 (C) 1.1  102 (D) 1.00
Direction for Q. Nos. 34 to 38: Use the following information to answer the these questions.

A galvanic cell is constructed from a half-cell containing a solid chromium electrode in 1.0 M
Cr(NO3)3 solution and a half-cell containing a solid tin electrode in 1.0 M Sn(NO3)2solution. The
half-cells are linked by an external circuit and by a KCl salt bridge.
Sn2+ + 2e− → Sn; E o = −0.14
and Cr3+ + 3e− → Cr; E o = −0.74
34. The balanced overall (net) cell reaction is
(A) Sn(s) + Cr3+(aq)  Sn2+(aq) + Cr(s)
(B) 3Sn(s) + 2Cr3+(aq)  3Sn2+(aq) + 2Cr(s)
(C) 3Sn2+(aq) + 2Cr(s)  3Sn(s) + 2Cr3+(aq)
(D) Sn2+(aq) + Cr(s)  Sn(s) + Cr3+(aq)

35. Which of the following statements is TRUE about this cell?


(A) The chromium electrode is the anode,and the tin electrode is the cathode.
(B) The cell electrons flow from the tin electrode to the chromium electrode.
(C) The cell diagram is Sn (s) | Sn2+, 1.0 M | salt bridge | Cr (s) | Cr3+, 1.0 M |.
(D) G > 0

36. This cell generates a potential of …..volts.


(A)0.30 V (B)−0.30 V (C) 0.60 V (D) −0.60

37. The value of the equilibrium constant at 25oC for this reaction is closest to
(A) 1.0  10−66 (B) 6.7  10−43 (C) 7.4  101 (D) 7.7  10 61

CENTERS : MUMBAI /DELHI /AKOLA /LUCKNOW /NASHIK /PUNE /NAGPUR /BOKARO /DUBAI # 48
ELECTROCHEMISTRY Rg. 2018 - 2020

38. The value of Go for this reaction in kJ is?


(A) −320 (B) −247 (C) 244 (D) −347

39. The value of equilibrium constant for a feasible cell reaction is


(A) <1 (B) 0 (C) =1 (D) >1

40. If the following half cells have the E 0 values as


Fe3  e  Fe 2 ; E 0  0.77 V and Fe 2  2e  Fe; E 0  0.44 V ; the E 0 of the half cell
Fe 3   3e  Fe will be
(A) 0.33 V (B) 1.21 V (C) -0.036 V (D) 0.605V

Nernst Equation

41. For the half - cell reaction: Au 3++ 3 e -  Au


The value of n used in Nernst equation is :
(A) 3 (B) 2 (C) 1 (D) 3  96500

42. The standard emf for the cell reaction : Zn  Cu 2   Zn 2   Cu is 1.10 volt at 25o C. The emf for the
cell reaction when 0.1 M Cu 2+ and 0.1 M Zn 2+ solutions are used at 25o C is :
(A) 1.10 volt (B) 0.110 volt (C) - 1.10 volt (D) 0.110 volt

43. The cell potential for the electrochemical reaction shown below depends upon the Cl− and Cu2+
concentrations. Calculate the cell potential (in V) at 25oC if [Cu2+] = 3.5 M and [Cl−] = 1.7 M.
Cu2+(aq) + 2Cl−(aq) + 2Ag(s)  Cu(s) + 2AgCl(s); E o = 0.12 V
(A) 0.15 V (B) −0.15 (C) 0.30 V (D) −0.30 V

44. What is the cell potential at 25oC for the reaction?3Ag(s) + NO 3 + 4H+ → 3Ag+ + NO(g) + 2H2O
When [NO 3 ]= 2.0 M. [Ag+] = 0.010 M, the pH is 1.00, and the pressure of NO is 0.20 atm?
Eo (Ag+ | Ag) = 0.799 V Eo(NO3- | NO) = 0.96 V
(A) 0.10 V (B) 0.16 V (C) 0.22 V (D) 0.30 V

45. What is E  for the following half reaction?


MX 2  s   2e  M  s   2X  ; E   ?
M 2  2e   M  s  ; E   0.100 V
MX 2  s   M 2  2X  ; K sp  e 10
0.0592 1 0.0592 1
(A) E  0.100  ln (B) E o  0.100  log
2 K sp 2 K sp
0.0592 0.0592 1
(C) E   0.100  log K sp (D) E  0.100  log
2 2 Ksp
46. The potential of the cell below was 1.05V . What is the pH ? E 0 Ag 
|Ag
 0.799V
+ +
Pt | H2 (1 atm) | H (aq) || Ag (0.1 M) | Ag
(A) 10.50 (B) 5.25 (C) 1.00 (D) 7.00

CENTERS : MUMBAI /DELHI /AKOLA /LUCKNOW /NASHIK /PUNE /NAGPUR /BOKARO /DUBAI # 49
ELECTROCHEMISTRY Rg. 2018 - 2020

47. Ag   aq   e 
 Ag  s  ; E   0.800 V
AgBr  s   e 
 Ag  s   Br   aq  ; E   0.071V
Br2 (l) + 2e-  2 Br- ; E   1.066 V
Use some of the data above to calculate Ksp at 25  C for AgBr .
(A) 6.3  10 2 (B) 4.4 x 10-13 (C) 1.9  10 15 (D) 2.4  10  34

48. Doubling all the coefficients in the equation for the cell reaction
(A) doubles both E  and  G  (B) doubles E  , but does not change  G 
(C) doubles  G  , but does not change E  (D) does not change E  or  G 

49. Calculate the cell potentials E at 25 C for the reaction


2Al(s) + 3 Fe2+ (aq)  2Al3+ (aq) + 3Fe (s)given that  Fe2   0.020 M,  Al3   0.10 M , and the
   
standard reduction potential is 1.66 V for Al3 Al and 0.45 V for Fe 2  Fe .
(A) 1.03V (B) 1.45 V (C) 1.18V (D) 1.20 V

50. How much will the potential of Zn Zn 2  change if the solution of Zn 2 is diluted 10 times?
(A)increase by 0.03V (B) decrease by 0.03V
(C) increase by 0. 059 (D)decrease by 0.059 V

Concentration Cells

51. Which of the following facts about the galvanic cell and concentration cell is correct?
(A) Galvanic cell is non-spontaneous whereas concentration cell is spontaneous.
(B) Galvanic cell has an overall cell reaction whereas concentration cell has no overall reaction.
(C) Two half cells of both the galvanic and concentration cells are chemically different.
(D) E cell equations (Nernst equation) of both the cells have the term E 0cell

52. A concentration cell is made up of the Fe Fe 2 couple, where the Fe 2  concentrations are 1.0M and
0.10 M . Which of the following statements best describes the operation of this cell?
(A) The anode compartment contains 0.10 M Fe 2 solution, which decrease in concentration as the
cell operates, and the cathode compartment contains a 1.0 M Fe 2  solution which also decreases
in concentration as the cell operates
(B) The anode compartment contains a 0.10 M Fe 2 solution, which increases in concentration as the
cell operates, and the cathode compartment contains a 1.0 M Fe 2  solution which decreases in
concentration as the cell operates.
(C) The anode compartment contains a 0.10 M Fe 2 solution, which decreases in concentration as the
cell operates, and the cathode compartment contains a 1.0 M Fe 2  solution which increases in
concentration as the cell operates.
(D) The anode compartment contains a 1.0 M Fe 2  solution, which decreases in concentration as the
cell operates and the cathode compartment contains a 0.10 M Fe 2 solution which increases in
concentration as the cell operates.

CENTERS : MUMBAI /DELHI /AKOLA /LUCKNOW /NASHIK /PUNE /NAGPUR /BOKARO /DUBAI # 50
ELECTROCHEMISTRY Rg. 2018 - 2020

53. What would you observe if you set up the following electrochemical cell:
Ag|AgNO3 (0.001 M) || AgNO3 ( 1 M ) | Ag
(A) Electrons will flow from left to right, causing a decrease in the [Ag+ ]concentration in the right
cell.
(B) Electrons will flow from right to left, causing an increase in the [Ag+ ] concentration in the left
Cell, and a decrease in the [Ag+ ]concentration in the right cell.
(C) Electrons will flow from left to right, causing an increase in the[Ag+ ] concentration in the left
cell,and a decrease in the [Ag+ ] concentration in the right cell.
(D) Electrons will flow from right to left, causing a decrease in the [Ag+ ]concentration in the right
cell.

54. The cell shown below generates a potential of 0.643V at 300 K.K sp for AgBr is?
Ag|AgBr (s) | NaBr ( 0.1 M) || AgNO3 (0.1 M) | Ag

(A) 1.15  10  9 (B) 2.00  10 30 (C)1.26 x 10 – 13 (D) 3.5  10 11

55. Calculate the cell potential (in V) at 25 C for a copper concentration cell in which the Cu 2 
concentration in one half-cell is equal to 0.050 M and the Cu 2  concentration in the other half-cell
is equal to 1.5M
Note: The number of moles of electrons transferred is equal to 2.
(A) 0.40 V (B) 0.80 V (C) 0.044 V (D) 0.34 V

Application of cell potential

56. The temperature coefficient of a galvanic cell is  5.0  10  5 VK  1 . During the discharge of the cell,
the cell temperature
(A) increase (B) decrease
(C) does not change (D) first increases and the decreases

57. The cell Pt H2 1atm  H ,pH  X Normal calomel electrode has EMF of 0.4747 Vat 25  C . The
standard oxidation potential of calomel electrode is 0.28 V , then pHof solution will be
(A) 6.6 (B) 3.3 (C) 13.2 (D) 1.1

58. Calculate K sp for PbI 2  s  at 25 C


PbI2  s   2e 
 Pb  s   2I   aq  ; E   0.365 V
Pb 2   aq   2e  
 Pb  s  ; E   0.126 V
(A) 4.5  10 13 (B) 9.1  10  5 (C) 2.5  10 17 (D) 8 x 10 -9

59. Given the reaction, 2 Hg  l   Cl2  g  


 Hg 2Cl2  s  . Assume that the standard Ecell voltage is
related to the Celsius temperature by the following equation and all substances are at standard state.
In doing this, you can assume any temperature (s) you desire.
 
At 298K 25 C , E cell  1.058 volts


Ecell  1.058  0.00017 t  25 C 
The  H   in kJ  for the reaction (as written) at 298 K is
(A) -321 (B) +107 (C) +214 (D) -214
CENTERS : MUMBAI /DELHI /AKOLA /LUCKNOW /NASHIK /PUNE /NAGPUR /BOKARO /DUBAI # 51
ELECTROCHEMISTRY Rg. 2018 - 2020

60. Which of the following graph correctly illustrates the dependence of the cell voltage, E cell , on the
pH for the reaction,
2Ag   aq   H 2  g  
 2Ag  s   2H   aq 
Assume the silver ion remains constant at,  Ag    1.0 M and the H 2 pressure remains constant at 1
 
atm.
(A) (B)

E cell

pH
(C) (D)

Electrolysis
61. In the electrolysis of NaCl
(A)Cl– ion is oxidised at anode (B)Cl– ion is reduced at anode
(C)Cl– ion is oxidised at cathode (D)Cl– ion neither reduced nor oxidised

62. NaOH is manufactured by the electrolysis of brine solution. The products of the reaction are
(A)Cl2 and H2 (B)Cl2 and Na-Hg (C)Cl2 and Na (d)Cl2 and O2

63. A solution of sodium sulphate in water is electrolysed using inert electrodes. The products at the
cathode and anode are respectively
(a)H2, O2 (b)O2, H2 (c)O2, Na (D)O2, SO2

64. The passage of current liberates H2 at cathode and Cl2 at anode. The solution is
(A)Copper chloride in water (B)NaCl in water
(C) H2SO4 (D)Water

65. Which of the following reactions occurs at the cathode ?


(A) 2OH–  H2O + O + 2e– (B) Ag  Ag+ + e–
(C) Fe2+  Fe3+ + e– (D) Cu 2+ + 2e–  Cu

66. Which of the following moves towards the anode during electrolysis of fused NaOH?
(A) Na+ (B) H+ (C) OH– (D) O2–

67. On electrolyzing a solution of dilute H2SO4 between platinum electrodes, the gas evolved at the anode
is
(A) SO2 (B) SO3 (C) O2 (D) H2

CENTERS : MUMBAI /DELHI /AKOLA /LUCKNOW /NASHIK /PUNE /NAGPUR /BOKARO /DUBAI # 52
ELECTROCHEMISTRY Rg. 2018 - 2020

68. Copper sulphate solution is electrolyzed using copper electrodes, the reaction taking place at anode is
(A) H+ + e– H (B) SO42– (aq)  SO4 + 2e–
(C) Cu 2+ + 2e– Cu (D) Cu (s) Cu2+ (aq) + 2e–

69. In electrolysis of NaCl, when Pt electrode is taken, then H2 is liberated at cathode while with Hg
cathode it forms sodium amalgam. This is because
(A)Hg is more reactive than Pt
(B)more voltage is required to reduce H+ at Hg than Pt
(C)Na is dissolved in Hg while it does not dissolve in Pt
(D)conc. of H+ ions is larger when Pt electrode is taken

70. Electrolytic cell is used to convert


(A)chemical energy to electrical energy (B)electrical energy to chemical energy
(C)chemical energy to mechanical energy (D)electrical energy to mechanical energy

Faraday’s Laws

71 Charge required to liberate 11.5 g sodium is


(A) 0.5 F (B) 0.1 F (C) 1.5 F (D)96500 coulombs

72. Law of electrolysis was given by


(A)Lamark (B)Ostwald (C)Faraday (D)Arrhenius

73. The amount of ion discharged during electrolysis is not directly proportional to
(A)Resistance (B)Time (C) Current (D)Chemical equivalent of the ion

74 When the same quantity of electricity is passed through the solution of different electrolytes in series,
the amounts of product obtained are proportional to their
(A)Atomic weights (B)Chemical equivalents
(C)Gm. molecular volume (D)Gm. atomic ions

75. The unit of electrochemical equivalent is


(A)gram (B)gram/ampere (C)grams/coulomb (D)coulomb/gram

76. The quantity of electricity needed to liberate one gram equivalent of an element is
(A) 1 ampere (B)96500 amperes (C)96500 coulombs (D)96500 Faradays
77. The quantity of electricity required to liberate one gm equivalent weight of an element is called
(A)Ampere (B) Volt (C)Ohm (D)Faraday

78. One Faraday of electricity will liberate one gram atom of the metal from a solution of
(A)AuCl3 (B)BaCl2 (C)CuSO4 (D)NaCl

79. A certain current liberated 0.504 gm of hydrogen in 2 hrs. How many grams of copper can be
liberated by the same current flowing for the same time in a copper sulphate solution ?
(A)12.7 gm (B)15.9 gm (C)31.8 gm (D)63.5 gm

80. An electrolytic cell contains a solution of Ag2SO4 and have platinum electrodes. A current is passed
until 1.6 gm of O2 has been liberated at anode. The amount of silver deposited at cathode would be
(A)107.88 gm (B)1.6 gm (C)0.8 gm (D)21.60 gm

CENTERS : MUMBAI /DELHI /AKOLA /LUCKNOW /NASHIK /PUNE /NAGPUR /BOKARO /DUBAI # 53
ELECTROCHEMISTRY Rg. 2018 - 2020

81. If a direct current deposits 19.5 grams of potassium (At. wt. 39) in one minute, the number of grams
of aluminium (At. wt. 27) deposited by the same current during the same time interval would be
(A)27.0 (B)13.5 (C)9.0 (D)4.5

82. When an electric current is passed through acidulated water, 112 ml of hydrogen gas at NTP is
collected at the cathode in 965 seconds. The current passed in amperes, is
(A) 1.0 (B) 0.5 (C) 0.1 (D) 2.0

83. An electric current is passed through an aqueous solution of the following, which one will
decompose ?
(A)Urea (B)Glucose (C)Silver nitrate (D)Ethyl alcohol

84. How many coulombs of electricity are consumed when 100 mA current is passed through a solution
of AgNO3 for half an hour during an electrolysis experiment?
(A) 108 (B) 180 (C) 1800 (D) 18000

85. On passing I ampere of electricity through an electrolyte solution for t second, m gm metal deposits
on cathode. The equivalent weight E of the metal is
I t Im 96500  m I  t  96500
(A) E = (B) E = (C) E = (D) E =
m  96500 t  96500 tI m

86. Which one is not correct ? Faraday’s laws hold


(A) At all pressures (B) Only at 298K (C) In all concentrations (D) In different solvent

87. Faraday’s laws of electrolysis are related to the


(a) Atomic number of the cation (b) Atomic number of the anion
(c) Equivalent weight of the electrolyte (d) Speed of the cation

88. Units of Faraday are


(A)Amperes (B) C (C) C MOL–1 (D) C SEC–1

89. The number of electrons involved in the reaction when a Faraday of electricity is passed through an
electrolyte in solution is
(A) 12  10 46 (B) 96540 (C) 8  10 16 (D) 6  1023

90. The amount of electricity that can deposit 108 gm of silver from AgNO3 solution is
(A)1 Ampere (B) 1 Coulomb (C)1 Faraday (D)None of the above

91. When during electrolysis of a solution of AgNO3, 9650 coulombs of charges pass through the
electroplating bath, the mass of silver deposited on the cathode will be
(A) 1.08 g (B) 10.8 g (C) 21.6 g (D) 108 g

92. Three Faradays electricity was passed through an aqueous solution of iron (II) bromide. The weight
of iron metal (At. wt. = 56) deposited at the cathode (in gm.) is
(A) 56 (B) 84 (C) 112 (D) 168

93. A current of strength 2.5 amp was passed through CuSO4 solution for 6 minutes 26 seconds. The
amount of copper deposited is(At. wt. of Cu = 63.5) ; Faraday = 96500 coulombs
(A) 0.3175 g (B) 3.175 g (C) 0.635 g (D) 6.35 g

CENTERS : MUMBAI /DELHI /AKOLA /LUCKNOW /NASHIK /PUNE /NAGPUR /BOKARO /DUBAI # 54
ELECTROCHEMISTRY Rg. 2018 - 2020

94. A certain quantity of electricity is passed through an aqueous solution of AgNO 3 and cupric salt
solution connected in series. The amount of Ag deposited is 1.08 gm, the amount of copper deposited
is (At. wt. Cu = 63.5; Ag = 108).
(A) 0.6454 g (B) 6.354 g (C) 0.3177 g (D) 3.177 g

95. The number of electrons required to deposit 1 gm atom of aluminium (At wt. = 27) from a solution
of aluminium chloride will be
(A) 1 N (B) 2 N (C) 3 N (D) 4 N

96. How many minutes will it take to plate out 2.19 g of chromium metal from a solution of Cr 3 using a
current of 35.2 A?
(A) 5.77 (B) 346 (C) 115 (D) 1.92

97. The number of Faradays needed to reduce 4 gm equivalents of Cu 2+ to Cu metal will be


(A) 1 (B) 2 (C) 1/2 (D) 4

98. The quantity of electricity required to reduce 12.3 g of nitrobenzene to aniline assuming 50% current
efficiency is
(A) 115800 coulombs (B)57900 coulombs
(C) 231600 coulombs (D)28950 coulombs

99. The amount of charge that must be passed through a solution containing Cu2+ in order to deposit
1 gm atom of copper (63.55) is
(A)1520 coulomb (B)3040 coulomb
(C)96500 coulomb (D)193000 coulomb

100. The cost of electricity required to deposit 1 g of Mg is Rs. 5.00. How much would it cost to deposit
10 g of Al ? (Al = 27, Mg = 24)
(A) Rs. 10.00 (B) Rs. 27.00 (C)Rs. 44.44 (D)Rs. 66.67

101. On passing 0.1 Faraday of electricity through aluminium chloride, the amount of aluminium metal
deposited on cathode is (Al = 27)
(A)0.27 g (B)0.3 g (C)0.9 g (D)2.7 g

102. An ion is reduced to the element when it absorbs 6  10 20 electrons. The number of equivalents of the
ion is
(A) 0.10 (B) 0.01 (C) 0.001 (D) 0.0001
103. Three Faraday of electricity are passed through molten Al2O3, aqueous solution of CuSO4 and molten
NaCl taken in different electrolytic cells. The amount of Al, Cu and Na deposited at the cathodes will
be in ratio of
(A)1 mole : 2 mole : 3 mole (B)3 mole : 2mole : 1 mole
(C)1 mole : 1.5 mole : 3 mole (D)1.5 mole : 2 mole : 3 mole

104. Chromium plating is applied by electrolysis to objects suspended in a dichromate solution, according
to the following (unbalanced) half reaction:
Cr2O72  aq   e  H   aq  
 Cr  s   H 2O  l  How many hours would it take to apply a
chromium plating of thickness 2.0  10 2 mm to a car bumper of surface area 0.25 m 2 in an
electrolysis cell carrying a current of 75.0 A ? [ The density of chromium is 7.19 g cm3 .]
(A) 2.2 h (B) 1.5 h (C) 3.0 h (D) 0.25 h
CENTERS : MUMBAI /DELHI /AKOLA /LUCKNOW /NASHIK /PUNE /NAGPUR /BOKARO /DUBAI # 55
ELECTROCHEMISTRY Rg. 2018 - 2020

Conductance
105. Electrolytes when dissolved in water dissociate into ions because
(A)They are unstable
(B)The water dissolves it
(C)The forces of repulsion increase
(D)The forces of electrostatic attraction are broken down by water.

106. Conductivity of a solution is directly proportional to


(A)Dilution (B)number of ions (C)Current density (D)volume of the solution

107. The best conductor of electricity is a 1 M solution of


(A)Boric acid (B)Acetic acid (C)Sulphuric acid (D)Phosphoric acid

108. Which of the following aqueous solutions will conduct an electric current quite well?
(A)Glycerol (B)HCl (C)Sugar (D)pure water

109. The one that is a good conductor of electricity in the following list of solids is
(A)Sodium chloride (B)Graphite (C)Diamond (D)Sodium bromide

110. Electrolytic conduction differs from metallic conduction in that in the case of electrolytic conduction
(A)the resistance increases with increasing temperature
(B)the resistance decreases with increasing temperature
(C)the flow of current does not generate heat
(D)the resistance is independent of the length of the conductor.

111. Which one of the following statements is not applicable to electrolytic conductors?
(A)New products show up at the electrodes
(B)Ions are responsible for carrying the current.
(C)Show a positive temperature coefficient for conductance.
(D)A single stream of electrons flows from cathode to anode.

112. The molar conductivity of a strong electrolyte


(A)increases on dilution (B)remains constant
(C)decreases on dilution (D)depends on density

113. When a solution of an electrolyte is heated the conductance of the solution


(A)increases because electrolyte conducts better
(B)decreases because of the increased heat
(C)decreases because of the dissociation of the electrolyte is suppressed
(D) increases because the electrolyte is dissociated more

114. The unit of molar conductivity is


(A)–1 cm–2 mol–1 (B)cm2 mol–1 (C)–1 cm2 mol–1 (D) cm2 mol

115. The molar conductivity is maximum for solution of concentration


(A)0.001 M (B) 0.005 M (C) 0.002 M (D) 0.004 M

116. On increasing the dilution, the specific conductance :


(A) Increases (B) decreases (C) Remains constant (D) none of these

CENTERS : MUMBAI /DELHI /AKOLA /LUCKNOW /NASHIK /PUNE /NAGPUR /BOKARO /DUBAI # 56
ELECTROCHEMISTRY Rg. 2018 - 2020

117. The distance between two electrodes of a cell is 2.5 cm and area of each electrode is 5 cm2 . The
cellconstant is :
(A) 2 (B) 12.5 (C) 7.5 (D) 0.5

118. The highest electrical conductivity of the following aqueous solution is of :


(A) 0.1 M acetic acid (B) 0.1 M chloroacetic acid
(C) 0.1 M fluoroacetic acid (D) 0.1 M difluoroacetic acid

119. Which of the following statements concerning conductance and molar conductance is true?
(A) Both conductance and molar conductance increase on increasing concentration.
(B)Both conductance and molar conductance decrease on increasing concentration.
(C)Conductance increases but molar conductance decrease on increasing concentration.
(D)Conductance decreases but molar conductance increases on decreases but molar conductance
increases on increasing concentration.

120. Which of the following statements concerning concentration dependence of molar conductivity for a
strong and a weak electrolyte is true?
(A)Molar conductivity increases linearly on increasing concentration for both types of electrolytes.
(B)Molar conductivity decreases linearly on increasing concentration for both type of electrolytes.
(C)Molar conductivity increases linearly on dilution for strong electrolyte but it is constant for weak
electrolyte.
(D)On dilution, molar conductivity increases linearly for strong electrolyte but for weak electrolyte,
the increase is gradual in higher concentration range but very rapid in lower concentration range.

121. Which of the following is not an essential requirement for Kohlrausch’s law of independent
migration of ions to hold true?
(A)The solution must be infinitely dilute. (B)Electrolyte must be strong.
(C)Electrolyte must be completely ionized. (D)There should be no inter-ionic interaction.

122. In a conductometric analysis, if temperature is changed slightly, which of the following will not
change?
(A)Specific conductance (B)Equivalent conductance
(C)Cell constant (D)Resistance of the solution

123. Given the following molar conductivity at infinite dilution and 25oC,
HCl: m = 426.2 S cm2 mol−1
KCl: m = 271.5 S cm2 mol−1
CH3COOK: m = 114.42 S cm2 mol−1
The molar conductance at infinite dilution and 250C, for acetic acid solution is
(A)583.28 S cm2 mol−1 (B)269.12 S cm2 mol−1
2 −1
(C)289.63 S cm mol (D)172.94 S cm2 mol−1

124. An aqueous solution of a weak acid (HA) has a molar conductance of 19 S cm2 mol−1 and it is only
1% ionized in the given condition. The limiting value of molar conductance of the above acid under
similar experimental condition is
(A)1900 S cm2 mol−1 (B)950 S cm2 mol−1
(C) 475 S cm2 mol−1 (D)Infinite

CENTERS : MUMBAI /DELHI /AKOLA /LUCKNOW /NASHIK /PUNE /NAGPUR /BOKARO /DUBAI # 57
ELECTROCHEMISTRY Rg. 2018 - 2020

FOUNDATION BUILDERS (SUBJECTIVE)

Representation of Cell diagrams, complete and half cell reactions:


1. Write the half-cell reaction for the following cells:
(A) Pt (s) |Ti3+ , Ti4+ || Fe3+, Fe2+ | Pt (s)
(B) Pb(s) | PbSO4(s) | SO 24  || Zn2+ | Zn(s)
(C) Pt(s) | Hg(l), Hg2Cl2 | Cl− || H+ | H2| Pt(s)
(D) Pt(s) | QH2, Q, H+ || Cl− | AgCl(s) | Ag(s)
QH2 = Hydroquinone and Q = Quinone
2. Write the electrochemical notations for the following reactions:
(A) H2(g) + Cu2+ → 2H+ + Cu(s)
(B) Cl2(g) + 2Fe2+ → 2Fe3+ + 2Cl−
(C) Zn(s) + 2AgCl(s) → 2Ag(s) + ZnCl2
1
(D) [Fe(CN)6]3− + H2(g) → [Fe(CN) 6]4− + H+
2
Electrode potential and standard electrode potential:
3. Answer whether the following reactions will occur spontaneously or not,
under standard condition.
(A) will Fe reduces Fe3+ to Fe2+ ? E o (Fe2+/Fe) = 0.44 V and E o (Fe3+| Fe2+) =0.77V
(B) will O2 (g) oxidizes gold to Au(CN) 2 in presence of CN−(aq) and OH− (aq) ?
E o [Au(CN) 2 /Au, CN−] = 0.60 V and E o (O2/OH, H2O) = 0.40 V.
(C) would you use silver spoon to stir a solution of Cu(NO3)2?
E o (Ag+ / Ag) = 0.80 V and E o (Cu2+ | Cu ) = 0.34 V
(D) reaction of Pb with AgNO3. E o (Ag+/Ag) = 0.80 V and E o (Pb2+/Pb) = −0.126V
(E) will Sn reduce Fe3+ to Fe2+ ? E o (Fe3+/Fe2+) = 0.77 V and Eo(Sn2+/Sn) = 0.15 V

4. For the cell reaction 2Ce4+ + Co → 2Ce3+ + Co2+


Eocell is 1.89 V. If E oCo2  | Co is – 0.28 V, what is the value of EoCe4  | Ce3  ?

5. Determine the standard reduction potential for the half reaction :


Cl2 + 2e–→ 2Cl–
Given Pt2+ + 2Cl– → Pt + Cl2 , E° = – 0.15 V
Pt2+ + 2e– → Pt E° = 1.20 V e−

6. Write cell reaction for the following cell and determine standard state
emf of the cell
Sn Ag
E (Ag+/Ag) = 0.80 V E o (Sn2+/Sn) = 0.14 V
o Sn2+ Ag+

7. Given the standard reduction potentials Tl+ + e– → Tl, E° = -0.34 V and Tl3+ + 2e– → Tl+ , E° =
1.25V. Examine the sponaneity of the reaction, 3Tl+ → 2Tl + Tl3+ . Also find E° for this
disproportionation.

 G and Keq:
8. Calculate the equilibrium constant for the reaction Fe + CuSO4  FeSO4 + Cu at 25°C. Given E°
(Fe/Fe2+) = 0.44 V, E°(Cu/Cu2+) = -0.336 V.
CENTERS : MUMBAI /DELHI /AKOLA /LUCKNOW /NASHIK /PUNE /NAGPUR /BOKARO /DUBAI # 58
ELECTROCHEMISTRY Rg. 2018 - 2020

9. For a cell Mg(s) | Mg2+(aq) || Ag+(aq) | Ag. Find the maximum work that can be obtained by
operating the cell. E°(Mg2+/Mg) = -2.37V, E°(Ag+ /Ag) = 0.8 V.

10. The standard reduction potential at 25°C for the reduction of water
2H2 O + 2e–  H2 + 2OH– is -0.8277 volt. Calculate  G° & the equilibrium constant for the
reaction 2H2O  H3O+ + OH– at 25°C.

11. At 25°C the value of K for the equilibrium Fe3+ + Ag  Fe2+ + Ag+ is 0.531 mol/litre. The
standard electrode potential for Ag+ + e  Ag is 0.799V. What is the standard potential for Fe3+ +
e  Fe2+?
12. The normal oxidation potential of Zn referred to SHE is 0.76 V and that of Cu is 0.34V at 25 C .
When excess of Zn is added to CuSO 4 , Zn displaces Cu 2  till equilibrium is reached. What is the
ratio of Zn 2 to Cu 2  ions at equilibrium?

13. Consider the following sequence of reactions occurring in the acid medium. All emf( standard) are
measured at 25 C and expressed in volt unit:
0.564 2.26 0.95 1.51 1.18
MnO 4  MnO 42    MnO 2   Mn 3   Mn 2    Mn
(a) Determine E  for: MnO 4  Mn 2
(b) Which will be the better oxidizing agent in acid medium MnO 4 or MnO 24  when the final
form of manganese is Mn 2 ?

NERNST EQUATION
14. Calculate the EMF of following cells at 25°C.
(i) Fe | FeSO4 (a = 0.1) || CuSO4 (a = 0.01) | Cu E°(Fe2+/Fe) = -0.44 V
(ii) Fe | Fe2+ (a = 0.3) || Sn2+ (a = 0.1) | Sn E°(Cu2+/Cu) = +0.34 V
(iii) Pt, H2(2atm) | HCl | H2 (10 atm), Pt. E°(Sn2+ /Sn) = -0.14 V

15. What is the electrode potential of Mg2+ | Mg electrode at 25°C, in which the concentration of Mg2+
is 0.01 M. E°(Mg2+ | Mg) = -2.36 V.

16. Calculate E° and E for the cell Sn | Sn2+ (1M) || Pb2+ (10-3M)| Pb,
E°(Sn2+ | Sn) = -0.14 V, E°(Pb2+ | Pb) = -0.13 V. What do you infer from cell EMF?

17. At what concentration of Cu2+ in a solution of CuSO4 will the reduction potential be zero at 25°C?
Can the oxidation potential become positive? E° (Cu | Cu2+) = -0.34V.

18. A zinc electrode is placed in a 0.1 M solution at 25°C. Assuming that the salt is 20% dissociated at
this dilution calculate the electrode potential. E°(Zn2+ | Zn) = -0.76 V.

19. Calculate the EMF of a Daniel cell when the concentration of ZnSO4 and CuSO4 are 0.001 M and
0.1 M respectively. The standard potential of the cell is 1.1 V.

CENTERS : MUMBAI /DELHI /AKOLA /LUCKNOW /NASHIK /PUNE /NAGPUR /BOKARO /DUBAI # 59
ELECTROCHEMISTRY Rg. 2018 - 2020

20. Calculate the pH of following solutions.


(i) Pt, H2| HCl, E = 0.25 V (ii) Pt, H2 | H2SO4 , E = 0.30 V

21. The EMF of the cell M | M n+(0.02 M) || H+(1M) | H2(g) (1 atm), Pt at 25°C is 0.81V. Calculate the
valency of the metal if the standard oxidation potential of the metal is 0.76V.

CONCENTRATION CELLS

22. EMF of the cell Zn | ZnSO4( a = 0.2) || ZnSO4 (a2) | Zn is -0.0088 V at 25°C.
Calculate the value of a2.

23. Equinormal solutions of two weak acids, HA(pKa = 3) and HB(pKa = 5) are each placed in contact
with standard hydrogen electrode at 25°C. When a cell is constructed by interconnecting them
through a salt bridge, find the emf of the cell.

24. In two vessels each containing 500ml water, 0.5 m mol of aniline (Kb = 10-9 ) and 25mmol of HCl
are added separately. Two hydrogen electrodes are constructed using these solutions. Calculate the
emf of cell made by connecting them appropriately.

25. The emf of the cell Ag | AgI | KI(0.05M) || AgNO3 (0.022 M) | Ag is 0.769 V. Calculate the solubility
product of AgI.

26. Consider the cell Ag | AgBr(s) | Br– || AgCl(s) |Cl– | Ag at 25°C. The solubility product constants of
AgBr and AgCl are respectively 5 × 10-13 and 1 × 10-10. For what ratio of the concentrations of Br–
and Cl– ions would the emf of the cell be zero?

27. Calculate the emf of the cell


Pt, H2(1.0 atm) | CH3 COOH(0.1M) || NH3(aq. 0.01M) | H2(1.0 atm), Pt
Ka(CH3COOH) = 1.8 × 10 -5, Kb (NH3) = 1.8 × 10-5.

APPLICATIONS

28. The cell Pt, H2 (1atm) | H+(pH = x) || Normal calomel Electrode has an EMF of 0.67V at 25°C.
Calculate the pH of the solution. The oxidation potential of the calomel electrode on hydrogen scale
is -0.28V.

29. Voltage of the cell Pt, H2 (1 atm) | HOCN (10 -3M) || Ag+ (0.8M) | Ag(s) is 1.0006 V. Calculate the
Ka for HOCN. Neglect [H+ ] because of oxidation of H2(g).
Ag+ + e → Ag(s) = 0.8 V.

30. Calculate the voltage, E, of the cell at 25°C


Mn(s) | Mn(OH)2 (s) | OH– (1.00 × 10-4 M) || Cu2+(0.19 M) | Cu(s)
given that Ksp = 1.9 × 10-13 for Mn(OH)2(s) E° (Mn2+/Mn) = -1.18 V ,E°(Cu 2+/Cu) = 0.34 V

CENTERS : MUMBAI /DELHI /AKOLA /LUCKNOW /NASHIK /PUNE /NAGPUR /BOKARO /DUBAI # 60
ELECTROCHEMISTRY Rg. 2018 - 2020

31. The overall formation constant for the reaction of 6 mol of CN– with cobalt(II) is 1 × 1019. The
standard reduction potential for the reaction [Co(CN)6 ]3– + e– → Co(CN)64– is -0.83 V.
Calculate the formation constant of [Co(CN)6 ]3– ; Given Co3+ + e– → Co2+; E° = 1.82 V

32. The voltage of the cell


Zn(s) | Zn(CN)42–(0.450M), CN– (2.65 × 10-3 M) || Zn2+(3.84 × 10-4 M) | Zn(s)
is E = + 0.099 V. Calculate the constant Kf for Zn2+ + 4CN–  Zn(CN)42–, the only
complexation of importance.

33. The voltage of the cell


Pt | H2 (g, 608torr) | HClO (4.34 × 10-3 M) || S.C.E. is E = + 0.532V. Calculae the dissociation
constant for HClO. Given, E(SCE) = 0.242.

 E o 
34. At 27oC,   = − 1.25  10 −3 VK−1 and E o = 1.36 V for the cellPt|H2(g)| HCl(aq)||Cl- |Cl2 |Pt
 T p
Calculate entropy and enthalpy change in their standard state.

ELECTROLYTIC CELLS

35. How long a current of 2A has to be passed through a solution of AgNO3 to coat a metal surface of
80 cm2 with 5  m thick layer? Density of silver = 10.8 g/cm3.

36. 3A current was passed through an aqueous solution of an unknown salt of Pd for 160 min 50 sec.
5.32 gm of Pd+n was deposited at cathode. Find n. (Atomic wt. of Pd = 106.4)

37. A metal is known to form fluoride MF2. When 10A of electricity is passed through a molten salt for
330 sec., 1.95 g of metal is deposited. Find the atomic weight of M. What will be the quantity of
electricity required to deposit the same mass of Cu from CuSO4?

38. 50 mL of 0.1 M CuSO4 solution is electrolyzed with a current of 0.965 A for a period of 200 sec.
The reactions at electrodes are:
Cathode : Cu2+ + 2e– Cu(s) Anode : 2H2O O2 + 4H+ + 4e.
Assuming no change in volume during electrolysis, calculate the molar concentration of Cu2+, H+
and SO42– at the end of electrolysis.

39. 10 g fairly concentrated solution of CuSO4 is electrolyzed using 0.01 F of electricity. Calculate :
(a) The weight of resulting solution (b) Equivalent of acid or alkali in the solution.

40. An electric current is passed through electrolytic cells in series one containing Ag(NO3)(aq). and
other H2SO4 (aq). What volume of O2 measured at 27°C, 750 mm3 Hg pressure would be liberated
from H2 SO4 if
(a) one mole of Ag+ is deposited from AgNO3 solution
 1 
(b) 8 × 1022 ions of Ag+ are deposited from AgNO3 solution.  R  Lt atm,Na  6  1023 
 12 
CENTERS : MUMBAI /DELHI /AKOLA /LUCKNOW /NASHIK /PUNE /NAGPUR /BOKARO /DUBAI # 61
ELECTROCHEMISTRY Rg. 2018 - 2020

41. Cd amalgam is prepared by electrolysis of a solution of CdCl2 using a mercury cathode. How long
should a currecnt of 5A be passed in order to prepare 12% Cd-Hg amalgam on a cathode of 2gm Hg
(Cd = 112.4).

42. After electrolysis of NaCl solution with inert electrodes for a certain period of time. 600 mL of the
solution was left, which was found to be 1N in NaOH. During the same time, 31.75 g of Cu was
deposited in the copper voltammeter in series with the electrolytic cell. Calculate the percentage
yield of NaOH obtained.

43. Same quantity of electricity is being used to liberate iodine (at anode) and a metal x (at cathode). The
mass of x liberated is 0.54g and the iodine is completely reduced by 50 cc of 0.1 M sodium
thiosulphate. Find the equivalent mass of x.

44. The standard reduction potential values, E°(Bi3+|Bi) and E°(Cu2+ | Cu) are 0.226 V and 0.344 V
respectively. A mixture of salts of bismuth and copper at unit concentration each is electrolyzed at
25°C. To what value can [Cu2+ ] be brought down before bismuth starts to deposit, in electrolysis.

45. A current of 2amps is used for 60 min. to electrolyze copper from a solution of Cu2+ at a potential of
0.337 volts.
(a) How many coulombs are used
(b) How many moles Cu(s) are plated out?
(c) What weight Cu(s) in grams is plated out?
(d) What power in watts was used?

46. If you electrolyze a solution of Ni2+ (aq) to form Ni(s) for 2.00 h using a current of 2.50 amp, what
mass of Ni(s) (AW = 58.69 g/mol) is produced at the cathode?

47. A battery was used to supply a constant current of what was believed to be exactly 0.45 A as read on
a meter in the external circuit. The cell was based on the electrolysis of a copper sulphate solution.
During the 30 min, that current was allowed to flow, a total of 0.3 g of copper metal was deposited at
the cathode. Determine the extent to which the meter was inaccurate

48. A balloon is being filled with hydrogen produced by the electrolysis of an aqueous solution of an
acid. How long will it take to generate enough hydrogen to lift 1.5 kg by using a current of 8.5 A?
(M air = 29)

49. A current of 3.7 A is passed for 6 hrs. between Ni electrodes in 0.5L of 2M solution of Ni(NO3)2.
What is the molarity of Ni2+solution at the end of electrolysis?

CONDUCTANCE
50. The resistance of a conductivity cell filled with 0.01N solution of NaCl is 210 ohm at18oC.Calculate
the equivalent conductivity of the solution. The cell constant of the conductivity cell is 0.88 cm-1.

51. The molar conductivity of 0.1 M CH3COOH solution is 4.6 S cm2 mole-1 . What is the specific
conductivity and resistivity of the solution ?

52. The conductivity of pure water in a conductivity cell with electrodes of cross sectional area 4 cm2
and 2 cm apart is 8 x 10- 7 cm-1.
(i) What is resistance of conductivity cell ?
(ii) What current would flow through the cell under an applied potential difference of 1 volt?
CENTERS : MUMBAI /DELHI /AKOLA /LUCKNOW /NASHIK /PUNE /NAGPUR /BOKARO /DUBAI # 62
ELECTROCHEMISTRY Rg. 2018 - 2020

53. Resistivity of 0.1M KCl solution is 213 ohm cm in a conductivity cell. Calculate the cell constant if
its resistance is 330 ohm.

54. Resistance of a 0.1M KCl solution in a conductance cell is 300 ohm and specific conductance of
0.1M KCl is 1.29 × 10-2 ohm-1 cm-1. The resistance of 0.1M NaCl solution in the same cell is 380
ohm. Calculate the equivalent conductance of the 0.1M NaCl solution.
55. For 0.01N KCl, the resistivity 709.22 ohm cm. Calculate the conductivity and equivalent
conductance.

56. A solution containing 2.08 g of anhydrous barium chloride in 400cc of water has a specific
conductivity 0.0058 ohm–1 cm–1. What are molar and equivalent conductivities of this solution.
57. Equivalent conductance of 0.01 N Na2SO4 solution is 112.4 ohm–1 cm2 eq–1. The equivalent
conductance at infinite dilution is 129.9 ohm–1 cm2. What is the degree of dissociation in 0.01 N
Na2SO4 solution?

58. Specific conductance of a saturated solution of AgBr is 8.486×10–7 ohm–1cm–1 at 250 C. Specific

conductance of pure water at 25°C is 0.75 ×10–6 ohm–1 cm–1 .  m For KBr , AgNO3 and KNO3
are 137.4 , 133 , 131 ( S cm2 mol–1 ) respectively. Calculate the solubility of AgBr in gm/litre.

59. Saturated solution of AgCl at 25°C has specific conductance of 1.12×10–6 ohm–1 cm–1. The  e
(Ag+ )and   (Cl-)are54.3 and 65.5 ohm–1 cm2 eq.–1 respectively. Calculate the solubility product
e

of AgCl at 25°C.

60. Hydrofluoric acid is weak acid. At 25°C, the molar conductivity of 0.002M HF is

176.2 ohm–1 cm2 mole–1. If its  m = 405.2 ohm–1 cm2 mole–1, calculate its degree of dissociation
and equilibrium constant at the given concentration.


61. The value of  m for HCl, NaCl and CH3 COONa are 426.1, 126.5 and 91 S cm2 mol–1

respectively. Calculate the value of  m for acetic acid. If the equivalent conductivity of the given
acetic acid is 48.15 at 25° C, calculate its degree of dissociation.

62. Calculate the specific conductance of a 0.1 M aqueous solution of NaCl at room temperature, given
that the mobilities of Na+ and Cl– ions at this temperature are 4.26×10–8 and 6.80×10–8 m2 volt–1
s–1 , respectively.

63. For the strong electroytes NaOH, NaCl and BaCl2 the molar ionic conductivities at infinite dilution
are 248.1×10–4, 126.5 ×10–4 and 280.0 ×10-4 mho cm2 mol–1 respectively. Calculate the molar
conductivity of Ba(OH)2 at infinite dilution.

64. At 25°C,  m (H+) = 3.4982 ×10–2 S m2 mol–1 and  m (OH–)=1.498 ×10–2 S m2 mol–1.
Given: Sp. conductance = 5.1 ×10–6 S m–1 for H2O, determine pH and Kw.

CENTERS : MUMBAI /DELHI /AKOLA /LUCKNOW /NASHIK /PUNE /NAGPUR /BOKARO /DUBAI # 63
ELECTROCHEMISTRY Rg. 2018 - 2020

GET EQUIPPED FOR IIT - JEE


SINGLE CHOICE
1. One faraday of electricity will liberate one gram mole of the metal from the solution of :
(A) BaCl2 (B) CuSO4 (C) AlCl3 (D) NaCl

2. A solution of sodium sulphate in water is electrolysed using mercury cathode & Pt anode. The
products at cathodeand anode are respectively.
(A) H2 , O2 (B) O2 , H2 (C)Na – Hg, O2 (D) O2 , SO2

3. The amount of electricity that can deposite 108 g of silver from silver nitrate solution is :
(A) 1 ampere (B) 1 coulomb (C) 1 faraday (D) 2 ampere

4. Given standard electrode potentials :


Fe3+ + 3e-  Fe E0 = -0.036V
Fe2+ + 2e-  Fe E0 = -0.44V
The standard electrode potential E0 of Fe3+ + e-  Fe2+ is :
(A)-0.476 volt (B) -0.404 volt (C) 0.440 volt (D)0.772 volt

5. The standard reduction potential values of three metallic cations X+ , Y+ , Z+ are 0.52 , -3.03 & -1.18
V respectively. The order of reducing power of the corresponding metals is :
(A) Y > Z > X (B) X > Y > Z (C) Z > Y > X (D) Z > X > Y

6. The molar conductance of NaCl, HCl and CH3COONa at infinite dilution are 126.45, 426.16 & 91
ohm -1 cm2 mol-1 respectively. The molar conductance of CH3COOH at infinite dilution is:
(A) 201. 28 ohm-1cm2mol - 1 (B) 390.71 ohm -1 cm2 mol -1
(C) 698.28 ohm -1 cm2 mol -1 (D) 540.48 ohm -1 cm2 mol -1

7. The specific conductance of a 0.1 N KCl solution at 230 C is 0.0112 ohm - 1 cm - 1 . The resistance
of the cell containing the solution at the same temperature was found to be 55 ohm. The cell constant
will be :
(A) 0.142 cm - 1 (B) 0.918 cm - 1 (C) 1.12 cm - 1 (D) 0.616 cm - 1

8. Which of the following plots represents correctly variation of equivalent conductance with dilution
for a strong electrolytes?

 

(A) Dilution (B) Dilution


(C) Dilution (D)


Dilution
CENTERS : MUMBAI /DELHI /AKOLA /LUCKNOW /NASHIK /PUNE /NAGPUR /BOKARO /DUBAI # 64
ELECTROCHEMISTRY Rg. 2018 - 2020

9. As a lead storage battery is charged


(A) lead dioxide dissolves
(B) sulphuric acid is regenerated
(C) lead electrode becomes coated with lead sulphate
(D) the concentration of sulphuric acid decreases.

10. When 9.65 coulombs of electricity is passed through a solution of silver nitrate
( atomic mass of Ag = 108 g mol - 1 ) , the amount of silver deposited is :
(A) 16.2 mg (B) 21.2 mg (C) 10.8 mg (D) 6.4 mg

11. The charge required to deposit 9 g of Al from Al3+ solution is : (At. wt . of Al = 27.0)
(A) 32163.3 C (B) 96500 C (C) 9650 C (D) 32163 C

12. Which of the following reaction is not possible at cathode?


(A)2Cr3+ + 7H2O  Cr2O72- + 14H+ (B) F2  2F-
+
(C) ½ O2 + 2H  H2O (D) None of the above
13. When a sample of copper with zinc impurity is to be purified by electrolysis, the appropriate
electrodes are:
Cathode Anode
(a) Pure zinc Pure copper
(b) Impure sample Pure copper
(c) Impure zinc Impure sample
(d) Pure copper Impure sample
14. Consider the following standard reduction potentials:
Half reaction (V)

Ni2   aq  2e   Ni  s  E   0.23 V

Fe2   aq   2e  Fe  s  E  0.41V

Mn2   aq  2e  Mn  s  E   1.03 V

Co2   aq  2e  Co  s  E   0.28 V

Cr 3   aq  3e  Cr  s  E   0.74 V

Which of the following metals could be used successfully to galvanize steel :


(A) Ni only (B) Ni and Co (C) Fe only (D) Mn and Cr
15. Equivalent conductance of BaCl2 ,H 2SO 4 and HCl are x1 , x2 and x3 Scm 2 equiv-1 at infinite

dilution. If specific conductance of saturated BaSO 4 solution is y ohm 1cm 1 , then Ksp of BaSO 4

103 y 106 y
(a) (b) 2
2  x1  x2  2 x3  4  x1  x2  x3 

106 y 2 x1  x2  2 x3
(c) 2
(d)
4  x1  x2  x3  106 y 2

CENTERS : MUMBAI /DELHI /AKOLA /LUCKNOW /NASHIK /PUNE /NAGPUR /BOKARO /DUBAI # 65
ELECTROCHEMISTRY Rg. 2018 - 2020

16. Benzoic acid solution is titrated with NaOH conductometrically, graphical representation of the
titration is:

Conductance
Conductance

Conductance

Conductance
(A) (B) (C) (D)

NaOH NaOH NaOH NaOH

17. Ionization constant of a weak acid (HA) in terms of  m and m is:
Cm C 2m
(A) Ka  (B) Ka 
( m   m  ) m (m  m )
C (m )2
(C) Ka  (D) none of these
m (m  m )

18. A flashlight cell has the cathodic reaction 2MnO2(s) + Zn+2 + 2e   Zn Mn 2O4(s)
If the flashlight cell is to give out 4.825 mA, how long could it run if initially 8.7 g of the limiting
reagent MnO2 is present? [Mn = 55, O = 16]
(A) 2  10 6 sec (B) 4  106 sec (C) 6  106 sec (D) 8  106sec

19. A hydrogen electrode is placed in a buffer solution of acetic acid and sodium acetate in the ratio y:x
and x : y has oxidation potential values E1 and E2 volts respectively. pKa value for acetic acid is:
E  E2 E  E2  E1  E 2 2E1  E 2
(A) 1 (B) 1 (C) (D)
0.118 0.118 0.118 0.118

20.The emf of the following three galvanic cells:


1. Zn/Zn2+ (1M) || Cu 2+(1M)/Cu 2. Zn/Zn2+ (0.1M) || Cu2+(1M) / Cu
3. Zn/Zn2+ (1M) || Cu 2+(0.1M)/Cu
are represented by E1,E2,E3. Which of the following statement is true?
(A) E1 E2 E3 (B) E3 E2 E1 (C) E3 E1 E2 (D) E2 E1 E3

MORE THAN ONE CORRECT

21. Which one of the following statements is(are) true regarding galvanic and electrolytic cells?
(A) In both cells, redox reactions take place.
(B) In electrolytic cells, a current is forced through a cell to produce a chemical change for which the
cell potential is negative.
(C) In both cells, reduction takes place at the cathode.
(D) The ion flow through the salt bridge in galvanic cell is opposite to the flow in electrolytic cell.

22. Which one of the following statements is (are) false in a concentration cell made of Fe in 0.010 M
Fe2+(aq) and Fe in 0.10 M Fe2+?
o
(A) The ECell =0 (B)G = 0
(C) The cell reaction is: Fe (aq, 0.010 M) → Fe2+(aq, 0.10 M)
2+

(D) At equilibrium, the [Fe2+]anode = [Fe2+]cathode

23. The emf of a galvanic cell depends upon


(A) Surface area of electrolyte (B)Concentration of electrolyte
(C) Volume of electrolyte (D)Temperature

CENTERS : MUMBAI /DELHI /AKOLA /LUCKNOW /NASHIK /PUNE /NAGPUR /BOKARO /DUBAI # 66
ELECTROCHEMISTRY Rg. 2018 - 2020

24. A chemist found that the standard electrode potential Eo (Zn2+/Zn) = −0.76 V. Which spontaneous
cell (s) will give magnitude of EMF = 0.76 V?
(A) Zn | Zn2+ (1 M) || H+ (1 M) | H2 (1atm) | Pt
(B) Zn | Zn2+ (4 M) || H+ (2 M) | H2 (1 atm) | Pt
(C) Zn | Zn2+ (10 M) || H+ (10 M) H2 (1 atm) | Pt
(D) Zn | Zn2+ (2 M) || H+ (2 M) | H2 (2 atm) | Pt

25. Standard reduction potentials of various reactions involving bromine are given as
1
BrO 4 1.74 V  1.47 V 1.6 V
 BrO 3  HOBr   Br2(l)   Br−
1.07

2
Given that HOBr undergoes disproportionate ion, which of the following set of products are
possible?
(A) BrO 4 , Br− (B) BrO 4 , Br2 (C) BrO 3 , Br2 (D) BrO 3 , Br−

26. Consider the cell Pt | H2 (P1 atm) | H+(x1 M)| | H+(x2 M) |H2 (P2 atm)| Pt. The cell reaction will be
spontaneous if
(A) p 1 = p2 and x1> x2 (B)p1 = p 2 and x1< x2
(C) x1 = x2and p 1>p2 (D) x1 = x2and p 1<p 2

27. In a standard solution of X  and Y  ions, gas Z2 is passed. X 2 is liberated and Z  is formed while
Y  remain unaffected. E xo , E yo and E zo represent standard reduction potentials of X2, Y2 and Z2
respectively. Which of the following can be inferred from given information?
1
(A) E zo > E xo (B) E zo > E yo (C) E yo > E xo (D) E zo  E xo  E yo 
2
28. When a lead storage battery discharges
(A) PbSO4 is formed (B) Pb is formed
(C) SO2 is consumed (D) H2SO4 is consumed

29. During electrolysis, O2(g) is evolved at anode in electrolysis of :


(A) dil. H2SO4 with Pt electrode (B) aqueous AgNO3 with Pt electrode
(C) dil. H2SO4 with Cu electrode (D) fused NaOH

30. The specific conductivity of a saturated solution of AgCl is 3.4  10 −6 S cm−1 at 25 oC.
If (Ag+) = 62.3 S cm2 mol−1, (Cl−) = 67.7 S cm2 mol−1, solubility of AgCl at 25 oC is
(A) 2.6  10−5 M (B) 3.75  10−3 g/L (C) 3.7  10−5 M (D) 2.6  10−3 g/L

31. A redox reaction occurs spontaneously if


(A) the free energy change is negative (G< 0)
(B) G> 0
(C) the standard state cell emf (Eo) is negative
(D) the emf of cell (ECell) is positive.
32. During electrolysis of aqueous CuBr2(aq) using Pt electrode.
(A)Br2(g) evolved at anode. (B)Cu(s) is deposited at cathode.
(C)Br2(g) is evolved at anode and H2(g) at cathode (D)H2(g) is evolved at anode.
33. A current of 2.68 A is passed for 1.0 h through an aqueous solution of CuSO4 using copper
electrodes. Select the correct statement(s) from the following. (Cu = 63.5)
(A) Increase in mass of cathode = 3.174 g
(B) Decrease in mass of anode = 3.174 g
(C) No change in mass of electrodes
(D) The ratio between the change in mass of cathode to anode is 1:2

CENTERS : MUMBAI /DELHI /AKOLA /LUCKNOW /NASHIK /PUNE /NAGPUR /BOKARO /DUBAI # 67
ELECTROCHEMISTRY Rg. 2018 - 2020

34. Ag | Ag+ ||KI| AgI | Ag emf is E0 , then Ksp of AgI is given as :


nF  E 0 
(A) K sp  log E 0 (B) ln K sp  nF   E0 
2.303RT  T 
nF nFE 0
(C) ln K sp  (D) log Ksp =
Eo 2.303RT
35. The standard emf of the following cell:
Cd(s) | CdCl2(aq) (0.10 M) ||Cl -| AgCl(s) | Ag(s)
In which the cell reaction is 2AgCl(s) + Cd(s) → 2Ag(s) + Cd 2+(aq) + 2Cl−(aq) is 0.6915 V at 100C
and 0.6753 V at 250C. The H of reaction at 25 oC is
(A) −192.5 kJ (B) −234.7 kJ (C) 123.5 kJ (D) −167.26 kJ

36. When 4.00 A of current is passed through a 1.0L 0.10 M Fe3+(aq) solution for 1.0 hr, it is partly
reduced to Fe(s) and partly to Fe2+(aq). The correct statement(s) is (are)?
Assume F = 96000.
(A) 0.10 mole of electrons are required to convert all Fe3+ to Fe2+.
(B) 0.025 mole of Fe(s) will be deposited.
(C) 0.075 mole of iron remains as Fe2+
(D) 0.050 mole of iron remains as Fe2+

37. For which electrolyte   v doesn’t hold good?

(A) CH3OH (B) HClO4 (C) HCOOH (D) NaNH2

38. When a galvanic cell starts operating, with passage of time,


(A) spontaneous character of the cell decreases (B) reaction quotient ‘Q’ decreases, ECell increases.
(C) useful work obtainable from the cell increases. (D) as cell stops producing voltage, Q = K.

39. Which of the following is/are true statement(s)?


(A) If Eo (Mn+/ M) is negative (< 0), H+ can be reduced to H2 by the metal.
(B) If Eo(Mn+/M) is positive, Mn+ can be reduced to M by H2.
(C) In a cell Mn+/M is attached to a hydrogen electrode. For a spontaneous cell reaction. M will act
as negative electrode if Eo(Mn+/M) is positive.
(D) Compounds of active metals like Zn, Na, Mg etc. are reduced by H2 while compounds of noble
metals as of Cu, Ag and Au are not reduced.

40. Given that, Ni2+ / Ni = 0.25 V, Cu2+ / Cu = 0.34 V,


Ag+ / Ag = 0.80 V and Zn2+ / Zn = –0.76 V
Which of the following reactions under standard condition will not take place in the specified
direction?
(A) Ni  (aq)  Cu (s)  Ni(s)  Cu 2  (aq) (B) Cu s  2Ag  aq  Cu 2 aq  2Ag s
(C) Cu(s)  2H  (aq)  Cu 2  (aq)  H 2 (g) (D) Zn s  2H  aq  Zn 2 aq  3H 2 g

41. Consider the following half reaction:


A(s) An+(aq) + ne− [An+] > 1.00 M. The correct statement(s) is/are?
(A) Electrode potential E(A/An+)decreases with increase in [An+]
(B) With constant [An+], E (An+/A) increases with rise in temperature
(C) At constant temperature E(An+/A) increases on increasing [An+]
(D) With constant [An+], E(A/An+) increases with rise in temperature

CENTERS : MUMBAI /DELHI /AKOLA /LUCKNOW /NASHIK /PUNE /NAGPUR /BOKARO /DUBAI # 68
ELECTROCHEMISTRY Rg. 2018 - 2020

42. Which of the following is (are) correct thermodynamic relationship?


(A) Go = −2.303 RT log K (B) G = Go+ 2.303 RT log K
0.0591 0.0591
o
(C) ECell  log K at 25 oC (D) E  E o  log K at 25 oC
n n

43. Consider the following electrochemical cell at 25 oC. Pb(s) | PbSO4 (saturated)|| PbI2(saturated)| Pb(s)
Which of the following expression can be used to determine emf of the cell?
0.0592  [ Pb 2 ]anode  0.0592  [ Pb 2 ]anode 
(A) E   log  2

 (B) E  log  2

2  [ Pb ]cathode  2  [ Pb ]cathode 
  K sp 1 / 3 
 
  4 
0.0592   PbI 2  0.0592  ( K sp ) PbI 2 
 
(C) E  log   (D) E  log
2 1/ 2
 ( K sp ) PbSO4  2  ( K sp ) PbSO 
 4 
 
 
 
44. Consider the following galvanic cell Cd(s) | Cd 2+ (1.0 M) || Cu2+ (1.0 M) | Cu(s)
If we wish to make a cell with a more positive value of emf using the same substances, we should
(A) Increase [Cd 2+] as well as [Cu2+] to 2.0 M (B)Reduce only [Cd 2+] to 0.1 M
(C) Increase only [Cu 2+] to 2.0 M (D)Decrease [Cd +] as well as [Cu 2+] to 0.10 M

45. In electrolysis of very dilute NaOH solution using platinum electrodes:


(A) H2 is evolved at cathode (B) H2 is produced at anode
(C) Na is obtained at cathode (D) O2 is produced at anode
COMPREHENSION TYPE
PASSAGE 1
The driving force  G diminishes to zero on the way to equilibrium, just as in any other spontaneous
process. Both  G and the corresponding cell potential are zero when the redox reaction comes to
equilibrium. The Nernst equation for the redox process of the cell may be given as:
RT
ECell  E o Cell  ln Q
nF
At equilibrium, the Nernst equation is given as :
0.0592V
Eo  log K
n
1. On the basis of information available for the reaction:
4 2
Al  O2  Al2O3 ; G   827 kJ / mole of O2 the minimum emf required to carry
3 3
out an electrolysis of Al2O3 is : ( Given : 1 F = 96500 C )
(A) 2.14 V (B) 4.28 V (C) 6.42 V (D) 8.56 V

2. The equilibrium constant Kc will be equal to Q when :


(A) E = EO (B) RT /nF = 1 (C) E = 0 (D) EO = 1

3. The nature of graph of Eocell against log Kc is a /an :


(A) Straight line (B) Parabola (C) Hyperbola (D) Elliptical curve

4. The equilibrium constant Kc for the reaction:


Cu ( s )  2 Ag  (aq )  Cu 2   2 Ag ( s ) Ecell
0
 0.46V will be :
(A) antilog 15.6 (B) antilog 2.5 (C) antilog 1.5 (D) antilog 12.2

CENTERS : MUMBAI /DELHI /AKOLA /LUCKNOW /NASHIK /PUNE /NAGPUR /BOKARO /DUBAI # 69
ELECTROCHEMISTRY Rg. 2018 - 2020

5. E0 for the electrochemical cell ,


Zn ( s )  Cu 2   Cu ( s )  Zn 2  ( aq ) is 1.1 volt at 25 oC . The equilibrium constant for the cell reaction
will be:
(A) 10 -37 (B) 10 37 (C) 10 - 39 (D) 1039

PASSAGE 2
Conductors allow the passage of electric current through them. Metallic and electrolytic are the two
types of conductors. Charge carriers in metallic and electrolytic conductors are free electrons and
free ions respectively. Specific conductance or conductivity of the electrolyte solution is given by the
l
following relation:    
A.
l
where,  = 1/R is the conductance and is the cell constant. Molar conductance  m and
A
equivalence conductance  e of an electrolyte solution are calculated using the following similar
1000 1000
relations : m    e  
M N
where, M and N are the molarity and normality of the solution respectively . Molar conductance of
strong electrolyte depends on concentration by the relation:
 m   0m  b c
where 0m = molar conductance at infinite dilution
c = concentration of the solution b = constant

1. Which of the following decreases on dilution of electrolyte solution?


(A) Equivalent conductance (B) Molar conductance
(C) Specific conductance (D) none

2. The correct order of equivalent conductance’s at infinite dilution of LiCl , NaCl and KCl is :
(A) LiCl > NaCl > KCl (B) KCl > NaCl > LiCl
(C) NaCl > KCl > LiCl (D) LiCl > KCl > NaCl

3. The conductance of a solution of an electrolytes is equal to that of its specific conductance. The cell
constant of the conductivity cell is equal to :
(A) resistance (B) faraday (C) zero (D) unity

4. Which of the following equality holds good for the strong electrolytes?
(A)     as c  1 (B)    0 as c  0
(C)    0 as c   (D)    0 as c  b

PASSAGE 3
At infinite dilution, when the dissociation of electrolyte is complete , each ion makes a definite
contribution towards the molar conductance of electrolyte, irrespective of the nature of the other ion
with which it was associated. The molar conductance of an electrolyte at infinite dilution can be
expressed as the sum of the contributions from its individual ions.
Ax By  xA y   yB x 
 0m ( Ax B y )  x A0 y   yB0x 

CENTERS : MUMBAI /DELHI /AKOLA /LUCKNOW /NASHIK /PUNE /NAGPUR /BOKARO /DUBAI # 70
ELECTROCHEMISTRY Rg. 2018 - 2020

where , x and y are the number of cations and anions respectively. The degree of ionisation of weak

electrolyte can be calculated as :   0m
m

1. The ionic conductance’s of Al3+ and SO42 ions at infinite dilution are x and y ohm -1 cm2 mol-1
respectively. If Kohlrausch’s law is valid , then molar conductance of aluminium sulphate at infinite
dilution will be :
(A) 3x + 2y (B) 3y + 2x (C) 2x + 2y (D) 3x + 3y

2. The molar conductance at infinite dilution for electrolytes BA and CA are 140 &120ohm - 1 cm2 mol-1.
If the molar conductance at infinite dilution of BX is 198 ohm - 1 cm2 mol-1, then at infinite dilution ,
the molar conductance of CX is :
(A) 178 (B) 198 (C) 218 (D) 130

3. The molar conductance of 0.001 M acetic acid is 50 ohm - 1 cm2 mol-1 . The maximum value of molar
conductance of acetic acid is 250 ohm - 1 cm2 mol-1 . What is the degree of dissociation of acetic
acid?
(A) 0.5 (B) 0.2 (C) 0.3 (D) 0.4

4. Which of the following solutions will have highest value of the molar conductance of CH3COOH?
(A) 1 M CH3COOH (B) 0.5 M CH3COOH
(C) 0.3M CH3COOH (D) 0.1MCH3COOH

5 Which is not the unit of molar conductance of an electrolyte solution:


(A) ohm -1 cm2 mol-1 (B) mho cm2 mol-1
(C) S cm2 mol-1 (D) ohm -1 cm-1mol-1

PASSAGE 4

The magnitude (but not the sign) of the standard reduction potentials of two metals X and Y are:
Y +2 + 2e-  Y | E10 | = 0.34 V
+2 -
X + 2e  X | E10 | = 0.25 V

When the two half-cells of X & Y are connected to construct a cell, electrons flow from X to Y.
When X is connected to standard hydrogen electrode (SHE), electrons flow from X to SHE.

1. Which of the following sign of E1o and E2 o are true?


o o
(A) Both are negative (B) E1 is negative and E2 is positive
(C) Both are positive (D) E1o is positive and E2 o is negative

2. If a half-cell X/X2+ (0.1 M) is connected to another half-cell Y/Y+2(1.0 M) by means of a salt bridge
and an external circuit at 25 oC, the cell voltage would be
(A) 0.06 V (B) 0.12 V (C) 0.62 V (D) 0.72 V

3. If standard emf (Eo) of a half-cell Y2+/Y+ is 0.15 V, the standard emf of the half-cell Y+/Y will be?
(A) 0.19 V (B) 0.53 V (C) 0.49 V (D) 0.64 V

CENTERS : MUMBAI /DELHI /AKOLA /LUCKNOW /NASHIK /PUNE /NAGPUR /BOKARO /DUBAI # 71
ELECTROCHEMISTRY Rg. 2018 - 2020

INTEGER TYPE
1. Equivalent conductance of 0.2 M (aq.) solution of a weak monobasic acid HA, is 10 S cm 2 eq-1
and that at infinite dilution is 200 S cm2 eq -1. What is the pH of the solution?

2. How many faradays of electricity is required for electrolysis of 72 gm of acidified water.

3. If an aq. NaCl solution is electrolyzed using a current of 5A for 200 min, volume of Cl2 gas in L
produced under STP conditions is _____ (F = 96500 C)

4. The charge required for discharging 115 gm of Na from molten NaCl is ____ F

ADVANCE CONCEPT TESTERS

1. During the discharge of a lead storage battery the density of sulphuric acid fell from 1.294 to 1.139
g. ml-1. H2SO4 of density 1.294 g mL-1 is 39% and that of density 1.139 g mL-1 is 20% by weight. The
battery holds 3.5L of acid and the volume practically remains constant during the discharge.
Calculate the number of ampere hours for which the battery must have been used. The discharging
reactions are:
Pb + SO42– PbSO4 + 2e-(anode) : PbO2 + 4H+ + SO42– + 2e– PbSO4 + 2H2O (cathode)

2. Assume that impure copper contains only iron, silver and gold as impurities. After passage of 140 A,
for 482.5s of the mass of the anode decreased by 22.260g and the cathode increased in mass by
22.011 g. Estimate the % iron and % copper originally present.

3. 100 ml CuSO4(aq) was electrolyzed using inert electrodes by passing 0.965 A till the pH of the
resulting solution was 1. The solution after electrolysis was neutralized, treated with excess KI and
titrated with 0.04 M Na2S2O3. Volume of Na2S2O3 required was 35 ml. Assuming no volume change
during electrolysis, Calculate :
(a) duration of electrolysis if current efficiency is 80%
(b) initial concentration (M) of CuSO4.

4. Calculate the cell potential of a cell having reaction: Ag2S + 2e–  2Ag + S2– in a solution buffered
at pH = 3 and which is also saturated with 0.1 M H2S.
For H2S : K1 = 10 –8 and K2 = 1.1 × 10–13, Ksp(Ag2S) = 2 × 10 –49, EAg  / Ag  0.8.

5. One of the methods of preparation of per disulphuric acid, H2S2O8, involve electrolytic oxidation of
H2SO4 at anode (2H2SO4 H2S2O8 + 2H+ + 2e–) with oxygen and hydrogen as by-products. In such
electrolysis, 9.722 L of H2 and 2.35 L of O2 were generated at STP. What is the weight of H2S2O8
formed?

6. In a fuel cell, H2 and O2 react to produce electricity. In the process, H2 gas is oxidized at the anode
and O2 at the cathode. If 67.2 litre of H2 at STP react in 15 minutes, what is the average current
produced? If the entire current is used for electrode deposition of Cu from Cu(II) solution, how many
grams of Cu will be deposited?
Anode : H2 +2OH– 2H2O + 2e- Cathode : O2 + 2H2O + 4e– 4OH–

CENTERS : MUMBAI /DELHI /AKOLA /LUCKNOW /NASHIK /PUNE /NAGPUR /BOKARO /DUBAI # 72
ELECTROCHEMISTRY Rg. 2018 - 2020

7. Calculate the potential of an indicator electrode versus the standard hydrogen electrode, which
originally contained 0.1M MnO4 and 0.8 M H+ and which was treated with 90% of the Fe2+
necessary to reduce all the MnO4 to Mn+2. MnO4 + 8H+ + 5e  Mn2+ + H2O, E° = 1.51 V.

8. The pKsp of AgI is 16.07. If the E° value for Ag+ | Ag is 0.7991 V. Find the E° for the half cell
reaction AgI(s) + e– Ag + I–.

9. Estimate the cell potential of a Daniel cell having 1 M Zn++ and originally having 1 M Cu ++ after
sufficient NH3 has been added to the cathode compartment to make NH3 concentration 2M.
Kf for [Cu(NH3)4]2+ = 1 × 1012, E° for the reaction,
Zn + Cu2+ Zn2+ + Cu is 1.1V.

10. Kd for complete dissociation of [Ag(NH3)2]+ into Ag+ and 2NH3 is 6 × 10 -8. Calculate E° for the
following half reaction; Ag(NH3) 2 + e– Ag + 2NH3
Ag+ + e– Ag, E° = 0.799 V

11. Calculate the equilibrium constant for the reaction:


3Sn(s) + 2Cr2O72– + 28H+ 3Sn4+ + 4Cr3+ + 14H2O
E° for Sn/Sn2+ = 0.136 V E° for Sn2+/ Sn4+ = - 0.154V
E° for Cr2O72–/Cr3+ = 1.33V

12. Calculate the equilibrium concentration of all ions in an ideal solution prepared by mixing 25.00 mL
of 0.100 M Tl+ with 25.00 mL of 0.200 M Co3+.
E°(Tl+/Tl3+) = - 1.25 V; E° (Co3+/Co2+) = 1.84 V

13. An acidic solution of Cu2+ salt containing 0.4 g of Cu2+ is electrolyzed until all the copper is
deposited. The electrolysis is continued for seven more minutes with the volume of solution kept at
100 ml and the current at 1.2 amp. Calculate the volume of gases evolved at NTP during the entire
electrolysis.

14. Hydrogen peroxide can be prepared by successive reactions:


2NH4HSO4 H2 + (NH4)2S2O8
(NH4)2S2O8 + 2H2O  2NH4HSO4 + H2O2

The first reaction is an electrolytic reaction the second is steam distillation. What amount of current
would have to be used in first reaction to produce enough intermediate to yield 100 gm pure H2O2
per hour? Assume 50% anode current efficiency.

15. In the refining of silver by electrolytic method what will be the weight of 100 gm Ag anode if
5 ampere current is passed for 2 hours? Purity of silver is 95% by weight.

16. Dal lake has water 8.2 ×10 12 litre approximately. A power reactor produces electricity at the rate of
1.5×106coulomb per second at an appropriate voltage.How many years would it take to electrolyze
the lake?

17. The equivalent conductance of 0.10 N solution of MgCl2 is 97.1 mho cm2 equi–1 at 25°C. a cell with
electrode that are 1.5 cm2 in surface area and 0.5 cm apart is filled with 0.1 N MgCl2 solution. How
much current will flow when potential difference between the electrodes is 5 volt.

CENTERS : MUMBAI /DELHI /AKOLA /LUCKNOW /NASHIK /PUNE /NAGPUR /BOKARO /DUBAI # 73
ELECTROCHEMISTRY Rg. 2018 - 2020

18. A dilute aqueous solution of KCl was placed between two electrodes 10 cm apart, across which a
potential of 6 volt was applied. How far would the K+ ion move in 2 hours at 25°C? Ionic
conductance of K+ ion at infinite dilution at 25°C is 73.52 ohm–1 cm2 mole–1?

19. When a solution of specific conductance 1.342 ohm–1 metre–1 was placed in a conductivity cell with
parallel electrodes, the resistance was found to be 170.5 ohm. Area of electrodes is 1.86×10–4 m2.
Calculate separation of electrodes.

20. The specific conductance at 25°C of a saturated solution of SrSO4 is 1.482×10–4 ohm–1 cm–1while
that of water used is 1.5×10–6 mho cm–1. Determine at 25°C the solubility in gm per litre of SrSO4 in
water. Molar ionic conductance of Sr2+ and SO42– ions at infinite dilution are 59.46 and 79.8 ohm–1
cm2 mole–1 respectively.
[ Sr = 87.6 , S = 32 , O = 16 ]

21. The voltage of the cell Pb(s) | PbSO4(s) | NaHSO4(0.600M) || Pb 2+(2.50 x 10–5M) | Pb(s)
is E = +0.061 V. Calculate K2, the dissociation constant forHSO4– .
Given : Pb (s) + SO42–(aq)  PbSO4 (s) + 2e– (E0 = 0.356) E0(Pb 2+/Pb) = –0.126 V

22. Determine at 298 for cell


Pt | Q, QH2, H+ || 1M KCl | Hg2Cl2(s) | Hg(l) | Pt
(A)it's emf when pH = 5.0
(B)the pH when Ecell = 0
(C)the positive electrode when pH = 7.5
given E0RP(RHS) = 0.28, E0RP(LHS) = 0.699

23. What is the voltage of the redox titration cell [SCE || redox titration mixture | Pt] when 5.00 mL of
0.2 M Ce4+ has been added in the titration of 20.00 mL of 0.100M Sn2+? Also find the voltage at the
equivalence point.
E°(Sn2+/Sn4+) = -0.154 V E°(Ce4+/Ce3+) = 1.44 V, E 0 SCE  0.242

24. 25.00 mL of 0.240M Cu + is titrated with 0.0500 M Cr2O72–,


using the cell [SCE || redox titration mixture | Pt]
Cr2O72- + 6Cu + + 14H+ 2Cr3++ 6Cu2++ 7H2O
E°(Cu2+/Cu+) = 0.153 V, E°(Cr2O72- |Cr3+) = 1.33V.
Both solution are buffered to pH = 2.000. What are the cell voltages when (a) 5.00 mL, (b) 10.00
mL and (c) 15.00 mL, of Cr2O72- are added.

25. A lead storage cell is discharged, which causes the sulphuric acid electrolyte to change from a
concentration of 34.5 weight percent (density 1.261 g ml-1 at 25°C) to one of 27 weight percent. The
original volume of electrolyte is 1 litre which remains constant. How many Faradays have left the
anode of the battery?
Overall reaction Pb(s) + PbO2(s) + 2H2SO4(l)  2PbSO4(s) + 2H2O(l)

26. A sample of water from a large swimming pool has a resistance of 9200  at 25°C when placed in a
certain conductance cell. When filled with 0.02 M KCl solution, the cell has a resistance of 85  at
25°C. 500 gm of NaCl were dissolved in the pool, which was thoroughly stirred. A sample of this
solution gave a resistance of 7600 . Calculate the volume of water in the pool.
CENTERS : MUMBAI /DELHI /AKOLA /LUCKNOW /NASHIK /PUNE /NAGPUR /BOKARO /DUBAI # 74
ELECTROCHEMISTRY Rg. 2018 - 2020

Given : Molar conductance of NaCl at that concentration is 126.5 –1 cm2 mol–1 and molar
conductivity of KCl at 0.02 M is 138 –1 cm2 mol–1.

27. Calculate the solubility and solubility product of Co 2 [Fe(CN)6] in water at 25 0C from the following
data:
Conductivity of a saturated solution of Co2[Fe(CN)6] is 2.06 × 10–6–1 cm–1 and that of water
used 4.1 × 10 –7–1 cm–1 . The ionic molar conductivities of Co2+ and Fe(CN)64– are 86.0 –1 cm2
mol–1 and 444.0 –1 cm–1mol–1.

WINDOW TO IIT-JEE
1. The standard reduction potentials at 298K for the following half cells are given:
Zn2+ (aq) + 2e-⇌ Zn (s) E° = -0.762V
Cr3+ (aq) + 3e-⇌ Cr(s) E° = - 0.740 V
+ -
2H (aq) + 2e ⇌ H2 (g) E° = 0.000 V
Fe3+ (aq) + e-⇌ Fe2+ (aq) E° = 0.770 V
Which is the strongest reducing agent? (JEE 1981)
2+
(a) Zn(s) (b) Cr(s) (c) H2(g) (d) Fe (aq)

2. Faraday’s laws of electrolysis are related to the (JEE 1983)


(a) atomic number of the cation
(b) atomic number of the anion
(c) equivalent weight of the electrolyte
(d) speed of the cation

3. A solution containing one mole per litre of each Cu(NO3)2, AgNO3, Hg2(NO3)2 and Mg(NO3)2 is
being electrolysed by using inert electrodes. The values of standard electrodepotentials in volts
(reduction potential ) are
Ag+/Ag = + 0.80, Hg 22 / 2 Hg  0.79
Cu2+/Cu = + 034, Mg2+/Mg = - 2.37
With increasing voltage, the sequence of deposition of metals on the cathode will be
(JEE 1984)
(a) Ag, Hg, Cu, Mg (b) Mg, Cu, Hg, Ag (c) Ag, Hg, Cu (d) Cu, Hg, Ag

4. The electric charge for electrode deposition of one gram equivalent of a substance is(JEE 1984)
(a) one ampere per second
(b) 96,500 coulombs per second
(c) one ampere for one hour
(d) charge on one mole of electrons

5. The reaction,
1
H 2 ( g )  AgCl ( s)  H  ( aq )  Ag ( s)  Cl  ( aq ) occurs in the galvanic cell (JEE 1985)
2
(a) Ag│AgCl (s) │ KCl(soln)│AgNO3│Ag
(b) Pt│H2(g) │ HCl (soln)│ AgNO3(soln) │Ag
(c) Pt│H2 (g)│HCl(soln)│AgCl(s)│Ag
(d) Pt/H2(G) │ KCl (soln) │AgCl(s) │Ag

CENTERS : MUMBAI /DELHI /AKOLA /LUCKNOW /NASHIK /PUNE /NAGPUR /BOKARO /DUBAI # 75
ELECTROCHEMISTRY Rg. 2018 - 2020

6. When a lead storage battery is discharged (JEE 1986)


(a) SO2 is evolved
(b) lead is formed
(c) lead sulphate is consumed
(d) sulphuric acid is consumed

7. The standard reduction potentials E°, for the half reactions are as
Zn = Zn2+ + 2e- , E° = +0.76V
Fe = Fe2+ + 2e-, E° = 0.41 V
The emf for the cell reaction,
Fe2+ + Zn → Zn2+ + Fe is (JEE 1989)
(a) -0.35V (b) +0.35V (c) +1.17 V (d) -1.17V

8. The standard reduction potentials of Cu2+/Cu and Cu2+/Cu + are 0.337 V and 0.153V respectively.
The standard electrode potential of Cu +/Cu half-cell is (JEE 1997)
(a) 0.184 V (b) 0.827 V (c) 0.521 V (d) 0.490V

9. The standard reduction potential values of three metallic cations, X,Y,Z are 0.52, -3.03 & -1.18 V
respectively. The order of reducing power of the corresponding metals is (JEE 1998)
(a) Y > Z > X (b) X > Y > Z (c) Z > Y > X (d) Z > X > Y

10. The gas X at 1 atm is bubbled through a solution containing a mixture of 1 M Y- and 1 MZ- at 25°
C, If the order of reduction potential is Z < Y > X, then (JEE 1999)
(a) Y will oxidise X and not Z
(b) Y will oxidise Z and not X
(c) Y will oxidise both X and Z
(d) Y will reduce both X and Z

11. For the electrochemical cell, (M/M+) || (X-│X), E°(M+/M) = 0.44 V and E° (X/X-) = 0.33 V.
From this data one can deduce that (JEE 2000)
(a) M + X → M+ + X- is the spontaneous reaction
(b) M+ + X- → M + X is the spontaneous reaction
(c) Ecell = 0.77 V
(d) Ecell = - 0.77 V

12. The correct order of equivalent conductance at infinite dilution of LiCl, NaCl and KCl is
(a) LiCl>NaCl>KCl (JEE 2001)
(b) KCl>NaCl>LiCl
(c) NaCl>KCl>LiCl
(d) LiCl>KCl>NaCl

13. Saturated solution of KNO3 is used to make ‘salt-bridge’ because (JEE 2001)
(a) Velocity of K+is greater than that of NO3
(b) Velocity of NO3 is greater than that of K+
(c) Velocities of both K+ and NO3 are nearly the same
(d) KNO3 is highly soluble in water

CENTERS : MUMBAI /DELHI /AKOLA /LUCKNOW /NASHIK /PUNE /NAGPUR /BOKARO /DUBAI # 76
ELECTROCHEMISTRY Rg. 2018 - 2020

14. Standard electrode potential data are useful for understanding the suitability of an oxidant in a
redox titration. Some half-cell reactions and their standard potentials are given below:
MnO4 ( aq )  8 H  ( Aq )  5e   Mn 2 ( aq )  4 H 2O ( l ), E   1.51V
Cr2O72  ( aq )  14 H  ( aq )  6e   2Cr 3 ( aq )  7 H 2O (l ), E   1.38V
Fe3+(aq) + e- → Fe2+(aq) E° = 0.77 V
- -
Cl2(g) + 2e → 2Cl (aq) E° = 1.40 V
Identify the incorrect statement regarding the quantitative estimation of aqueous Fe(NO3)2
(JEE 2002)
(a) MnO4- can be used in aqueous HCl
(b) Cr2O72- can be used in aqeousHCl
(c) MnO4- can be used in aqueous H2SO4
(d) Cr2O72- can be used in aqueous H2SO4

15. In the electrolytic cell, flow of electrons is from (JEE 2003)


(a) cathode to anode in solution
(b) cathode to anode through external supply
(c) cathode to anode through internal supply
(d) anode to cathode through internal supply

16. Zn │ Zn2+(a = 0.1 M) ||Fe2+ (a = 0.01 M) │Fe.


The emf of the above cell is 0.2905 V. Equilibrium constant for the cell reaction is
(JEE 2004)
(a) 100.32/0.059 (b) 100.32/0.0295 (c) 10 0.26/0.0295 (d) 100.32/0.295

17. The half cell reactions for rusting of iron are:


1
2H+ + 2e- + O2 → H2O(l); E° = +1.23 V
2
Fe2++ 2e- → Fe(S). E° = -0.44V
∆ G° (in kJ) for the reaction is (JEE 2005)
(a) -76 (b) -322 (c) -122 (d) -176

18. Electrolysis of dilute aqueous NaCl solution was carried out by passing 10mA current. The time
required to liberate 0.01 mole of H2 gas at the cathode is (1 F = 96500C mol-1)
(JEE 2008)
(a) 9.65 × 104 s (b) 19.3 × 104 s (c) 28.95 × 10 4 s (d) 38.6 × 10 4s

19. AgNO3 (aqueous) was added to an aqueous KCl solution gradually and the conductivity of the
solution was measured. The plot of conductance (  ) versus the volume of AgNO3 is

(JEE 2011)
(a) (P) (b) (Q) (c) (R) (d) (S)
CENTERS : MUMBAI /DELHI /AKOLA /LUCKNOW /NASHIK /PUNE /NAGPUR /BOKARO /DUBAI # 77
ELECTROCHEMISTRY Rg. 2018 - 2020

20. Consider the following cell reaction,


2Fe(s) + O2(g) + 4H+(aq) → 2Fe+ (aq) + 2H2 O(l), E° = 1.67 V
At [Fe2+] = 10 -3 M, P(O2) = 0.1 atm and pH = 3 , the cell potential at 25° C is (JEE 2011)
(a) 1.47 V (b) 1.77 V (c) 1.87 V (d) 1.57 V

Objective question II (More than one correct)


1. For the reduction of NO3- ion in an aqueous solution E° is +0.96 V, Values of E° for some metal
ions are given below
V2+(aq) + 2e- → V E° = -1.19V
Fe3+ (aq) + 3e- → Fe E° = -0.04 V
Au3+ (aq) + 3e- → Au E° = +1.40 V
Hg2+(aq) + 2e- → Hg E° = + 0.86 V
The pair (s) of metals that is (are) oxidized by NO-3 in aqueous solution is(are) (JEE 2009)
(a) V and Hg (b) Hg and Fe (c) Fe and Au (d) Fe and V

Comprehension –I
Tollen’s reagent is used for the detection of aldehydes. When a solution of AgNO3 is add to
glucose with NH4OH, then gluconic acid is formed,
Ag+ + e- → Ag; E°red = 0.80V
C6H12O6 + H2O → C6 H12O7 + 2H+ + 2e-;E°oxi = - 0.05V
gluconic acid
Ag(NH3) 2 + e- → Ag(s) + 2NH3; E°red = 0.337 V
 RT F 
Use2.303  F  0.0592 and RT  38.92 at 298 K  (JEE 2006)

1. 2Ag+ + C6H12O6 + H2O → 2Ag(s) + C6H12O7 + 2H+


Find ln K of this reaction.
(a) 66.13 (b) 58.38 (c) 28.30 (d) 46.29

2. When ammonia is added to the solution, pH is raised to 11. Which half-cell reaction is affected by
pH and by how much?
(a) Eoxi will increase by a factor of 0.65 form E°oxi
(b) Eoxi will decrease by a factor of 0.65 from E°oxi
(c) Ered will increase by a factor of 0.65 from E°red
(d) Ered will decrease by a factor of 0.65 from E°red

3. Ammonia is always added in this reaction. Which of the following must be incorrect?
(a) NH3 combines with Ag+ to form a complex
(b) Ag(NH3) 2 is a stronger oxidising reagent than Ag+
(c) In absence of NH3 silver salt of gluconic acid is formed
(d) NH3 has affected the standard reduction potential of glucose/gluconic acid electrode

Comprehension – II
Chemical reactions involve interaction of atoms and molecules. A large number of atoms /
molecules (approximately 6.023 ×1023) are present in a few grams of any chemical compound
varying with their with their atomic/molecular masses. To handle such large numbers conveniently,
the mole concept was introduced. This concept has implications in diverse areas such as analytical
chemistry, biochemistry, electrochemistry and radiochemistry. The following example illustrates a

CENTERS : MUMBAI /DELHI /AKOLA /LUCKNOW /NASHIK /PUNE /NAGPUR /BOKARO /DUBAI # 78
ELECTROCHEMISTRY Rg. 2018 - 2020

typical case, involving chemical / electrochemical reaction, which requires a clear understanding of
the mole concept.
A 4.0 M aqueous solution of NaCl is prepared and 500 mL of this solution is electrolysed. This
leads to the evolution of chlorine gas at one of the electrodes
(atomic mass: Na = 23, Hg = 200, 1F = 96500C). (JEE 2007)

4. The total number of moles of chlorine gas evolved is


(a) 0.5 (b) 1.0 (c) 2.0 (d) 3.0

5. If the cathode is a Hg electrode, the maximum weight (in gram) of amalgam formed from this
solution is
(a) 200 (b) 225 (c) 400 (d) 446

6. The total charge (coulombs) required for complete electrolysis is


(a) 24125 (b) 48250 (c) 96500 (d) 193000

Comprehension – III
Redox reactions play a pivotal role in chemistry and biology. The values of standard redox
potential (E°) of two half-cell reactions decide which way the reaction is expected to proceed. A
simple example is a Daniel cell in which zinc goes into solution and copper gets deposited. Given
below are a set of half-cell reactions (acidic medium) along with their E° (V with respect to normal
hydrogen electrode) values. Using this data obtain the correct explanations to following questions:

I2 + 2e- → 2I- E° = 0.54


- -
Cl2 + 2e → 2Cl E° = 1.36
Mn3+ + e- → Mn2+ E° = 1.50
Fe3+ + e- → Fe2+ E° = 0.77
O2+ 4H+ + 4e- → 2H2O E° = 1.23 (JEE 2007)

7. Among the following, identify the correct statement


(a) chloride ion is oxidised by O2
(b) Fe2+ is oxidised by iodine
(c) iodide ion is oxidised by chlorine
(d) Mn2+ is oxidised by chlorine

8. While Fe3+ is stable, Mn3+ is not stable in acid solution because


(a) O2 oxidises Mn2+ to Mn3+
(b) O2 oxidises both Mn2+ to Mn3+ and Fe2+ to Fe3+
(c) Fe3+ oxidises H2 O to O2
(d) Mn3+ oxidises H2O to O2

9. Sodium fusion extract, obtained from aniline, on treatment with iron(II) sulphate and H2SO4 in
presence of air gives a Prussian blue precipitate. the blue colour is due to the formation of
(a) Fe4[Fe(CN)6]3
(b) Fe3[Fe(CN)6]2
(c) Fe4[Fe(CN)6]2
(d) Fe3[Fe(CN)6]3
CENTERS : MUMBAI /DELHI /AKOLA /LUCKNOW /NASHIK /PUNE /NAGPUR /BOKARO /DUBAI # 79
ELECTROCHEMISTRY Rg. 2018 - 2020

Comprehension – IV
The concentration of potassium ions inside a biological cell is at least twenty times higher than the
outside. The resulting potential difference across the cell is important in several processes such as
transmission of nerve impulses and maintaining the ion balance. A simple model for such a
concentration cell involving a metal M is: (JEE 2010)
10. For the above cell
(a) Ecell< 0; ∆ G > 0
(b) Ecell> 0; ∆ G< 0
(c) Ecell< 0; ∆ G° > 0
(d) Ecell> 0; ∆ G° < 0
11. If the 0.05 molar solution of M+ is replaced by a 0.0025 molar M+ solution, then the magnitude of
the cell potential would be
(a) 35 mV (b) 70 mV (c) 140 mV (d) 700 mV
Subjective Questions

1. A current of 3.7 A is passed for 6 h between nickel electrodes in 0.5 L of a 2.0 M solution of
Ni(NO3)2. What will be the Molarity of solution at the end of electrolysis? (JEE 1978)

2. Consider the cell


Zn │ Zn2+ (aq) (1.0M)  Cu2+(aq) (1.0M) │ Cu
The standard reduction potentials are 0.350 V for
Cu2+ (aq) + 2e- → Cu and -0.763 V for Zn2+ (aq) + 2e- → Zn
(i) Write down the cell reaction
(ii) Calculate the emf of the cell
(iii) Is the cell reaction spontaneous or not (JEE 1982)

3. In electrolysis experiment current was passed for 5 h through two cells connected in series. The
first cell contains a solution of gold and the second contains copper sulphate solution. 9.85g of gold
was deposited in the first cell. If the oxidation number of gold is +3, find the amount of copper
deposited on the cathode of the second cell. Also calculate the magnitude of the current in ampere.
(At. wt. of Au = 197 and At. wt. of Cu = 63.5) (JEE 1983)

4. How long a current of 3A has to be passed through a solution of silver nitrate to coat a metal
surface of 80 cm2? (Density of silver is 10.5 g/cm3) with a 0.005 mm thick layer.
(JEE 1985)
5. Give reasons in one or two sentences.
“Anhydrous HCl is a bad conductor of electricity but aqueous HCl is a good conductor.”
(JEE 1985)
6. The emfof a cell corresponding to the reaction.
Zn(s) + 2H+ (aq) → Zn2+ (0.1M) + H2, (g, 1atm)
is 0.28 V at 25° C.
Write the half-cell reactions and calculate the pH of the solution at the hydrogen electrode.
E° (Zn2+ / Zn) = -0.76V, E° H  / H = 0 (JEE 1996)
2

7. During the discharge of a lead storage battery, the density of sulphuric acid fell from 1.294 to
1.139 g/mL. Sulphuric acid of density 1.294 g/mL is 39% H2SO4 by weight and that of density
1.139 g/mL is 20% H2SO4 by weight. The battery holds 3.5L of the acid and the volume remained
practically constant during the discharge.

CENTERS : MUMBAI /DELHI /AKOLA /LUCKNOW /NASHIK /PUNE /NAGPUR /BOKARO /DUBAI # 80
ELECTROCHEMISTRY Rg. 2018 - 2020

Calculate the number of ampere-hours for which the battery must have been used. The charging
and discharging reactions are
Pb + SO 24 = PbSO4 + 2e-(charging)
PbO2 + 4H+ + SO 24  + 2e- = PbSO4 + 2H2O (discharging) (JEE 1986)

8. A 100 watt, 110 V incandescent lamp is connected in series with an electrolyte cell containing
cadmium sulphate solution. What weight of cadmium will be deposited by the current flowing for
10 h? (JEE 1987)

9. A cell contains two hydrogen electrodes. The negative electrode is in contact with a solution of
10 -6 M hydrogen ions. The emf of the cell is 0.118 V at 25° C. Calculate the concentration of
hydrogen ions at the positive electrode. (JEE 1988)

10. In a fuel cell hydrogen and oxygen react to produces electricity. In the process hydrogen gas is
oxidised at the anode and oxygen at the cathode. If 67.2 L of H2 at STP react in 15 min, what is
the average current produced? If the entire current is used for electro-deposition of copper from
copper(II) Solution how many grams of copper will be deposited?
Anode reaction : H2 + 2OH-→ 2H2O +2e-
Cathode reaction: O2 + 2H2O +4e-→ 4OH- (JEE 1988)

11. An acidic solution of Cu2+ salt containing 0.4 g of Cu2+ is electrolysed until all the copper is
deposited. The electrolysis is continued for seven more minutes with the volume of solution kept
at 100 mL and the current at 1.2 A. Calculate the volume of gases evolved at NTP during the
entire electrolysis. (JEE 1989)

12. The standard reduction potential at 25° C of the reaction. 2H2O + 2e-⇌ H2 + 2OH-, is
-0.8277V. Calculate the equilibrium constant for the reaction,
2H2O ⇌ H3O+ + OH- at 25°C. (JEE 1989)

13. The standard reduction potential of Cu2+/Cu and Ag+/Ag electrodes are 0.337 and 0.799 V
respectively. Construct a galvanic cell using these electrodes so that its standard emf is positive.
For what concentration of Ag+ will the emf of the cell, at 25° C, be zero if the concentration of
Cu2+ is 0.01 M. (JEE 1990)

14. Calculate the quantity of electricity that would be required to reduce 12.3g of nitrobenzene to
aniline, if the current efficiency for the process is 50%. If the potential drop across the cell is 3.0V,
how much energy will be consumed? (JEE 1990)

15. Zinc granules are added in excess to a 500 mL of 1.0 M nickel nitrate solution at 25°C until the
equilibrium is reached. If the standard reduction potential of Zn2+/ Zn and Ni2+ / Ni are – 0.75V
and -0.24 V respectively.Find out the concentration of Ni2+ in solution at equilibrium
(JEE 1991)

16. A current of 1.70 A is passed through 300.0 mL of 0.160M solution of a ZnSO4 for 230 s with a
current efficiency of 90%. Find out the Molarity of Zn2+ after the deposition Zn. Assume the
volume of the solution to remain constant during the electrolysis. (JEE 1991)

CENTERS : MUMBAI /DELHI /AKOLA /LUCKNOW /NASHIK /PUNE /NAGPUR /BOKARO /DUBAI # 81
ELECTROCHEMISTRY Rg. 2018 - 2020

17. For the galvanic cell,


Ag │AgCl(s), KCl(0.2M) ǁ KBr(0.001M), AgBr(s) │ Ag
Calculate the emf generated and assign correct polarity to each electrode for a spontaneous process
after taking into account the cell reaction at 25°C. (JEE 1992)
[Ksp(AgCl) = 2.8 × 10-10 , Ksp (AgBr) =3.3 × 10-13]

18. An aqueous solution of NaCl on electrolysis gives H2(g), Cl2(g) and NaOH according to the
reaction.
2Cl-(aq) + 2H2O = 2OH-(aq) + H2(g) + Cl2(g)
A direct current of 25 A with a current efficiency of 62% is passed through 20 L of NaCl solution
(20% by weight). Write down the reactions taking place at the anode and cathode. How long will
it take to produce 1 kg of Cl2 ? What will be the Molarity of solution with respect to hydroxide
ion? (Assume no loss due to evaporation) (JEE 1992)

19. The standard reduction-potential for the half-cell


NO 3 (aq) + 2H+ + e- → NO2 (g) + H2O is 0.78 V
(i) Calculate the reduction-potential in 8MH+.
(ii) What will be the reduction-potential of the half-cell in a neutral solution? Assume all the
other species to be at unit concentration. (JEE 1993)

20. Chromium metal can be plated out from an acidic solution containing CrO3according to the
following equation.
CrO3(aq) + 6H+(aq) + 6e- → Cr(s) + 2H2O
Calculate
(i) How many gram of chromium will be plated out by 24,000 C and
(ii) How long will it take to plate out 1.5g of chromium by using 12.5 A current? (JEE 1993)

21. The standard reduction potential of the Ag+/Ag electrode at 298 K is 0.799V. Given that for Agl,
Ksp = 8.7 × 10 -17, evaluate the potential of the Ag+/Ag electrode in a saturated solution of Agl. Also
calculate the standard reduction potential of the I- / AgI/Ag electrode. (JEE 1994)
22. The Edison storage cell is represented as:
Fe(s) / FeO(s)/KOH(aq) /Ni2O3(s) / Ni(s)
The half-cell reactions are:
Ni2O3(s) + H2O(l) + 2e-⇌ 2NiO(s) + 2OH- E° = + 0.40 V
FeO(s) + H2O(l) + 2e-⇌ Fe(s) + 2OH- E° = - 0.87 V
(i) What is the cell reaction?
(ii) What is the cell emf? How does it depend on the concentration of KOH?
(iii) What is the maximum amount of electrical energy that can be obtained from one mole of
Ni2O3 ? (JEE 1994)

23. An excess of liquid mercury is added to an acidified solution of 1.0 × 10-3 M Fe3+. It is found that
5% of Fe3+remains at equilibrium at 25°C. Calculate E° (Hg2+/Hg) assuming that the only reaction
that occurs is
2Hg + 2Fe3+ → Hg 22  + 2Fe2+
Given, E°(Fe3+/Fe2+) = 0.77V (JEE 1995)

24. The standard reduction potential for Cu2+/Cu is +0.34V. Calculate the reduction potential at pH = 14
for the above couple. Ksp of Cu (OH)2is 1.0 ×10 -19. (JEE 1996)

CENTERS : MUMBAI /DELHI /AKOLA /LUCKNOW /NASHIK /PUNE /NAGPUR /BOKARO /DUBAI # 82
ELECTROCHEMISTRY Rg. 2018 - 2020

25. How many gram of silver could be plated out on a serving tray by electrolysis of a solution
containing silver in +1 oxidation state for a period of 8.0 h at a current of 8.46 A? What is the area
of the tray, if the thickness of the silver plating is 0.00254 cm? Density of silver is 10.5 g/cm3.
(JEE 1997)
26. Calculate the equilibrium constant for the reaction
Fe2+ + Ce4+⇌ Fe3+ + Ce3+
Given, E0(Ce4+/Ce3+) = 1.44 V, E°(Fe3+/Fe2+)=0.68V (JEE 1997)

27. Calculate the equilibrium constant for the reaction, 2Fe3+ + 3I-⇌2 Fe2++ I3-. The standard
reduction potentials in acidic conditions are 0.77 V and 0.54 V respectively for
Fe3+ /Fe2+ and I3-/I- couples. (JEE 1998)

28. Find the solubility product of a saturated solution of Ag2CrO4 in water at 298K, if the emf of the

cell Ag Ag  sat. Ag 2 CrO4 so ln. || Ag+ (0.1M) │ Ag is 0.164 V at 298 K. (JEE 1998)

29. A cell, Ag │Ag+|| Cu 2+ │ Cu, initially contains 1 M Ag+ and 1 M Cu2+ ions. Calculate the change
in the cell potential after the passage of 9.65 A of current for 1 h. (JEE 1999)

30. Copper sulphate solution (250mL) was electrolysed using a platinum anode and a copper cathode.
A constant current of 2 mA was passed for 16 min. It was found that after electrolysis the
absorbance of the solution was reduced to 50% of its original value. Calculate the concentration of
copper sulphate inthe solution to begin with. (JEE 2000)

31. The following electrochemical cell has been set-up :


Pt(1) │Fe3, Fe2+ (a=1) │ Ce4+, Ce3+ (a=1) │ Pt(2)
E° (Fe3+ , Fe2+) = 0.77 V and E° (Ce4+, Ce3+) = 1.61V
If an ammeter is connected between the two platinum electrodes, predict the direction of flow of
current, will the current increases or decreases with time? (JEE 2000)

32. The standard potential of the following cell is 0.23 V at 15°C and 0.21V at 35°C.
Pt│H2(g) │ HCl(aq) │AgCl(s) │Ag(s)
(i) Write the cell reaction.
(ii) Calculate ∆H° and ∆S° a for the cell reaction by assuming that these quantities remain
unchanged in the range 15°C to 35°C.
(iii) Calculate the solubility of AgCl in water at 25°C.
Given:The standard reduction potential of the Ag+(aq) / Ag(s) is 0.80 V at 25°C.
(JEE 2001)

33. (a) Will pH value of water be same at temperature 25°C and 4°C. Justify in not more than 2 or 3
sentences.
(b)Two students use same stock solution of ZnSO4 and a solution of CuSO4 . The emf of one cell
is 0.03 V higher than the other. The concentration of CuSO4 in the cell with higher emf value is
0.5M. Find out the concentration of CuSO4 in the other cell.
Given: 2.303RT/F = 0.06V (JEE 2003)

34. Find the equilibrium constant for the reaction


Cu2+ + In2+⇌ Cu+ + In3+
Given that (JEE 2004)
E° cu2  / cu  0.15V , E  In2  / In  0.4V ,
 0.42V
E° In3 / In 
CENTERS : MUMBAI /DELHI /AKOLA /LUCKNOW /NASHIK /PUNE /NAGPUR /BOKARO /DUBAI # 83
ELECTROCHEMISTRY Rg. 2018 - 2020

35. (a) Calculate ∆ of the following reaction:


Ag+(aq) + Cl- (aq) → AgCl(s)
Given:
∆G f (AgCl) -190 kJ/mol
∆G f (Cl)- -129kJ/mol
∆G f (Ag+)
77kJ/mol
Represent the above reaction in form of a cell.
Calculate E° of the cell. Find log10Kspof AgCl. (JEE 2005)
(b) 6.539×10-2 g of metallic Zn(u = 65.39) was added to 100mL of saturated solution of AgCl.
 Zn 2 
Calculate log10  2 . Given that
 Ag  
Ag+ + e- → Ag E° = 0.80V
2+ -
Zn + 2e → Zn E° = - 76V
Also find how many mole of Ag will be formed?

36. We have taken a saturated solution of AgBr,Ksp is 12 × 10 -14. If 10-7 M of AgNO3 are added to 1 L
of this solution, find conductivity (specific conductance) of this solution in terms of 10 -7 Sm-1 units.
(JEE 2006)
 3 2 1
Given,  Ag  6  10 Sm mol ,
 

  Br    8  10 3 Sm 2 mol 1 ,

  NO   7  10  3 Sm 2 mol 1 ,
 3

CENTERS : MUMBAI /DELHI /AKOLA /LUCKNOW /NASHIK /PUNE /NAGPUR /BOKARO /DUBAI # 84
ELECTROCHEMISTRY Rg. 2018 - 2020

ANSWER KEY

Foundation Builders Objective

1. (D) 2. (B) 3. (D) 4. (B) 5. (A) 6. (B) 7. (B)


8. (B) 9. (C) 10. (D) 11. (D) 12. (B) 13. (C) 14.(D)
15. (A) 16. (A) 17. (A) 18. (D) 19. (B) 20. (A) 21. (A)
22. (D) 23. (B) 24. (B) 25. (D) 26. (C) 27. (D) 28. (B)
29. (A) 30. (C) 31. (B) 32. (A) 33. (A) 34. (C) 35. (A)
36. (C) 37. (D) 38. (D) 39. (D) 40. (C) 41. (A) 42. (A)
43. (A) 44. (C) 45. (B) 46. (B) 47.(B) 48. (C) 49. (C)
50 .(A) 51. (B) 52. (B) 53. (C) 54. (C) 55. (C) 56. (B)
57. (B) 58. (D) 59. (D) 60. (B) 61. (A) 62. (A) 63. (A)
64. (B) 65. (D) 66. (C) 67. (C) 68. (D) 69. (B) 70. (B)
71. (A) 72. (C) 73. (A) 74. (B) 75. (C) 76. (C) 77. (D)
78. (D) 79. (B) 80. (D) 81. (D) 82. (A) 83. (C) 84.(B)
85. (C) 86. (B) 87. (C) 88. (C) 89. (D) 90. (C) 91. (B)
92. (B) 93. (A) 94. (C) 95. (C) 96. (A) 97. (D) 98. (A)
99. (D) 100. (D) 101. (C) 102. (C) 103. (C) 104. (B) 105. (D)
106. (B) 107. (C) 108. (B) 109. (B) 110. (B) 111. (D) 112. (A)
113. (D) 114. (C) 115. (A) 116. (B) 117. (D) 118.(D) 119. (C)
120. (D) 121. (B) 122. (C) 123. (B) 124. (A)

FOUNDATION BUILDERS SUBJECTIVE


3. (a) Spontaneous (b) Spontaneous
(c) Yes, reaction is non-spontaneous (e) Spontaneous
(f) Spontaneous

4. 1.61 V 5. 1.35 V 6 . 0.94 V 7.E° = -1.59V, non-spontaneous

8. Kc = 2 × 1026 9.  G° = -611.8 kJ 10. K = 10-14,  G° = +159.75 kJ


11.E° = 0.7828 V 12. [Zn2+]/[Cu2+] = 1.941 × 1037 13. . (a) 1.508 V, (b) MnO 24 

14. (i) E = 0.7505 V; (ii) E = 0.2859 V; (iii) E = -0.0206V 15. E = -2.419 V

16. E0cell = +0.01V, Ecell = -0.0785V,


correct representation is Pb | Pb2+(10-3M) || Sn2+(1M) | Sn

CENTERS : MUMBAI /DELHI /AKOLA /LUCKNOW /NASHIK /PUNE /NAGPUR /BOKARO /DUBAI # 85
ELECTROCHEMISTRY Rg. 2018 - 2020

17. [Cu2+] = 3 × 10 -12 M for E = 0, Yes 18. E = -0.81 eV 19. E = 1.159V

20. (i) pH = 4.24, (ii) pH = 5.08 21. n = 2 22. a2 = 0.1006M 23. E = 0.059
24. E = 0.395 V 25. Ksp = 1.01 × 10-16 26. [Br–] : [Cl–] = 1 : 200 27. -0.46 V

28.. pH = 6.61 29. Ka = 1.488 × 10-4 30.1.638 V 31.Kf = 8.227 × 1063

32. 5.39 x 1016 33.2.73 x 10 -8 34. H  334.85 kJ, S  241.25J / K 35.t = 193 sec.

36.n = 6 37.A = 114, Q = 5926.8C 38.Cu2+ = 0.08M, H+ = 0.04M, SO42- = 0.1M

39.Final weight = 9.6g, 0.01 Eq of acid 40.(a) V(O2) = 6.33 L, (b) V(O2 ) = 0.844 L

41.t = 93.66 sec. 42.  = 60% 43.44.[Cu2+ ] = 10-4 M

45. (a) 2A  60 min  60 min/s = 7200C ; (b) (7200/2)  (1/96500) = 0.0373 moles
(c) 0.0373  63.5 = 2.37 g ; (d) 0.337 V  2A = 0.67 W

46. 5.47 g 47. 11% 48. 350.4 hrs 49. 2M

50. 419 S cm 2 equivalent –1 51. 0.00046 S cm -1 ; 2174 ohm cm

52. (i) 6.25 × 10 5 ohm, (ii) 1.6 × 10 -6 amp 53. 1.549 cm –1

54. 101.8 ohm -1 cm 2 / gm equivalent 55. 0.0141 mho g eq –1 m 2 , 0.141 mho m -1

56. (i) 232 Mho cm 2 mol –1 , (ii) 116 Mho cm 2 equivalent –1

57. 0.865 58. 1.33 ×10 –4 gm/litre 59. 8.74 × 10 –11 mole 2 /litre 2

60.  = 0.435 , k = 6.7×10 –4 61. (i) 390.6 S cm2 mol–1 (ii) 12.32%

62. 1.067 S m –1 63. 523.2 ×10 –4 mho cm 2 mol –1 64.(i) 6.99 (ii) 1.042 × 10–14

(GET EQUIPPED FOR IIT – JEE)


SINGLE OPTION CORRECT

1. (D) 2 .(C) 3. (C) 4. (D) 5. (A) 6. (B) 7. (D)

8. (B) 9. (B) 10. (C) 11. (B) 12. (A) 13. (D) 14. (D)

15. (C) 16. (D) 17. (B) 18. (A) 19. (A) 20. (D)

CENTERS : MUMBAI /DELHI /AKOLA /LUCKNOW /NASHIK /PUNE /NAGPUR /BOKARO /DUBAI # 86
ELECTROCHEMISTRY Rg. 2018 - 2020

MORE THAN ONE OPTION CORRECT

21. (ABC) 22. (BC) 23. (BD) 24. (ABD) 25. (BC) 26. (BC) 27. (AC)

28. (AD) 29. (ABD) 30. (AB) 31. (AD) 32. (AB) 33. (AB) 34. (D)

35. (A) 36. (ABC) 37. (B) 38. (AD) 39. (ABC) 40. (AC)

41. (ABC) 42. (AC) 43. (AC) 44. (BC) 45. (AD)

COMPREHENSION TYPE

PASSAGE 1 1.(A) 2.(C) 3. (A) 4. (A) 5.(B)


PASSAGE 2 1.(C) 2.(B) 3. (D) 4.(B)
PASSAGE 3 1.(B) 2.(A) 3. (B) 4. (D) 5.(D)
PASSAGE 4 1.(D) 2.(C) 3. (B)
INTEGER TYPE
1. 2 2. 8 3. 7 4. 5

ADVANCE CONCEPT TESTERS

1. 265 Amp. hr. 2. Cu = 98.88%, Fe = 0.85% 3. 1250 s, 0.64 M

4. – 0.166 V 5. 43.456 g 6. 643.33 amp, 190.5 g

7. 1.39 V 8. E° = -0.15V 9. E° = 0.71 V

10. 0.373 V 11. K = 10268

12. Tl+ = 1.077 x 10-8 ; Co3+ =2.154 × 10 -8

13. V(O2) = 99.68 mL, V(H2) = 58.46 mL, Total vol. = 158.1 mL

14. 315.36 A 15. 57.5894 gm 16. 1.9 million year

17. 0.1456 ampere 18. 3.29 cm 19. 4.25×10 –2 metre

20. 0.1934 gm/litre 21. 1.062 x 10–2

22. (a) –0.124 V, (b) 7.1, (c) calomel electrode

23. -0.102V, 0.341V 24.(a) -0.117 V, (b) -0.089 V,(c) -0.061

25.1.238 26.2.014 × 105 dm3 27.KSP = 7.6872 × 10–17

CENTERS : MUMBAI /DELHI /AKOLA /LUCKNOW /NASHIK /PUNE /NAGPUR /BOKARO /DUBAI # 87
ELECTROCHEMISTRY Rg. 2018 - 2020

SELECTED IIT –JEE PROBLEMS FROM PREVIOUS YEARS

Objective Questions I
1. (a) 2. (c) 3.(c) 4.(d) 5.(c) 6.(d)
7.(b) 8.(c) 9. (a) 10. (a) 11. (b) 12. (b)
13.(c) 14. (a) 15. (c) 16. (b) 17. (b) 18. (b)
19. (d) 20. (d)

More than one may be correct


1. (a,b,d)

Comprehension Based Questions

Comprehension- I
1. (b) 2. (c) 3. (d)

Comprehension – II
4. (b) 5. (d) 6. (d)

Comprehension - III
7. (c) 8. (d) 9. (a)

Comprehension –IV
10. (b) 11. (c)

Subjective Questions

1. 1.172

2. (i) The cell reaction is


Zn + Cu 2+  Zn2+ + Cu
(ii) 1.113 V
(iii)spontaneous.

3. 0.80 A
4. 125s
5. In anhydrous state, HCl is not ionized and no charge carrier ions are available, hence bad conductor.
However, in aqueous solution, HCl is fully ionized producing H+ and Cl- and conducts electricity.
6. 8.6
7. 265Ah

8. 19.1g

9. 10-4 M

10. 190.50 g

11. O2(g) at NTP= 99.79 mL


H2 (g) at NTP = 58.46 mL

CENTERS : MUMBAI /DELHI /AKOLA /LUCKNOW /NASHIK /PUNE /NAGPUR /BOKARO /DUBAI # 88
ELECTROCHEMISTRY Rg. 2018 - 2020

12. 1.04× 10-14.


13. [Ag+] = 1.57 × 10 -9 M
14. 115800 C , 347.40kJ
15. K = 1.7 × 1017 , 1M.
16. 0.154 M
17. 0.037 V.
18. 1.4085M
19. (i) 0.887 V (ii) -0.0488 V
20. (i) 2.15 g (ii) 22.27 min
21. 0.324 V, -0.151 V
22. (i) Fe(s) + Ni2O3(s) → 2NiO(S) + FeO(s) E° = 1.27 V
(ii) emf is independent of concentration of KOH
(iii) 245.11 kJ is the maximum amount of obtainable energy.

23. 0.7926 V
24. -0.222V
25. 80.01 g , 3000 cm 2
26. 6.88 × 1012
27. K=5.89 × 107
28. 2.45 × 10-12
29. 0.01 V (decreased)
30. C = 8 × 10-5 M
 Fe3  Ce3 
31. Also E = E° - 0.0592 log  2   4 
 Fe  Ce 
As electrolysis proceeds, E will decrease and therefore, current will decrease.

1
32. (i) Cell reaction : H2  AgCl ( s )  Ag  H   Cl 
2
(ii) ∆ H° =-49.987 kJ
(iii) Ksp = 1.6 × 10 -10

33. (a) Kw is a function of temperature, [H+]will change with temperature.


(b) 0.05 M
34. K = 1010
35. (a) 0.59V
(b) 2 × 10 -3.
36. 55 (in terms of 10-7 Sm-1)

CENTERS : MUMBAI /DELHI /AKOLA /LUCKNOW /NASHIK /PUNE /NAGPUR /BOKARO /DUBAI # 89
ELECTROCHEMISTRY Rg. 2018 - 2020

CENTERS : MUMBAI /DELHI /AKOLA /LUCKNOW /NASHIK /PUNE /NAGPUR /BOKARO /DUBAI # 90

You might also like